HESI Pharmacology Study Guide

अब Quizwiz के साथ अपने होमवर्क और परीक्षाओं को एस करें!

219.) A health care provider instructs a client with rheumatoid arthritis to take ibuprofen (Motrin). The nurse reinforces the instructions, knowing that the normal adult dose for this client is which of the following? 1. 100 mg orally twice a day 2. 200 mg orally twice a day 3. 400 mg orally three times a day 4. 1000 mg orally four times a day

3. 400 mg orally three times a day Rationale: For acute or chronic rheumatoid arthritis or osteoarthritis, the normal oral adult dose is 400 to 800 mg three or four times daily.

Vasopressin therapy is prescribed for a client with a diagnosis of bleeding esophageal varices. The nurse is preparing to administer the medication to the client. Which essential item is needed during the administration of this medication? 1. An airway 2. A suction setup 3. A cardiac monitor 4. A tracheotomy set

3. A cardiac monitor The major action of vasopressin is constriction of the splanchnic blood flow. Continuous electrocardiogram and blood pressure monitoring are essential because of the constrictive effects of the medication on the coronary arteries.

191.) A child is brought to the emergency department for treatment of an acute asthma attack. The nurse prepares to administer which of the following medications first? 1. Oral corticosteroids 2. A leukotriene modifier 3. A β2 agonist 4. A nonsteroidal anti-inflammatory

3. A β2 agonist Rationale: In treating an acute asthma attack, a short acting β2 agonist such as albuterol (Proventil HFA) will be given to produce bronchodilation. Options 1, 2, and 4 are long-term control (preventive) medications.

233.) Diphenhydramine hydrochloride (Benadryl) is used in the treatment of allergic rhinitis for a hospitalized client with a chronic psychotic disorder. The client asks the nurse why the medication is being discontinued before hospital discharge. The nurse responds, knowing that: 1. Allergic symptoms are short in duration. 2. This medication promotes long-term extrapyramidal symptoms. 3. Addictive properties are enhanced in the presence of psychotropic medications. 4. Poor compliance causes this medication to fail to reach its therapeutic blood level.

3. Addictive properties are enhanced in the presence of psychotropic medications. Rationale: The addictive properties of diphenhydramine hydrochloride are enhanced when used with psychotropic medications. Allergic symptoms may not be short term and will occur if allergens are present in the environment. Poor compliance may be a problem with psychotic clients but is not the subject of the question. Diphenhydramine hydrochloride may be used for extrapyramidal symptoms and mild medication-induced movement disorders.

35.) The client has a PRN prescription for loperamide hydrochloride (Imodium). The nurse understands that this medication is used for which condition? 1. Constipation 2. Abdominal pain 3. An episode of diarrhea 4. Hematest-positive nasogastric tube drainage

3. An episode of diarrhea Rationale: Loperamide is an antidiarrheal agent. It is used to manage acute and also chronic diarrhea in conditions such as inflammatory bowel disease. Loperamide also can be used to reduce the volume of drainage from an ileostomy. It is not used for the conditions in options 1, 2, and 4.

174.) A client with portosystemic encephalopathy is receiving oral lactulose (Chronulac) daily. The nurse assesses which of the following to determine medication effectiveness? 1. Lung sounds 2. Blood pressure 3. Blood ammonia level 4. Serum potassium level

3. Blood ammonia level Rationale: Lactulose is a hyperosmotic laxative and ammonia detoxicant. It is used to prevent or treat portosystemic encephalopathy, including hepatic precoma and coma. It also is used to treat constipation. The medication retains ammonia in the colon (decreases the blood ammonia concentration), producing an osmotic effect. It promotes increased peristalsis and bowel evacuation, expelling ammonia from the colon.

96.) The nurse is caring for a postrenal transplant client taking cyclosporine (Sandimmune, Gengraf, Neoral). The nurse notes an increase in one of the client's vital signs, and the client is complaining of a headache. What is the vital sign that is most likely increased? 1. Pulse 2. Respirations 3. Blood pressure 4. Pulse oximetry

3. Blood pressure Rationale: Hypertension can occur in a client taking cyclosporine (Sandimmune, Gengraf, Neoral), and because this client is also complaining of a headache, the blood pressure is the vital sign to be monitoring most closely. Other adverse effects include infection, nephrotoxicity, and hirsutism. Options 1, 2, and 4 are unrelated to the use of this medication.

153.) A client is diagnosed with pulmonary embolism and is to be treated with streptokinase (Streptase). A nurse would report which priority data collection finding to the registered nurse before initiating this therapy? 1. Adventitious breath sounds 2. Temperature of 99.4° F orally 3. Blood pressure of 198/110 mm Hg 4. Respiratory rate of 28 breaths/min

3. Blood pressure of 198/110 mm Hg Rationale: Thrombolytic therapy is contraindicated in a number of preexisting conditions in which there is a risk of uncontrolled bleeding, similar to the case in anticoagulant therapy. Thrombolytic therapy also is contraindicated in severe uncontrolled hypertension because of the risk of cerebral hemorrhage. Therefore the nurse would report the results of the blood pressure to the registered nurse before initiating therapy. The findings in options 1, 2, and 4 may be present in the client with pulmonary embolism.

69.) A nurse who is administering bethanechol chloride (Urecholine) is monitoring for acute toxicity associated with the medication. The nurse checks the client for which sign of toxicity? 1. Dry skin 2. Dry mouth 3. Bradycardia 4. Signs of dehydration

3. Bradycardia Rationale: Toxicity (overdose) produces manifestations of excessive muscarinic stimulation such as salivation, sweating, involuntary urination and defecation, bradycardia, and severe hypotension. Treatment includes supportive measures and the administration of atropine sulfate subcutaneously or intravenously.

1) A nurse is caring for a client with hyperparathyroidism and notes that the client's serum calcium level is 13 mg/dL. Which medication should the nurse prepare to administer as prescribed to the client? 1. Calcium chloride 2. Calcium gluconate 3. Calcitonin (Miacalcin) 4. Large doses of vitamin D

3. Calcitonin (Miacalcin) Rationale: The normal serum calcium level is 8.6 to 10.0 mg/dL. This client is experiencing hypercalcemia. Calcium gluconate and calcium chloride are medications used for the treatment of tetany, which occurs as a result of acute hypocalcemia. In hypercalcemia, large doses of vitamin D need to be avoided. Calcitonin, a thyroid hormone, decreases the plasma calcium level by inhibiting bone resorption and lowering the serum calcium concentration.

49.) A client has been started on long-term therapy with rifampin (Rifadin). A nurse teaches the client that the medication: 1. Should always be taken with food or antacids 2. Should be double-dosed if one dose is forgotten 3. Causes orange discoloration of sweat, tears, urine, and feces 4. May be discontinued independently if symptoms are gone in 3 months

3. Causes orange discoloration of sweat, tears, urine, and feces Rationale: Rifampin should be taken exactly as directed as part of TB therapy. Doses should not be doubled or skipped. The client should not stop therapy until directed to do so by a health care provider. The medication should be administered on an empty stomach unless it causes gastrointestinal upset, and then it may be taken with food. Antacids, if prescribed, should be taken at least 1 hour before the medication. Rifampin causes orange-red discoloration of body secretions and will permanently stain soft contact lenses.

38.) An older client recently has been taking cimetidine (Tagamet). The nurse monitors the client for which most frequent central nervous system side effect of this medication? 1. Tremors 2. Dizziness 3. Confusion 4. Hallucinations

3. Confusion Rationale: Cimetidine is a histamine 2 (H2)-receptor antagonist. Older clients are especially susceptible to central nervous system side effects of cimetidine. The most frequent of these is confusion. Less common central nervous system side effects include headache, dizziness, drowsiness, and hallucinations.

16.) The clinic nurse is reviewing a teaching plan for the client receiving an antineoplastic medication. When implementing the plan, the nurse tells the client: 1. To take aspirin (acetylsalicylic acid) as needed for headache 2. Drink beverages containing alcohol in moderate amounts each evening 3. Consult with health care providers (HCPs) before receiving immunizations 4. That it is not necessary to consult HCPs before receiving a flu vaccine at the local health fair

3. Consult with health care providers (HCPs) before receiving immunizations Rationale: Because antineoplastic medications lower the resistance of the body, clients must be informed not to receive immunizations without a HCP's approval. Clients also need to avoid contact with individuals who have recently received a live virus vaccine. Clients need to avoid aspirin and aspirin-containing products to minimize the risk of bleeding, and they need to avoid alcohol to minimize the risk of toxicity and side effects.

148.) A client is taking cetirizine hydrochloride (Zyrtec). The nurse checks for which of the following side effects of this medication? 1. Diarrhea 2. Excitability 3. Drowsiness 4. Excess salivation

3. Drowsiness Rationale: A frequent side effect of cetirizine hydrochloride (Zyrtec), an antihistamine, is drowsiness or sedation. Others include blurred vision, hypertension (and sometimes hypotension), dry mouth, constipation, urinary retention, and sweating.

71.) After kidney transplantation, cyclosporine (Sand immune) is prescribed for a client. Which laboratory result would indicate an adverse effect from the use of this medication? 1. Decreased creatinine level 2. Decreased hemoglobin level 3. Elevated blood urea nitrogen level 4. Decreased white blood cell count

3. Elevated blood urea nitrogen level Rationale: Nephrotoxicity can occur from the use of cyclosporine (Sandimmune). Nephrotoxicity is evaluated by monitoring for elevated blood urea nitrogen (BUN) and serum creatinine levels. Cyclosporine is an immunosuppressant but does not depress the bone marrow.

154.) A nurse is reinforcing dietary instructions to a client who has been prescribed cyclosporine (Sandimmune). Which food item would the nurse instruct the client to avoid? 1. Red meats 2. Orange juice 3. Grapefruit juice 4. Green, leafy vegetables

3. Grapefruit juice Rationale: A compound present in grapefruit juice inhibits metabolism of cyclosporine. As a result, the consumption of grapefruit juice can raise cyclosporine levels by 50% to 100%, thereby greatly increasing the risk of toxicity. Grapefruit juice needs to be avoided. Red meats, orange juice, and green leafy vegetables are acceptable to consume.

60.) A nurse is planning to administer hydrochlorothiazide (HydroDIURIL) to a client. The nurse understands that which of the following are concerns related to the administration of this medication? 1. Hypouricemia, hyperkalemia 2. Increased risk of osteoporosis 3. Hypokalemia, hyperglycemia, sulfa allergy 4. Hyperkalemia, hypoglycemia, penicillin allergy

3. Hypokalemia, hyperglycemia, sulfa allergy Rationale: Thiazide diuretics such as hydrochlorothiazide are sulfa-based medications, and a client with a sulfa allergy is at risk for an allergic reaction. Also, clients are at risk for hypokalemia, hyperglycemia, hypercalcemia, hyperlipidemia, and hyperuricemia.

130.) The nurse is analyzing the laboratory results of a client with leukemia who has received a regimen of chemotherapy. Which laboratory value would the nurse specifically note as a result of the massive cell destruction that occurred from the chemotherapy? 1. Anemia 2. Decreased platelets 3. Increased uric acid level 4. Decreased leukocyte count

3. Increased uric acid level Rationale: Hyperuricemia is especially common following treatment for leukemias and lymphomas because chemotherapy results in a massive cell kill. Although options 1, 2, and 4 also may be noted, an increased uric acid level is related specifically to cell destruction.

122.) A client who has begun taking fosinopril (Monopril) is very distressed, telling the nurse that he cannot taste food normally since beginning the medication 2 weeks ago. The nurse provides the best support to the client by: 1. Telling the client not to take the medication with food 2. Suggesting that the client taper the dose until taste returns to normal 3. Informing the client that impaired taste is expected and generally disappears in 2 to 3 months 4. Requesting that the health care provider (HCP) change the prescription to another brand of angiotensin-converting enzyme (ACE) inhibitor

3. Informing the client that impaired taste is expected and generally disappears in 2 to 3 months Rationale: ACE inhibitors, such as fosinopril, cause temporary impairment of taste (dysgeusia). The nurse can tell the client that this effect usually disappears in 2 to 3 months, even with continued therapy, and provide nutritional counseling if appropriate to avoid weight loss. Options 1, 2, and 4 are inappropriate actions. Taking this medication with or without food does not affect absorption and action. The dosage should never be tapered without HCP approval and the medication should never be stopped abruptly.

5.) Mafenide acetate (Sulfamylon) is prescribed for the client with a burn injury. When applying the medication, the client complains of local discomfort and burning. Which of the following is the most appropriate nursing action? 1. Notifying the registered nurse 2. Discontinuing the medication 3. Informing the client that this is normal 4. Applying a thinner film than prescribed to the burn site

3. Informing the client that this is normal Rationale: Mafenide acetate is bacteriostatic for gram-negative and gram-positive organisms and is used to treat burns to reduce bacteria present in avascular tissues. The client should be informed that the medication will cause local discomfort and burning and that this is a normal reaction; therefore options 1, 2, and 4 are incorrect

243.) A hospitalized client is having the dosage of clonazepam (Klonopin) adjusted. The nurse should plan to: 1. Weigh the client daily. 2. Observe for ecchymosis. 3. Institute seizure precautions. 4. Monitor blood glucose levels.

3. Institute seizure precautions. Rationale: Clonazepam is a benzodiazepine used as an anticonvulsant. During initial therapy and during periods of dosage adjustment, the nurse should initiate seizure precautions for the client. Options 1, 2, and 4 are not associated with the use of this medication.

241.) A client with a history of simple partial seizures is taking clorazepate (Tranxene), and asks the nurse if there is a risk of addiction. The nurse's response is based on the understanding that clorazepate: 1. Is not habit forming, either physically or psychologically 2. Leads to physical tolerance, but only after 10 or more years of therapy 3. Leads to physical and psychological dependence with prolonged high-dose therapy 4. Can result in psychological dependence only, because of the nature of the medication

3. Leads to physical and psychological dependence with prolonged high-dose therapy Rationale: Clorazepate is classified as an anticonvulsant, antianxiety agent, and sedative-hypnotic (benzodiazepine). One of the concerns with clorazepate therapy is that the medication can lead to physical or psychological dependence with prolonged therapy at high doses. For this reason, the amount of medication that is readily available to the client at any one time is restricted. **Eliminate options 2 and 4 first because of the closed-ended word "only"**

52.) A client with tuberculosis is being started on antituberculosis therapy with isoniazid (INH). Before giving the client the first dose, a nurse ensures that which of the following baseline studies has been completed? 1. Electrolyte levels 2. Coagulation times 3. Liver enzyme levels 4. Serum creatinine level

3. Liver enzyme levels Rationale: INH therapy can cause an elevation of hepatic enzyme levels and hepatitis. Therefore, liver enzyme levels are monitored when therapy is initiated and during the first 3 months of therapy. They may be monitored longer in the client who is greater than age 50 or abuses alcohol.

59.) A client is diagnosed with an acute myocardial infarction and is receiving tissue plasminogen activator, alteplase (Activase, tPA). Which action is a priority nursing intervention? 1. Monitor for renal failure. 2. Monitor psychosocial status. 3. Monitor for signs of bleeding. 4. Have heparin sodium available.

3. Monitor for signs of bleeding. Rationale: Tissue plasminogen activator is a thrombolytic. Hemorrhage is a complication of any type of thrombolytic medication. The client is monitored for bleeding. Monitoring for renal failure and monitoring the client's psychosocial status are important but are not the most critical interventions. Heparin is given after thrombolytic therapy, but the question is not asking about follow-up medications.

151.) A client is being treated for acute congestive heart failure with intravenously administered bumetanide. The vital signs are as follows: blood pressure, 100/60 mm Hg; pulse, 96 beats/min; and respirations, 24 breaths/min. After the initial dose, which of the following is the priority assessment? 1. Monitoring weight loss 2. Monitoring temperature 3. Monitoring blood pressure 4. Monitoring potassium level

3. Monitoring blood pressure Rationale: Bumetanide is a loop diuretic. Hypotension is a common side effect associated with the use of this medication. The other options also require assessment but are not the priority. **priority ABCs—airway, breathing, and circulation**

201.) A nurse is preparing to administer eardrops to an infant. The nurse plans to: 1. Pull up and back on the ear and direct the solution onto the eardrum. 2. Pull down and back on the ear and direct the solution onto the eardrum. 3. Pull down and back on the ear and direct the solution toward the wall of the canal. 4. Pull up and back on the ear lobe and direct the solution toward the wall of the canal.

3. Pull down and back on the ear and direct the solution toward the wall of the canal. Rationale: When administering eardrops to an infant, the nurse pulls the ear down and straight back. In the adult or a child older than 3 years, the ear is pulled up and back to straighten the auditory canal. The medication is administered by aiming it at the wall of the canal rather than directly onto the eardrum.

The nurse is reading a computer print out of the results of a cerebrospinal fluid (CSF) analysis performed on an adult client who underwent a lumbar puncture. The nurse knows that a reported value of zero is normal for which substance in CSF? 1. Protein 2. Glucose 3. Red Blood Cells 4. White Blood Cells

3. Red Blood Cells The adult with normal CSF has no red blood cells in the CSF. The client may have small numbers of white blood cells (0-3 per mm^3). Protein (15-45 mg/DL) and glucose (40-80 mg/DL) normally are present in the CSF.

37.) The client has begun medication therapy with pancrelipase (Pancrease MT). The nurse evaluates that the medication is having the optimal intended benefit if which effect is observed? 1. Weight loss 2. Relief of heartburn 3. Reduction of steatorrhea 4. Absence of abdominal pain

3. Reduction of steatorrhea Rationale: Pancrelipase (Pancrease MT) is a pancreatic enzyme used in clients with pancreatitis as a digestive aid. The medication should reduce the amount of fatty stools (steatorrhea). Another intended effect could be improved nutritional status. It is not used to treat abdominal pain or heartburn. Its use could result in weight gain but should not result in weight loss if it is aiding in digestion.

137.) A nurse is reinforcing instructions for a client regarding intranasal desmopressin acetate (DDAVP). The nurse tells the client that which of the following is a side effect of the medication? 1. Headache 2. Vulval pain 3. Runny nose 4. Flushed skin

3. Runny nose Rationale: Desmopressin administered by the intranasal route can cause a runny or stuffy nose. Headache, vulval pain, and flushed skin are side effects if the medication is administered by the intravenous (IV) route.

111.) A client arrives at the health care clinic and tells the nurse that he has been doubling his daily dosage of bupropion hydrochloride (Wellbutrin) to help him get better faster. The nurse understands that the client is now at risk for which of the following? 1. Insomnia 2. Weight gain 3. Seizure activity 4. Orthostatic hypotension

3. Seizure activity Rationale: Bupropion does not cause significant orthostatic blood pressure changes. Seizure activity is common in dosages greater than 450 mg daily. Bupropion frequently causes a drop in body weight. Insomnia is a side effect, but seizure activity causes a greater client risk.

95.) The nurse is reviewing the results of serum laboratory studies drawn on a client with acquired immunodeficiency syndrome who is receiving didanosine (Videx). The nurse interprets that the client may have the medication discontinued by the health care provider if which of the following significantly elevated results is noted? 1. Serum protein 2. Blood glucose 3. Serum amylase 4. Serum creatinine

3. Serum amylase Rationale: Didanosine (Videx) can cause pancreatitis. A serum amylase level that is increased 1.5 to 2 times normal may signify pancreatitis in the client with acquired immunodeficiency syndrome and is potentially fatal. The medication may have to be discontinued. The medication is also hepatotoxic and can result in liver failure.

98.) The nurse is assigned to care for a client with cytomegalovirus retinitis and acquired immunodeficiency syndrome who is receiving foscarnet. The nurse should check the latest results of which of the following laboratory studies while the client is taking this medication? 1. CD4 cell count 2. Serum albumin 3. Serum creatinine 4. Lymphocyte count

3. Serum creatinine Rationale: Foscarnet is toxic to the kidneys. Serum creatinine is monitored before therapy, two to three times per week during induction therapy, and at least weekly during maintenance therapy. Foscarnet may also cause decreased levels of calcium, magnesium, phosphorus, and potassium. Thus these levels are also measured with the same frequency.

211.) A client with epilepsy is taking the prescribed dose of phenytoin (Dilantin) to control seizures. A phenytoin blood level is drawn, and the results reveal a level of 35 mcg/ml. Which of the following symptoms would be expected as a result of this laboratory result? 1. Nystagmus 2. Tachycardia 3. Slurred speech 4. No symptoms, because this is a normal therapeutic level

3. Slurred speech Rationale: The therapeutic phenytoin level is 10 to 20 mcg/mL. At a level higher than 20 mcg/mL, involuntary movements of the eyeballs (nystagmus) appear. At a level higher than 30 mcg/mL, ataxia and slurred speech occur.

A client experiences major depression after a stroke. The physician prescribes venlafaxine (Effexor) extended-release capsules by mouth, once daily. The client has difficulty swallowing pills since his stroke. How should the nurse intervene? 1. Request that the physician prescribe another form of the drug. 2. Place the capsule in water, allow it to dissolve, and then administer it to the client. 3. Sprinkle the contents of the capsule over applesauce and administer it to the client. 4.Tell the client to chew the capsule after placing it in his mouth. Like

3. Sprinkle the contents of the capsule over applesauce and administer it to the client. The extended-release capsule can be opened and its contents sprinkled on applesauce immediately before administration. The client should drink water after swallowing the mixture to ensure that all of the pellets are administered. The extended-release capsules of venlafaxine shouldn't be divided, crushed, chewed, or placed in water. The physician doesn't need to prescribe another form of the drug.

187.) A clinic nurse prepares to administer an MMR (measles, mumps, rubella) vaccine to a child. How is this vaccine best administered? 1. Intramuscularly in the deltoid muscle 2. Subcutaneously in the gluteal muscle 3. Subcutaneously in the outer aspect of the upper arm 4. Intramuscularly in the anterolateral aspect of the thigh

3. Subcutaneously in the outer aspect of the upper arm Rationale: The MMR vaccine is administered subcutaneously in the outer aspect of the upper arm. The gluteal muscle is most often used for intramuscular injections. The MMR vaccine is not administered by the intramuscular route.

46.) A postoperative client has received a dose of naloxone hydrochloride for respiratory depression shortly after transfer to the nursing unit from the postanesthesia care unit. After administration of the medication, the nurse checks the client for: 1. Pupillary changes 2. Scattered lung wheezes 3. Sudden increase in pain 4. Sudden episodes of diarrhea

3. Sudden increase in pain Rationale: Naloxone hydrochloride is an antidote to opioids and may also be given to the postoperative client to treat respiratory depression. When given to the postoperative client for respiratory depression, it may also reverse the effects of analgesics. Therefore, the nurse must check the client for a sudden increase in the level of pain experienced. Options 1, 2, and 4 are not associated with this medication.

58.) Isosorbide mononitrate (Imdur) is prescribed for a client with angina pectoris. The client tells the nurse that the medication is causing a chronic headache. The nurse appropriately suggests that the client: 1. Cut the dose in half. 2. Discontinue the medication. 3. Take the medication with food. 4. Contact the health care provider (HCP).

3. Take the medication with food. Rationale: Isosorbide mononitrate is an antianginal medication. Headache is a frequent side effect of isosorbide mononitrate and usually disappears during continued therapy. If a headache occurs during therapy, the client should be instructed to take the medication with food or meals. It is not necessary to contact the HCP unless the headaches persist with therapy. It is not appropriate to instruct the client to discontinue therapy or adjust the dosages.

22.) A nurse is caring for a client after thyroidectomy and notes that calcium gluconate is prescribed for the client. The nurse determines that this medication has been prescribed to: 1. Treat thyroid storm. 2. Prevent cardiac irritability. 3. Treat hypocalcemic tetany. 4. Stimulate the release of parathyroid hormone.

3. Treat hypocalcemic tetany. Rationale: Hypocalcemia can develop after thyroidectomy if the parathyroid glands are accidentally removed or injured during surgery. Manifestations develop 1 to 7 days after surgery. If the client develops numbness and tingling around the mouth, fingertips, or toes or muscle spasms or twitching, the health care provider is notified immediately. Calcium gluconate should be kept at the bedside.

a 27 y/o primigravida asks the nurse how much IRON she needs during her pregnancy.

30 mg

a physician orders ATENOLOL for a client with type1 diabetes mellitus. Which response is best?

" ATENOLOL may mask symptoms of hypoglycemia"

A client with iron deficiency anemia is scheduled for discharge. Which instruction about prescribed FERROUS GLUCONATE therapy should the nurse include in the teaching plan?

"Take the medication on an empty stomach

Akinesia

"Without movement." Absence or poverty of movement that results in a mask like facial expression and impaired postural reflexes.

ARBs

(Cozaar) Losartan, valsartan, less symptoms than ACE

Mineralocorticoids

(e.g. aldosterone) are secreted by the adrenal cortex to increase the rate of sodium reabsorption by the kidneys, thereby increasing blood levels of sodium. Result is increased water reabsorption by the kidneys and increased blood volume. mineralocorticoids increase the excretion of potassium and hydrogen ions into the urine, thereby decreeing blood levels of potassium and hydrogen ions. In adrenal cortex insufficiency, it is necessary to replace a glucocorticoid and, in some individuals, a mineralocorticoid such as fludrocortisone.

meperidine hydrochloride (Demerol)

**Use with caution (or not at all) in elderly patients and those who require long-term analgesia or who have kidney dysfunction. **Active metabolite, normeperidine, can accumulate to toxic levels and lead to seizures, so it's def not for long-term pain treatment. Indications: Acute pain during postop, or in EDs for acute migraines. Methods of admin: Oral, injectable forms

Antiepileptic Drugs used to treat status epilepticus

*Diazepam* Lorazepam Phenobarbital Phenytoin

Mood-stabilizing drugs

- Used to treat bipolar illness (cycles of mania, hypomania, and depression). Catecholamines play an important pathophysiologic role in the development of mania. Serotonin also appears to be involved. Lithium has been in use for many years and is still used to effectively alleviate the symptoms of acute mania.

Septic Shock

- Usually results from a gram-negative bacteremia that leads to circulatory insufficiency. Inadequate tissue perfusion generally results in hypotension, oliguria, tachycardia, elevated temp, and tachypnea. - Mechanism:bacterial substances interact with body cell membranes and systems, esp. coag and the complete system, resulting in injury to cells and alterations in blood flow in the body.

Glucocorticoids

- Widely used for anti-inflammatory and immunomodulating effects. - Patients receiving systemic corticosteroid preps for non endocrine disorders are at risk for adverse effects

Thiazide Diuretics

- chlorothiazide, hydrochlorothiazide - Interfere with sodium ion transport at loop of Henle and inhibit carbonic anhydrase activity at distal tubule sites - Metolazone is a thiazide-like diuretic.

Loop-diuretics

- ethacrynic acid, furosemide, bumetanide, torsemide - Interfere with active transport of sodium ions in loop of Henle and inhibit sodium chloride and water reabsorption at proximal tubule sites. - May be given IV

Glucocorticoids are used...

- in replacement therapy for adrenocortical insufficiency - To treat: allergic reactions, anaphylactic reactions not responsive to other therapies, collagen disorders such as SLE, deem conditions, hematologic disorders, neoplastic disease, ophthalmic disorders, respiratory disorders, rheumatic disorders, and septic shock. - To reduce inflammation post head trauma, may have negative outcomes.

Haloperidol

-Indicated primarily for long-term treatment of psychosis. Haloperidol decanoate is useful for patients with schizophrenia who have been non adherent with their drug regimen. -Contraindicated in patients who have shown a hypersensitivity reaction, those in a comatose state, those taking large amounts of CNS depressants, and those with Parkinson's. -Can cause extrapyramidal symptoms as well as tar dive dyskinesia.

Memantine (Namenda)

-Not a cholinergic drug -Used for Alzheimer's dementia -NMDA receptor antagonist owing to its inhibitory activity sat the NMDA receptors in the CNS. Stimulation of these reeptors is believed to be part of the alzheimer's process. -Memantine blocks this stimulation and thereby helps to reduce or arrest the patient's degenerative cognitive symptoms. -As with all other currently available medications for this debilitating illness, the effects of this drug are likely to be temporary but may still afford some improvement in quality of life and general functioning for some patients. -Only current contraindication is known drug allergy. -Adverse effects are relatively uncommon but include hypotension, headache, GI upset, musculoskeletal pain, dyspnea, ataxia, and fatigue. -No clearly defined interactions.

Phentermine

-Symptathomimetic anorexiant that is structurally related to amphetamines but with much lower abuse potential.

Fluoxetine (Prozac)

-Top prescribed antidepressant in the US -Indicated for depression, bulimia, OCD, panic disorders, premenstrual dysphoric disorder. -Contraindications include known allergy and concurrent MAOI use. -Adverse effects include anxiety, dizziness, drowsiness, insomnia -Interacting drugs include benzos, buspirone, antipsychotics, and propafenone.

Adverse effects of alpha-adrenergics

-Unwanted CNS effects of the alpha-adrenergic drugs include: headaches restlessness excitement insomnia euphoria -Possible cardiovascular include: chest pain vasoconstriction hypertension tachycardia palpitations dysrhythmias -Other: anorexia dry mouth N/V taste changes (rarely)

Ophthalmic Indications for adrenergics

-Work in much the same way as nasal decongestants, except that they affect the vasculature of the eye. Stimulate alpha-adrenergic receptors located on small arterioles in the eye and temporarily relieve conjunctival congestion by causing arteriolar vasoconstriction. -Can also be used to reduce intraocular pressure, which makes them useful in the treatment of open-angle glaucoma. Can also dilate pupils, which makes them useful for diagnostic eye exams. Produce these effects by stimulating alpha- or beta2-adrenergic receptors, or both. -Two adrenergics used for this purpose are epinephrine and dipivefrin.

CYCLOSPORINE and METHOTREXATE are prescribed for a client with severe rheumatoid arthritis. which points should the nurse address when teaching this client? SATA

-keep well hydrated to maximize the therapeutic effects of methotrexate -avoid st. johns wort, echinacea, and melatonin, as they interfere with immunosuppression -both methotrexate and cyclosporine suppress the immune system

Interactions with anticholinergics

-other anticholinergics (amantadine, antihistamines, tricyclic antidepressants); can result in additive effects -phenothiazines; can result in reduced antipsychotic effects -digoxin; increases effects of dig

fenoldopam (Corlopam)

-peripheral dopamine 1 agonist indicated for parenteral use in lowering BP. -produces effects by inducing arteriolar vasodilation mainly through stimulation f D1 receptors. Appears to be as effective as sodium nitroprusside for short-term treatment of severe hypertension and may have beneficial effects on renal function bc it increases renal blood flow.

Midodrine (ProAmatine)

-prodrug that is converted int eh liver to its active form, desglymidodrine. This active metabolite is responsible for the primary pharmacologic action of midodrine, which is alpha1-adrenergic receptor stimulation. This alpha1 stimulation causes constriction of both arterioles and veins, resulting in peripheral vasoconstriction.. -Primarily indicated for treatment of symptomatic orthostatic hypotension. -Last dose of day should not be given after 6PM

Corticosteroids suppress HPA axis and endogenous release of cortisol.

...

How likely is a person allergic to a penicillin to have cross-sensitivity to second-and third-generation cephalosporins?

0.05% or less

Therapeutic range digoxin:

0.5-2

31.) A community health nurse visits a client at home. Prednisone 10 mg orally daily has been prescribed for the client and the nurse reinforces teaching for the client about the medication. Which statement, if made by the client, indicates that further teaching is necessary? 1. "I can take aspirin or my antihistamine if I need it." 2. "I need to take the medication every day at the same time." 3. "I need to avoid coffee, tea, cola, and chocolate in my diet." 4. "If I gain more than 5 pounds a week, I will call my doctor."

1. "I can take aspirin or my antihistamine if I need it." Rationale: Aspirin and other over-the-counter medications should not be taken unless the client consults with the health care provider (HCP). The client needs to take the medication at the same time every day and should be instructed not to stop the medication. A slight weight gain as a result of an improved appetite is expected, but after the dosage is stabilized, a weight gain of 5 lb or more weekly should be reported to the HCP. Caffeine-containing foods and fluids need to be avoided because they may contribute to steroid-ulcer development.

220.) A adult client with muscle spasms is taking an oral maintenance dose of baclofen (Lioresal). The nurse reviews the medication record, expecting that which dose should be prescribed? 1. 15 mg four times a day 2. 25 mg four times a day 3. 30 mg four times a day 4. 40 mg four times a day

1. 15 mg four times a day Rationale: Baclofen is dispensed in 10- and 20-mg tablets for oral use. Dosages are low initially and then gradually increased. Maintenance doses range from 15 to 20 mg administered three or four times a day.

115.) A client received 20 units of NPH insulin subcutaneously at 8:00 AM. The nurse should check the client for a potential hypoglycemic reaction at what time? 1. 5:00 PM 2. 10:00 AM 3. 11:00 AM 4. 11:00 PM

1. 5:00 PM Rationale: NPH is intermediate-acting insulin. Its onset of action is 1 to 2½ hours, it peaks in 4 to 12 hours, and its duration of action is 24 hours. Hypoglycemic reactions most likely occur during peak time.

10.) The clinic nurse is performing an admission assessment on a client. The nurse notes that the client is taking azelaic acid (Azelex). Because of the medication prescription, the nurse would suspect that the client is being treated for: 1. Acne 2. Eczema 3. Hair loss 4. Herpes simplex

1. Acne Rationale: Azelaic acid is a topical medication used to treat mild to moderate acne. The acid appears to work by suppressing the growth of Propionibacterium acnes and decreasing the proliferation of keratinocytes. Options 2, 3, and 4 are incorrect.

145.) A nurse has a prescription to give a client albuterol (Proventil HFA) (two puffs) and beclomethasone dipropionate (Qvar) (nasal inhalation, two puffs), by metered-dose inhaler. The nurse administers the medication by giving the: 1. Albuterol first and then the beclomethasone dipropionate 2. Beclomethasone dipropionate first and then the albuterol 3. Alternating a single puff of each, beginning with the albuterol 4. Alternating a single puff of each, beginning with the beclomethasone dipropionate

1. Albuterol first and then the beclomethasone dipropionate Rationale: Albuterol is a bronchodilator. Beclomethasone dipropionate is a glucocorticoid. Bronchodilators are always administered before glucocorticoids when both are to be given on the same time schedule. This allows for widening of the air passages by the bronchodilator, which then makes the glucocorticoid more effective.

26.) Glimepiride (Amaryl) is prescribed for a client with diabetes mellitus. A nurse reinforces instructions for the client and tells the client to avoid which of the following while taking this medication? 1. Alcohol 2. Organ meats 3. Whole-grain cereals 4. Carbonated beverages

1. Alcohol Rationale: When alcohol is combined with glimepiride (Amaryl), a disulfiram-like reaction may occur. This syndrome includes flushing, palpitations, and nausea. Alcohol can also potentiate the hypoglycemic effects of the medication. Clients need to be instructed to avoid alcohol consumption while taking this medication. The items in options 2, 3, and 4 do not need to be avoided.

142.) A health care provider has written a prescription for ranitidine (Zantac), once daily. The nurse should schedule the medication for which of the following times? 1. At bedtime 2. After lunch 3. With supper 4. Before breakfast

1. At bedtime Rationale: A single daily dose of ranitidine is usually scheduled to be given at bedtime. This allows for a prolonged effect, and the greatest protection of the gastric mucosa. **recall that ranitidine suppresses secretions of gastric acids**

100.) Saquinavir (Invirase) is prescribed for the client who is human immunodeficiency virus seropositive. The nurse reinforces medication instructions and tells the client to: 1. Avoid sun exposure. 2. Eat low-calorie foods. 3. Eat foods that are low in fat. 4. Take the medication on an empty stomach.

1. Avoid sun exposure. Rationale: Saquinavir (Invirase) is an antiretroviral (protease inhibitor) used with other antiretroviral medications to manage human immunodeficiency virus infection. Saquinavir is administered with meals and is best absorbed if the client consumes high-calorie, high-fat meals. Saquinavir can cause photosensitivity, and the nurse should instruct the client to avoid sun exposure.

33.) The home health care nurse is visiting a client who was recently diagnosed with type 2 diabetes mellitus. The client is prescribed repaglinide (Prandin) and metformin (Glucophage) and asks the nurse to explain these medications. The nurse should reinforce which instructions to the client? Select all that apply. 1. Diarrhea can occur secondary to the metformin. 2. The repaglinide is not taken if a meal is skipped. 3. The repaglinide is taken 30 minutes before eating. 4. Candy or another simple sugar is carried and used to treat mild hypoglycemia episodes. 5. Metformin increases hepatic glucose production to prevent hypoglycemia associated with repaglinide. 6. Muscle pain is an expected side effect of metformin and may be treated with acetaminophen (Tylenol).

1. Diarrhea can occur secondary to the metformin. 2. The repaglinide is not taken if a meal is skipped. 3. The repaglinide is taken 30 minutes before eating. 4. Candy or another simple sugar is carried and used to treat mild hypoglycemia episodes. Rationale: Repaglinide is a rapid-acting oral hypoglycemic agent that stimulates pancreatic insulin secretion that should be taken before meals, and that should be withheld if the client does not eat. Hypoglycemia is a side effect of repaglinide and the client should always be prepared by carrying a simple sugar with her or him at all times. Metformin is an oral hypoglycemic given in combination with repaglinide and works by decreasing hepatic glucose production. A common side effect of metformin is diarrhea. Muscle pain may occur as an adverse effect from metformin but it might signify a more serious condition that warrants health care provider notification, not the use of acetaminophen.

116.) A nurse administers a dose of scopolamine (Transderm-Scop) to a postoperative client. The nurse tells the client to expect which of the following side effects of this medication? 1. Dry mouth 2. Diaphoresis 3. Excessive urination 4. Pupillary constriction

1. Dry mouth Rationale: Scopolamine is an anticholinergic medication for the prevention of nausea and vomiting that causes the frequent side effects of dry mouth, urinary retention, decreased sweating, and dilation of the pupils. The other options describe the opposite effects of cholinergic-blocking agents and therefore are incorrect.

93.) The client who is human immunodeficiency virus seropositive has been taking stavudine (d4t, Zerit). The nurse monitors which of the following most closely while the client is taking this medication? 1. Gait 2. Appetite 3. Level of consciousness 4. Hemoglobin and hematocrit blood levels

1. Gait Rationale: Stavudine (d4t, Zerit) is an antiretroviral used to manage human immunodeficiency virus infection in clients who do not respond to or who cannot tolerate conventional therapy. The medication can cause peripheral neuropathy, and the nurse should monitor the client's gait closely and ask the client about paresthesia. Options 2, 3, and 4 are unrelated to the use of the medication.

125.) A nurse is preparing to administer digoxin (Lanoxin), 0.125 mg orally, to a client with heart failure. Which vital sign is most important for the nurse to check before administering the medication? 1. Heart rate 2. Temperature 3. Respirations 4. Blood pressure

1. Heart rate Rationale: Digoxin is a cardiac glycoside that is used to treat heart failure and acts by increasing the force of myocardial contraction. Because bradycardia may be a clinical sign of toxicity, the nurse counts the apical heart rate for 1 full minute before administering the medication. If the pulse rate is less than 60 beats/minute in an adult client, the nurse would withhold the medication and report the pulse rate to the registered nurse, who would then contact the health care provider.

43.) A histamine (H2)-receptor antagonist will be prescribed for a client. The nurse understands that which medications are H2-receptor antagonists? Select all that apply. 1. Nizatidine (Axid) 2. Ranitidine (Zantac) 3. Famotidine (Pepcid) 4. Cimetidine (Tagamet) 5. Esomeprazole (Nexium) 6. Lansoprazole (Prevacid)

1. Nizatidine (Axid) 2. Ranitidine (Zantac) 3. Famotidine (Pepcid) 4. Cimetidine (Tagamet) Rationale: H2-receptor antagonists suppress secretion of gastric acid, alleviate symptoms of heartburn, and assist in preventing complications of peptic ulcer disease. These medications also suppress gastric acid secretions and are used in active ulcer disease, erosive esophagitis, and pathological hypersecretory conditions. The other medications listed are proton pump inhibitors. H2-receptor antagonists medication names end with -dine. Proton pump inhibitors medication names end with -zole.

109.) A client taking buspirone (BuSpar) for 1 month returns to the clinic for a follow-up visit. Which of the following would indicate medication effectiveness? 1. No rapid heartbeats or anxiety 2. No paranoid thought processes 3. No thought broadcasting or delusions 4. No reports of alcohol withdrawal symptoms

1. No rapid heartbeats or anxiety Rationale: Buspirone hydrochloride is not recommended for the treatment of drug or alcohol withdrawal, paranoid thought disorders, or schizophrenia (thought broadcasting or delusions). Buspirone hydrochloride is most often indicated for the treatment of anxiety and aggression.

171.) A nurse is preparing to administer furosemide (Lasix) to a client with a diagnosis of heart failure. The most important laboratory test result for the nurse to check before administering this medication is: 1. Potassium level 2. Creatinine level 3. Cholesterol level 4. Blood urea nitrogen

1. Potassium level Rationale: Furosemide is a loop diuretic. The medication causes a decrease in the client's electrolytes, especially potassium, sodium, and chloride. Administering furosemide to a client with low electrolyte levels could precipitate ventricular dysrhythmias. Options 2 and 4 reflect renal function. The cholesterol level is unrelated to the administration of this medication.

30.) A client with diabetes mellitus visits a health care clinic. The client's diabetes mellitus previously had been well controlled with glyburide (DiaBeta) daily, but recently the fasting blood glucose level has been 180 to 200 mg/dL. Which medication, if added to the client's regimen, may have contributed to the hyperglycemia? 1. Prednisone 2. Phenelzine (Nardil) 3. Atenolol (Tenormin) 4. Allopurinol (Zyloprim)

1. Prednisone Rationale: Prednisone may decrease the effect of oral hypoglycemics, insulin, diuretics, and potassium supplements. Option 2, a monoamine oxidase inhibitor, and option 3, a β-blocker, have their own intrinsic hypoglycemic activity. Option 4 decreases urinary excretion of sulfonylurea agents, causing increased levels of the oral agents, which can lead to hypoglycemia.

102.) A client with human immunodeficiency virus is taking nevirapine (Viramune). The nurse should monitor for which adverse effects of the medication? Select all that apply. 1. Rash 2. Hepatotoxicity 3. Hyperglycemia 4. Peripheral neuropathy 5. Reduced bone mineral density

1. Rash 2. Hepatotoxicity Rationale: Nevirapine (Viramune) is a non-nucleoside reverse transcriptase inhibitors (NRTI) that is used to treat HIV infection. It is used in combination with other antiretroviral medications to treat HIV. Adverse effects include rash, Stevens-Johnson syndrome, hepatitis, and increased transaminase levels. Hyperglycemia, peripheral neuropathy, and reduced bone density are not adverse effects of this medication.

72.) Cinoxacin (Cinobac), a urinary antiseptic, is prescribed for the client. The nurse reviews the client's medical record and should contact the health care provider (HCP) regarding which documented finding to verify the prescription? Refer to chart. 1. Renal insufficiency 2. Chest x-ray: normal 3. Blood glucose, 102 mg/dL 4. Folic acid (vitamin B6) 0.5 mg, orally daily

1. Renal insufficiency Rationale: Cinoxacin should be administered with caution in clients with renal impairment. The dosage should be reduced, and failure to do so could result in accumulation of cinoxacin to toxic levels. Therefore the nurse would verify the prescription if the client had a documented history of renal insufficiency. The laboratory and diagnostic test results are normal findings. Folic acid (vitamin B6) may be prescribed for a client with renal insufficiency to prevent anemia.

53.) Rifabutin (Mycobutin) is prescribed for a client with active Mycobacterium avium complex (MAC) disease and tuberculosis. The nurse monitors for which side effects of the medication? Select all that apply. 1. Signs of hepatitis 2. Flu-like syndrome 3. Low neutrophil count 4. Vitamin B6 deficiency 5. Ocular pain or blurred vision 6. Tingling and numbness of the fingers

1. Signs of hepatitis 2. Flu-like syndrome 3. Low neutrophil count 5. Ocular pain or blurred vision Rationale: Rifabutin (Mycobutin) may be prescribed for a client with active MAC disease and tuberculosis. It inhibits mycobacterial DNA-dependent RNA polymerase and suppresses protein synthesis. Side effects include rash, gastrointestinal disturbances, neutropenia (low neutrophil count), red-orange body secretions, uveitis (blurred vision and eye pain), myositis, arthralgia, hepatitis, chest pain with dyspnea, and flu-like syndrome. Vitamin B6 deficiency and numbness and tingling in the extremities are associated with the use of isoniazid (INH). Ethambutol (Myambutol) also causes peripheral neuritis.

213.) A client is admitted to the hospital with complaints of back spasms. The client states, "I have been taking two or three aspirin every 4 hours for the past week and it hasn't helped my back." Aspirin intoxication is suspected. Which of the following complaints would indicate aspirin intoxication? 1. Tinnitus 2. Constipation 3. Photosensitivity 4. Abdominal cramps

1. Tinnitus Rationale: Mild intoxication with acetylsalicylic acid (aspirin) is called salicylism and is commonly experienced when the daily dosage is higher than 4 g. Tinnitus (ringing in the ears) is the most frequently occurring effect noted with intoxication. Hyperventilation may occur because salicylate stimulates the respiratory center. Fever may result because salicylate interferes with the metabolic pathways involved with oxygen consumption and heat production. Options 2, 3, and 4 are incorrect.

8.) A client with severe acne is seen in the clinic and the health care provider (HCP) prescribes isotretinoin. The nurse reviews the client's medication record and would contact the (HCP) if the client is taking which medication? 1. Vitamin A 2. Digoxin (Lanoxin) 3. Furosemide (Lasix) 4. Phenytoin (Dilantin)

1. Vitamin A Rationale: Isotretinoin is a metabolite of vitamin A and can produce generalized intensification of isotretinoin toxicity. Because of the potential for increased toxicity, vitamin A supplements should be discontinued before isotretinoin therapy. Options 2, 3, and 4 are not contraindicated with the use of isotretinoin.

239.) Which of the following precautions will the nurse specifically take during the administration of ribavirin (Virazole) to a child with respiratory syncytial virus (RSV)? 1. Wearing goggles 2. Wearing a gown 3. Wearing a gown and a mask 4. Handwashing before administration

1. Wearing goggles Rationale: Some caregivers experience headaches, burning nasal passages and eyes, and crystallization of soft contact lenses as a result of administration of ribavirin. Specific to this medication is the use of goggles. A gown is not necessary. A mask may be worn. Handwashing is to be performed before and after any child contact.

24.) A nurse is reinforcing teaching for a client regarding how to mix regular insulin and NPH insulin in the same syringe. Which of the following actions, if performed by the client, indicates the need for further teaching? 1. Withdraws the NPH insulin first 2. Withdraws the regular insulin first 3. Injects air into NPH insulin vial first 4. Injects an amount of air equal to the desired dose of insulin into the vial

1. Withdraws the NPH insulin first Rationale: When preparing a mixture of regular insulin with another insulin preparation, the regular insulin is drawn into the syringe first. This sequence will avoid contaminating the vial of regular insulin with insulin of another type. Options 2, 3, and 4 identify the correct actions for preparing NPH and regular insulin.

234.) A hospitalized client is started on phenelzine sulfate (Nardil) for the treatment of depression. At lunchtime, a tray is delivered to the client. Which food item on the tray will the nurse remove? 1. Yogurt 2. Crackers 3. Tossed salad 4. Oatmeal cookies

1. Yogurt Rationale: Phenelzine sulfate is a monoamine oxidase inhibitor (MAOI). The client should avoid taking in foods that are high in tyramine. These foods could trigger a potentially fatal hypertensive crisis. Foods to avoid include yogurt, aged cheeses, smoked or processed meats, red wines, and fruits such as avocados, raisins, or figs.

A 65-year-old man is scheduled for a colon resection this morning. Last night he had polyethylene glycolelectrolyte solution and a soapsuds enema. This morning he passes a medium amount of soft, brown stool. The nurse should know that this 1. indicates that the bowel preparation is incomplete. 2. is evidence that the patient ate something after midnight. 3. is an expected finding before this type of surgery. 4. is the last stool that was left in the colon.

1. indicates that the bowel preparation is incomplete (1) correct-colon should not have remaining soft stool (2) anything eaten after midnight would not appear as stool by the next morning (3) not expected; need to clean GI tract for surgery (4) assumption, not substantiated

6.) The burn client is receiving treatments of topical mafenide acetate (Sulfamylon) to the site of injury. The nurse monitors the client, knowing that which of the following indicates that a systemic effect has occurred? 1.Hyperventilation 2.Elevated blood pressure 3.Local pain at the burn site 4.Local rash at the burn site

1.Hyperventilation Rationale: Mafenide acetate is a carbonic anhydrase inhibitor and can suppress renal excretion of acid, thereby causing acidosis. Clients receiving this treatment should be monitored for signs of an acid-base imbalance (hyperventilation). If this occurs, the medication should be discontinued for 1 to 2 days. Options 3 and 4 describe local rather than systemic effects. An elevated blood pressure may be expected from the pain that occurs with a burn injury.

194.) A nurse provides instructions to a client regarding the use of tretinoin (Retin-A). Which statement by the client indicates the need for further instructions? 1. "Optimal results will be seen after 6 weeks." 2. "I should apply a very thin layer to my skin." 3. "I should wash my hands thoroughly after applying the medication." 4. "I should cleanse my skin thoroughly before applying the medication."

2. "I should apply a very thin layer to my skin." Rationale: Tretinoin is applied liberally to the skin. The hands are washed thoroughly immediately after applying. Therapeutic results should be seen after 2 to 3 weeks but may not be optimal until after 6 weeks. The skin needs to be cleansed thoroughly before applying the medication.

78.) A client is taking phenytoin (Dilantin) for seizure control and a sample for a serum drug level is drawn. Which of the following indicates a therapeutic serum drug range? 1. 5 to 10 mcg/mL 2. 10 to 20 mcg/mL 3. 20 to 30 mcg/mL 4. 30 to 40 mcg/mL

2. 10 to 20 mcg/mL Rationale: The therapeutic serum drug level range for phenytoin (Dilantin) is 10 to 20 mcg/mL. ** A helpful hint may be to remember that the theophylline therapeutic range and the acetaminophen (Tylenol) therapeutic range are the same as the phenytoin (Dilantin) therapeutic range.**

64.) Nalidixic acid (NegGram) is prescribed for a client with a urinary tract infection. On review of the client's record, the nurse notes that the client is taking warfarin sodium (Coumadin) daily. Which prescription should the nurse anticipate for this client? 1. Discontinuation of warfarin sodium (Coumadin) 2. A decrease in the warfarin sodium (Coumadin) dosage 3. An increase in the warfarin sodium (Coumadin) dosage 4. A decrease in the usual dose of nalidixic acid (NegGram)

2. A decrease in the warfarin sodium (Coumadin) dosage Rationale: Nalidixic acid can intensify the effects of oral anticoagulants by displacing these agents from binding sites on plasma protein. When an oral anticoagulant is combined with nalidixic acid, a decrease in the anticoagulant dosage may be needed.

88.) Dantrolene sodium (Dantrium) is prescribed for a client experiencing flexor spasms, and the client asks the nurse about the action of the medication. The nurse responds, knowing that the therapeutic action of this medication is which of the following? 1. Depresses spinal reflexes 2. Acts directly on the skeletal muscle to relieve spasticity 3. Acts within the spinal cord to suppress hyperactive reflexes 4. Acts on the central nervous system (CNS) to suppress spasms

2. Acts directly on the skeletal muscle to relieve spasticity Rationale: Dantrium acts directly on skeletal muscle to relieve muscle spasticity. The primary action is the suppression of calcium release from the sarcoplasmic reticulum. This in turn decreases the ability of the skeletal muscle to contract. **Options 1, 3, and 4 are all comparable or alike in that they address CNS suppression and the depression of reflexes. Therefore, eliminate these options.**

74.) A client with myasthenia gravis is receiving pyridostigmine (Mestinon). The nurse monitors for signs and symptoms of cholinergic crisis caused by overdose of the medication. The nurse checks the medication supply to ensure that which medication is available for administration if a cholinergic crisis occurs? 1. Vitamin K 2. Atropine sulfate 3. Protamine sulfate 4. Acetylcysteine (Mucomyst)

2. Atropine sulfate Rationale: The antidote for cholinergic crisis is atropine sulfate. Vitamin K is the antidote for warfarin (Coumadin). Protamine sulfate is the antidote for heparin, and acetylcysteine (Mucomyst) is the antidote for acetaminophen (Tylenol).

9.) The nurse is applying a topical corticosteroid to a client with eczema. The nurse would monitor for the potential for increased systemic absorption of the medication if the medication were being applied to which of the following body areas? 1. Back 2. Axilla 3. Soles of the feet 4. Palms of the hands

2. Axilla Rationale: Topical corticosteroids can be absorbed into the systemic circulation. Absorption is higher from regions where the skin is especially permeable (scalp, axilla, face, eyelids, neck, perineum, genitalia), and lower from regions in which permeability is poor (back, palms, soles).

199.) A nurse is applying a topical glucocorticoid to a client with eczema. The nurse monitors for systemic absorption of the medication if the medication is being applied to which of the following body areas? 1. Back 2. Axilla 3. Soles of the feet 4. Palms of the hands

2. Axilla Rationale: Topical glucocorticoids can be absorbed into the systemic circulation. Absorption is higher from regions where the skin is especially permeable (scalp, axillae, face, eyelids, neck, perineum, genitalia), and lower from regions where penetrability is poor (back, palms, soles). **Eliminate options 3 and 4 because these body areas are similar in terms of skin characteristics**

224.) Neuroleptic malignant syndrome is suspected in a client who is taking chlorpromazine. Which medication would the nurse prepare in anticipation of being prescribed to treat this adverse effect related to the use of chlorpromazine? 1. Protamine sulfate 2. Bromocriptine (Parlodel) 3. Phytonadione (vitamin K) 4. Enalapril maleate (Vasotec)

2. Bromocriptine (Parlodel) Rationale: Bromocriptine is an antiparkinsonian prolactin inhibitor used in the treatment of neuroleptic malignant syndrome. Vitamin K is the antidote for warfarin (Coumadin) overdose. Protamine sulfate is the antidote for heparin overdose. Enalapril maleate is an antihypertensive used in the treatment of hypertension.

20.) The client with metastatic breast cancer is receiving tamoxifen. The nurse specifically monitors which laboratory value while the client is taking this medication? 1. Glucose level 2. Calcium level 3. Potassium level 4. Prothrombin time

2. Calcium level Rationale: Tamoxifen may increase calcium, cholesterol, and triglyceride levels. Before the initiation of therapy, a complete blood count, platelet count, and serum calcium levels should be assessed. These blood levels, along with cholesterol and triglyceride levels, should be monitored periodically during therapy. The nurse should assess for hypercalcemia while the client is taking this medication. Signs of hypercalcemia include increased urine volume, excessive thirst, nausea, vomiting, constipation, hypotonicity of muscles, and deep bone and flank pain.

170.) Atenolol hydrochloride (Tenormin) is prescribed for a hospitalized client. The nurse should perform which of the following as a priority action before administering the medication? 1. Listen to the client's lung sounds. 2. Check the client's blood pressure. 3. Check the recent electrolyte levels. 4. Assess the client for muscle weakness.

2. Check the client's blood pressure. Rationale: Atenolol hydrochloride is a beta-blocker used to treat hypertension. Therefore the priority nursing action before administration of the medication is to check the client's blood pressure. The nurse also checks the client's apical heart rate. If the systolic blood pressure is below 90 mm Hg or the apical pulse is 60 beats per minute or lower, the medication is withheld and the registered nurse and/or health care provider is notified. The nurse would check baseline renal and liver function tests. The medication may cause weakness, and the nurse would assist the client with activities if weakness occurs. **Beta-blockers have "-lol" at the end of the medication name**

146.) A client has begun therapy with theophylline (Theo-24). The nurse tells the client to limit the intake of which of the following while taking this medication? 1. Oranges and pineapple 2. Coffee, cola, and chocolate 3. Oysters, lobster, and shrimp 4. Cottage cheese, cream cheese, and dairy creamers

2. Coffee, cola, and chocolate Rationale: Theophylline is a xanthine bronchodilator. The nurse teaches the client to limit the intake of xanthine-containing foods while taking this medication. These include coffee, cola, and chocolate.

108.) A nurse is performing a follow-up teaching session with a client discharged 1 month ago who is taking fluoxetine (Prozac). What information would be important for the nurse to gather regarding the adverse effects related to the medication? 1. Cardiovascular symptoms 2. Gastrointestinal dysfunctions 3. Problems with mouth dryness 4. Problems with excessive sweating

2. Gastrointestinal dysfunctions Rationale: The most common adverse effects related to fluoxetine include central nervous system (CNS) and gastrointestinal (GI) system dysfunction. This medication affects the GI system by causing nausea and vomiting, cramping, and diarrhea. Options 1, 3, and 4 are not adverse effects of this medication.

101.) Ketoconazole is prescribed for a client with a diagnosis of candidiasis. Select the interventions that the nurse includes when administering this medication. Select all that apply. 1. Restrict fluid intake. 2. Instruct the client to avoid alcohol. 3. Monitor hepatic and liver function studies. 4. Administer the medication with an antacid. 5. Instruct the client to avoid exposure to the sun. 6. Administer the medication on an empty stomach.

2. Instruct the client to avoid alcohol. 3. Monitor hepatic and liver function studies. 5. Instruct the client to avoid exposure to the sun. Rationale: Ketoconazole is an antifungal medication. It is administered with food (not on an empty stomach) and antacids are avoided for 2 hours after taking the medication to ensure absorption. The medication is hepatotoxic and the nurse monitors liver function studies. The client is instructed to avoid exposure to the sun because the medication increases photosensitivity. The client is also instructed to avoid alcohol. There is no reason for the client to restrict fluid intake. In fact, this could be harmful to the client.

65.) A nurse is reinforcing discharge instructions to a client receiving sulfisoxazole. Which of the following should be included in the list of instructions? 1. Restrict fluid intake. 2. Maintain a high fluid intake. 3. If the urine turns dark brown, call the health care provider (HCP) immediately. 4. Decrease the dosage when symptoms are improving to prevent an allergic response.

2. Maintain a high fluid intake. Rationale: Each dose of sulfisoxazole should be administered with a full glass of water, and the client should maintain a high fluid intake. The medication is more soluble in alkaline urine. The client should not be instructed to taper or discontinue the dose. Some forms of sulfisoxazole cause urine to turn dark brown or red. This does not indicate the need to notify the HCP.

204.) A client receives a dose of edrophonium (Enlon). The client shows improvement in muscle strength for a period of time following the injection. The nurse interprets that this finding is compatible with: 1. Multiple sclerosis 2. Myasthenia gravis 3. Muscular dystrophy 4. Amyotrophic lateral sclerosis

2. Myasthenia gravis Rationale: Myasthenia gravis can often be diagnosed based on clinical signs and symptoms. The diagnosis can be confirmed by injecting the client with a dose of edrophonium . This medication inhibits the breakdown of an enzyme in the neuromuscular junction, so more acetylcholine binds to receptors. If the muscle is strengthened for 3 to 5 minutes after this injection, it confirms a diagnosis of myasthenia gravis. Another medication, neostigmine (Prostigmin), also may be used because its effect lasts for 1 to 2 hours, providing a better analysis. For either medication, atropine sulfate should be available as the antidote.

227.) When teaching a client who is being started on imipramine hydrochloride (Tofranil), the nurse would inform the client that the desired effects of the medication may: 1. Start during the first week of administration 2. Not occur for 2 to 3 weeks of administration 3. Start during the second week of administration 4. Not occur until after a month of administration

2. Not occur for 2 to 3 weeks of administration Rationale: The therapeutic effects of administration of imipramine hydrochloride may not occur for 2 to 3 weeks after the antidepressant therapy has been initiated. Therefore options 1, 3, and 4 are incorrect.

169.) Insulin glargine (Lantus) is prescribed for a client with diabetes mellitus. The nurse tells the client that it is best to take the insulin: 1. 1 hour after each meal 2. Once daily, at the same time each day 3. 15 minutes before breakfast, lunch, and dinner 4. Before each meal, on the basis of the blood glucose level

2. Once daily, at the same time each day Rationale: Insulin glargine is a long-acting recombinant DNA human insulin used to treat type 1 and type 2 diabetes mellitus. It has a 24-hour duration of action and is administered once a day, at the same time each day.

47.) A client has been taking isoniazid (INH) for 2 months. The client complains to a nurse about numbness, paresthesias, and tingling in the extremities. The nurse interprets that the client is experiencing: 1. Hypercalcemia 2. Peripheral neuritis 3. Small blood vessel spasm 4. Impaired peripheral circulation

2. Peripheral neuritis Rationale: A common side effect of the TB drug INH is peripheral neuritis. This is manifested by numbness, tingling, and paresthesias in the extremities. This side effect can be minimized by pyridoxine (vitamin B6) intake. Options 1, 3, and 4 are incorrect.

228.) A client receiving an anxiolytic medication complains that he feels very "faint" when he tries to get out of bed in the morning. The nurse recognizes this complaint as a symptom of: 1. Cardiac dysrhythmias 2. Postural hypotension 3. Psychosomatic symptoms 4. Respiratory insufficiency

2. Postural hypotension Rationale: Anxiolytic medications can cause postural hypotension. The client needs to be taught to rise to a sitting position and get out of bed slowly because of this adverse effect related to the medication. Options 1, 3, and 4 are unrelated to the use of this medication.

186.) A nurse prepares to administer sodium polystyrene sulfonate (Kayexalate) to a client. Before administering the medication, the nurse reviews the action of the medication and understands that it: 1. Releases bicarbonate in exchange for primarily sodium ions 2. Releases sodium ions in exchange for primarily potassium ions 3. Releases potassium ions in exchange for primarily sodium ions 4. Releases sodium ions in exchange for primarily bicarbonate ions

2. Releases sodium ions in exchange for primarily potassium ions Rationale: Sodium polystyrene sulfonate is a cation exchange resin used in the treatment of hyperkalemia. The resin either passes through the intestine or is retained in the colon. It releases sodium ions in exchange for primarily potassium ions. The therapeutic effect occurs 2 to 12 hours after oral administration and longer after rectal administration.

40.) The client who chronically uses nonsteroidal anti-inflammatory drugs has been taking misoprostol (Cytotec). The nurse determines that the medication is having the intended therapeutic effect if which of the following is noted? 1. Resolved diarrhea 2. Relief of epigastric pain 3. Decreased platelet count 4. Decreased white blood cell count

2. Relief of epigastric pain Rationale: The client who chronically uses nonsteroidal anti-inflammatory drugs (NSAIDs) is prone to gastric mucosal injury. Misoprostol is a gastric protectant and is given specifically to prevent this occurrence. Diarrhea can be a side effect of the medication, but is not an intended effect. Options 3 and 4 are incorrect.

23.) A client who has been newly diagnosed with diabetes mellitus has been stabilized with daily insulin injections. Which information should the nurse teach when carrying out plans for discharge? 1. Keep insulin vials refrigerated at all times. 2. Rotate the insulin injection sites systematically. 3. Increase the amount of insulin before unusual exercise. 4. Monitor the urine acetone level to determine the insulin dosage.

2. Rotate the insulin injection sites systematically. Rationale: Insulin dosages should not be adjusted or increased before unusual exercise. If acetone is found in the urine, it may possibly indicate the need for additional insulin. To minimize the discomfort associated with insulin injections, the insulin should be administered at room temperature. Injection sites should be systematically rotated from one area to another. The client should be instructed to give injections in one area, about 1 inch apart, until the whole area has been used and then to change to another site. This prevents dramatic changes in daily insulin absorption.

216.) A nurse is caring for a client with gout who is taking Colcrys (colchicine). The client has been instructed to restrict the diet to low-purine foods. Which of the following foods should the nurse instruct the client to avoid while taking this medication? 1. Spinach 2. Scallops 3. Potatoes 4. Ice cream

2. Scallops Rationale: Colchicine is a medication used for clients with gout to inhibit the reabsorption of uric acid by the kidney and promote excretion of uric acid in the urine. Uric acid is produced when purine is catabolized. Clients are instructed to modify their diet and limit excessive purine intake. High-purine foods to avoid or limit include organ meats, roe, sardines, scallops, anchovies, broth, mincemeat, herring, shrimp, mackerel, gravy, and yeast.

172.) A nurse provides dietary instructions to a client who will be taking warfarin sodium (Coumadin). The nurse tells the client to avoid which food item? 1. Grapes 2. Spinach 3. Watermelon 4. Cottage cheese

2. Spinach Rationale: Warfarin sodium is an anticoagulant. Anticoagulant medications act by antagonizing the action of vitamin K, which is needed for clotting. When a client is taking an anticoagulant, foods high in vitamin K often are omitted from the diet. Vitamin K-rich foods include green, leafy vegetables, fish, liver, coffee, and tea.

188.) The nurse should anticipate that the most likely medication to be prescribed prophylactically for a child with spina bifida (myelomeningocele) who has a neurogenic bladder would be: 1. Prednisone 2. Sulfisoxazole 3. Furosemide (Lasix) 4. Intravenous immune globulin (IVIG)

2. Sulfisoxazole Rationale: A neurogenic bladder prevents the bladder from completely emptying because of the decrease in muscle tone. The most likely medication to be prescribed to prevent urinary tract infection would be an antibiotic. A common prescribed medication is sulfisoxazole. Prednisone relieves allergic reactions and inflammation rather than preventing infection. Furosemide promotes diuresis and decreases edema caused by congestive heart failure. IVIG assists with antibody production in immunocompromised clients.

147.) A client with a prescription to take theophylline (Theo-24) daily has been given medication instructions by the nurse. The nurse determines that the client needs further information about the medication if the client states that he or she will: 1. Drink at least 2 L of fluid per day. 2. Take the daily dose at bedtime. 3. Avoid changing brands of the medication without health care provider (HCP) approval. 4. Avoid over-the-counter (OTC) cough and cold medications unless approved by the HCP.

2. Take the daily dose at bedtime. Rationale: The client taking a single daily dose of theophylline, a xanthine bronchodilator, should take the medication early in the morning. This enables the client to have maximal benefit from the medication during daytime activities. In addition, this medication causes insomnia. The client should take in at least 2 L of fluid per day to decrease viscosity of secretions. The client should check with the physician before changing brands of the medication. The client also checks with the HCP before taking OTC cough, cold, or other respiratory preparations because they could cause interactive effects, increasing the side effects of theophylline and causing dysrhythmias.

45.) A client has a prescription to take guaifenesin (Humibid) every 4 hours, as needed. The nurse determines that the client understands the most effective use of this medication if the client states that he or she will: 1. Watch for irritability as a side effect. 2. Take the tablet with a full glass of water. 3. Take an extra dose if the cough is accompanied by fever. 4. Crush the sustained-release tablet if immediate relief is needed.

2. Take the tablet with a full glass of water. Rationale: Guaifenesin is an expectorant. It should be taken with a full glass of water to decrease viscosity of secretions. Sustained-release preparations should not be broken open, crushed, or chewed. The medication may occasionally cause dizziness, headache, or drowsiness as side effects. The client should contact the health care provider if the cough lasts longer than 1 week or is accompanied by fever, rash, sore throat, or persistent headache.

214.) A health care provider initiates carbidopa/levodopa (Sinemet) therapy for the client with Parkinson's disease. A few days after the client starts the medication, the client complains of nausea and vomiting. The nurse tells the client that: 1. Taking an antiemetic is the best measure to prevent the nausea. 2. Taking the medication with food will help to prevent the nausea. 3. This is an expected side effect of the medication and will decrease over time. 4. The nausea and vomiting will decrease when the dose of levodopa is stabilized.

2. Taking the medication with food will help to prevent the nausea. Rationale: If carbidopa/levodopa is causing nausea and vomiting, the nurse would tell the client that taking the medication with food will prevent the nausea. Additionally, the client should be instructed not to take the medication with a high-protein meal because the high-protein will affect absorption. Antiemetics from the phenothiazine class should not be used because they block the therapeutic action of dopamine. **eliminate options 3 and 4 because they are comparable or alike**

57.) A nurse is monitoring a client who is taking propranolol (Inderal LA). Which data collection finding would indicate a potential serious complication associated with propranolol? 1. The development of complaints of insomnia 2. The development of audible expiratory wheezes 3. A baseline blood pressure of 150/80 mm Hg followed by a blood pressure of 138/72 mm Hg after two doses of the medication 4. A baseline resting heart rate of 88 beats/min followed by a resting heart rate of 72 beats/min after two doses of the medication

2. The development of audible expiratory wheezes Rationale: Audible expiratory wheezes may indicate a serious adverse reaction, bronchospasm. β-Blockers may induce this reaction, particularly in clients with chronic obstructive pulmonary disease or asthma. Normal decreases in blood pressure and heart rate are expected. Insomnia is a frequent mild side effect and should be monitored.

61.) A home health care nurse is visiting a client with elevated triglyceride levels and a serum cholesterol level of 398 mg/dL. The client is taking cholestyramine (Questran). Which of the following statements, if made by the client, indicates the need for further education? 1. "Constipation and bloating might be a problem." 2. "I'll continue to watch my diet and reduce my fats." 3. "Walking a mile each day will help the whole process." 4. "I'll continue my nicotinic acid from the health food store."

4. "I'll continue my nicotinic acid from the health food store." Rationale: Nicotinic acid, even an over-the-counter form, should be avoided because it may lead to liver abnormalities. All lipid-lowering medications also can cause liver abnormalities, so a combination of nicotinic acid and cholestyramine resin is to be avoided. Constipation and bloating are the two most common side effects. Walking and the reduction of fats in the diet are therapeutic measures to reduce cholesterol and triglyceride levels.

28.) The health care provider (HCP) prescribes exenatide (Byetta) for a client with type 1 diabetes mellitus who takes insulin. The nurse knows that which of the following is the appropriate intervention? 1. The medication is administered within 60 minutes before the morning and evening meal. 2. The medication is withheld and the HCP is called to question the prescription for the client. 3. The client is monitored for gastrointestinal side effects after administration of the medication. 4. The insulin is withdrawn from the Penlet into an insulin syringe to prepare for administration.

2. The medication is withheld and the HCP is called to question the prescription for the client. Rationale: Exenatide (Byetta) is an incretin mimetic used for type 2 diabetes mellitus only. It is not recommended for clients taking insulin. Hence, the nurse should hold the medication and question the HCP regarding this prescription. Although options 1 and 3 are correct statements about the medication, in this situation the medication should not be administered. The medication is packaged in prefilled pens ready for injection without the need for drawing it up into another syringe.

83.) The client has been on treatment for rheumatoid arthritis for 3 weeks. During the administration of etanercept (Enbrel), it is most important for the nurse to check: 1. The injection site for itching and edema 2. The white blood cell counts and platelet counts 3. Whether the client is experiencing fatigue and joint pain 4. A metallic taste in the mouth, with a loss of appetite

2. The white blood cell counts and platelet counts Rationale: Infection and pancytopenia are side effects of etanercept (Enbrel). Laboratory studies are performed before and during drug treatment. The appearance of abnormal white blood cell counts and abnormal platelet counts can alert the nurse to a potentially life-threatening infection. Injection site itching is a common occurrence following administration. A metallic taste with loss of appetite are not common signs of side effects of this medication.

126.) A nurse is caring for a client who has been prescribed furosemide (Lasix) and is monitoring for adverse effects associated with this medication. Which of the following should the nurse recognize as a potential adverse effect Select all that apply. 1. Nausea 2. Tinnitus 3. Hypotension 4. Hypokalemia 5. Photosensitivity 6. Increased urinary frequency

2. Tinnitus 3. Hypotension 4. Hypokalemia Rationale: Furosemide is a loop diuretic; therefore, an expected effect is increased urinary frequency. Nausea is a frequent side effect, not an adverse effect. Photosensitivity is an occasional side effect. Adverse effects include tinnitus (ototoxicity), hypotension, and hypokalemia and occur as a result of sudden volume depletion.

51.) Cycloserine (Seromycin) is added to the medication regimen for a client with tuberculosis. Which of the following would the nurse include in the client-teaching plan regarding this medication? 1. To take the medication before meals 2. To return to the clinic weekly for serum drug-level testing 3. It is not necessary to call the health care provider (HCP) if a skin rash occurs. 4. It is not necessary to restrict alcohol intake with this medication.

2. To return to the clinic weekly for serum drug-level testing Rationale: Cycloserine (Seromycin) is an antitubercular medication that requires weekly serum drug level determinations to monitor for the potential of neurotoxicity. Serum drug levels lower than 30 mcg/mL reduce the incidence of neurotoxicity. The medication must be taken after meals to prevent gastrointestinal irritation. The client must be instructed to notify the HCP if a skin rash or signs of central nervous system toxicity are noted. Alcohol must be avoided because it increases the risk of seizure activity.

82.) A client is receiving meperidine hydrochloride (Demerol) for pain. Which of the following are side effects of this medication. Select all that apply. 1. Diarrhea 2. Tremors 3. Drowsiness 4. Hypotension 5. Urinary frequency 6. Increased respiratory rate

2. Tremors 3. Drowsiness 4. Hypotension Rationale: Meperidine hydrochloride is an opioid analgesic. Side effects include respiratory depression, drowsiness, hypotension, constipation, urinary retention, nausea, vomiting, and tremors.

135.) A nurse reinforces medication instructions to a client who is taking levothyroxine (Synthroid). The nurse instructs the client to notify the health care provider (HCP) if which of the following occurs? 1. Fatigue 2. Tremors 3. Cold intolerance 4. Excessively dry skin

2. Tremors Rationale: Excessive doses of levothyroxine (Synthroid) can produce signs and symptoms of hyperthyroidism. These include tachycardia, chest pain, tremors, nervousness, insomnia, hyperthermia, heat intolerance, and sweating. The client should be instructed to notify the HCP if these occur. Options 1, 3, and 4 are signs of hypothyroidism.

7.) Isotretinoin is prescribed for a client with severe acne. Before the administration of this medication, the nurse anticipates that which laboratory test will be prescribed? 1. Platelet count 2. Triglyceride level 3. Complete blood count 4. White blood cell count

2. Triglyceride level Rationale: Isotretinoin can elevate triglyceride levels. Blood triglyceride levels should be measured before treatment and periodically thereafter until the effect on the triglycerides has been evaluated. Options 1, 3, and 4 do not need to be monitored specifically during this treatment.

14.) The client with acute myelocytic leukemia is being treated with busulfan (Myleran). Which laboratory value would the nurse specifically monitor during treatment with this medication? 1. Clotting time 2. Uric acid level 3. Potassium level 4. Blood glucose level

2. Uric acid level Rationale: Busulfan (Myleran) can cause an increase in the uric acid level. Hyperuricemia can produce uric acid nephropathy, renal stones, and acute renal failure. Options 1, 3, and 4 are not specifically related to this medication.

68.) Bethanechol chloride (Urecholine) is prescribed for a client with urinary retention. Which disorder would be a contraindication to the administration of this medication? 1. Gastric atony 2. Urinary strictures 3. Neurogenic atony 4. Gastroesophageal reflux

2. Urinary strictures Rationale: Bethanechol chloride (Urecholine) can be harmful to clients with urinary tract obstruction or weakness of the bladder wall. The medication has the ability to contract the bladder and thereby increase pressure within the urinary tract. Elevation of pressure within the urinary tract could rupture the bladder in clients with these conditions.

238.) Ribavirin (Virazole) is prescribed for the hospitalized child with respiratory syncytial virus (RSV). The nurse prepares to administer this medication via which of the following routes? 1. Orally 2. Via face mask 3. Intravenously 4. Intramuscularly

2. Via face mask Rationale: Ribavirin is an antiviral respiratory medication used mainly in hospitalized children with severe RSV and in high-risk children. Administration is via hood, face mask, or oxygen tent. The medication is most effective if administered within the first 3 days of the infection.

one hour after receiving PYRIDOSTIGMINE bromide, a client reports difficulty swallowing and excessive respiratory secretions. The nurse notifies the physician and prepares to administer which medication? 1. additional pyridostigmine bromide 2. atropine 3. edrophonium 4. acyclovir

2. atropine (anticholinergic) pyridostigmine (cholinergic) tx myasthenia gravis pt is experiencing a cholinergic crisis!!

55.) A client who is receiving digoxin (Lanoxin) daily has a serum potassium level of 3.0 mEq/L and is complaining of anorexia. A health care provider prescribes a digoxin level to rule out digoxin toxicity. A nurse checks the results, knowing that which of the following is the therapeutic serum level (range) for digoxin? 1. 3 to 5 ng/mL 2. 0.5 to 2 ng/mL 3. 1.2 to 2.8 ng/mL 4. 3.5 to 5.5 ng/mL

2.) 0.5 to 2 ng/mL Rationale: Therapeutic levels for digoxin range from 0.5 to 2 ng/mL. Therefore, options 1, 3, and 4 are incorrect.

You are administering VANCOMYCIN 500 mg IV to a client with a MRSA wound infection when you notice that the client's neck and face are becoming flushed. Which action should you take next? 1.Discontinue the vancomycin infusion. 2.Slow the rate of the vancomycin infusion. 3.Obtain an order for an antihistamine. 4.Check the client's temperature.

2.Slow the rate of the vancomycin infusion "RED MAN SYNDROME" occurs when vancomycin is infused too quickly. Because the client needs the medication to treat the infection, the vancomycin should not be discontinued. Antihistamines may help decrease the flushing, but vancomycin should be administered over at least 60 minutes. Although the client's temperature will be monitored, a temperature elevation is not the most likely cause of the client's flushing.

Interactions w/ 2nd gen antidepressants

2nd gen antidepressants are highly bound to albumin. When given with other drugs that are highly albumin-bound (phenytoin, warfarin), they compete for binding sites. This results in more free, unbound drug and therefore a more pronounced drug effect.

128.) A nurse is providing instructions to an adolescent who has a history of seizures and is taking an anticonvulsant medication. Which of the following statements indicates that the client understands the instructions? 1. "I will never be able to drive a car." 2. "My anticonvulsant medication will clear up my skin." 3. "I can't drink alcohol while I am taking my medication." 4. "If I forget my morning medication, I can take two pills at bedtime."

3. "I can't drink alcohol while I am taking my medication." Rationale: Alcohol will lower the seizure threshold and should be avoided. Adolescents can obtain a driver's license in most states when they have been seizure free for 1 year. Anticonvulsants cause acne and oily skin; therefore a dermatologist may need to be consulted. If an anticonvulsant medication is missed, the health care provider should be notified.

106.) Fluoxetine (Prozac) is prescribed for the client. The nurse reinforces instructions to the client regarding the administration of the medication. Which statement by the client indicates an understanding about administration of the medication? 1. "I should take the medication with my evening meal." 2. "I should take the medication at noon with an antacid." 3. "I should take the medication in the morning when I first arise." 4. "I should take the medication right before bedtime with a snack."

3. "I should take the medication in the morning when I first arise." Rationale: Fluoxetine hydrochloride is administered in the early morning without consideration to meals. **Eliminate options 1, 2, and 4 because they are comparable or alike and indicate taking the medication with an antacid or food.**

197.) Collagenase (Santyl) is prescribed for a client with a severe burn to the hand. The nurse provides instructions to the client regarding the use of the medication. Which statement by the client indicates an accurate understanding of the use of this medication? 1. "I will apply the ointment once a day and leave it open to the air." 2. "I will apply the ointment twice a day and leave it open to the air." 3. "I will apply the ointment once a day and cover it with a sterile dressing." 4. "I will apply the ointment at bedtime and in the morning and cover it with a sterile dressing."

3. "I will apply the ointment once a day and cover it with a sterile dressing." Rationale: Collagenase is used to promote debridement of dermal lesions and severe burns. It is usually applied once daily and covered with a sterile dressing.

164.) A client receives a prescription for methocarbamol (Robaxin), and the nurse reinforces instructions to the client regarding the medication. Which client statement would indicate a need for further instructions? 1. "My urine may turn brown or green." 2. "This medication is prescribed to help relieve my muscle spasms." 3. "If my vision becomes blurred, I don't need to be concerned about it." 4. "I need to call my doctor if I experience nasal congestion from this medication."

3. "If my vision becomes blurred, I don't need to be concerned about it." Rationale: The client needs to be told that the urine may turn brown, black, or green. Other adverse effects include blurred vision, nasal congestion, urticaria, and rash. The client needs to be instructed that, if these adverse effects occur, the health care provider needs to be notified. The medication is used to relieve muscle spasms.

11.) The health care provider has prescribed silver sulfadiazine (Silvadene) for the client with a partial-thickness burn, which has cultured positive for gram-negative bacteria. The nurse is reinforcing information to the client about the medication. Which statement made by the client indicates a lack of understanding about the treatments? 1. "The medication is an antibacterial." 2. "The medication will help heal the burn." 3. "The medication will permanently stain my skin." 4. "The medication should be applied directly to the wound."

3. "The medication will permanently stain my skin." Rationale: Silver sulfadiazine (Silvadene) is an antibacterial that has a broad spectrum of activity against gram-negative bacteria, gram-positive bacteria, and yeast. It is applied directly to the wound to assist in healing. It does not stain the skin.

226.) A client receiving lithium carbonate (Lithobid) complains of loose, watery stools and difficulty walking. The nurse would expect the serum lithium level to be which of the following? 1. 0.7 mEq/L 2. 1.0 mEq/L 3. 1.2 mEq/L 4. 1.7 mEq/L

4. 1.7 mEq/L Rationale: The therapeutic serum level of lithium ranges from 0.6 to 1.2 mEq/L. Serum lithium levels above the therapeutic level will produce signs of toxicity.

75.) A client with myasthenia gravis becomes increasingly weak. The health care provider prepares to identify whether the client is reacting to an overdose of the medication (cholinergic crisis) or increasing severity of the disease (myasthenic crisis). An injection of edrophonium (Enlon) is administered. Which of the following indicates that the client is in cholinergic crisis? 1. No change in the condition 2. Complaints of muscle spasms 3. An improvement of the weakness 4. A temporary worsening of the condition

4. A temporary worsening of the condition Rationale: An edrophonium (Enlon) injection, a cholinergic drug, makes the client in cholinergic crisis temporarily worse. This is known as a negative test. An improvement of weakness would occur if the client were experiencing myasthenia gravis. Options 1 and 2 would not occur in either crisis.

181.) A client is taking ticlopidine hydrochloride (Ticlid). The nurse tells the client to avoid which of the following while taking this medication? 1. Vitamin C 2. Vitamin D 3. Acetaminophen (Tylenol) 4. Acetylsalicylic acid (aspirin)

4. Acetylsalicylic acid (aspirin) Rationale: Ticlopidine hydrochloride is a platelet aggregation inhibitor. It is used to decrease the risk of thrombotic strokes in clients with precursor symptoms. Because it is an antiplatelet agent, other medications that precipitate or aggravate bleeding should be avoided during its use. Therefore, aspirin or any aspirin-containing product should be avoided.

To avoid a falsely elevated serum digoxin level, the nurse should wait how long after administering oral digoxin (lanoxin) to draw a blood sample? 1. At least 1 hour 2. At least 4 hours 3. At least 6 hours 4. At least 8 hours

4. At least 8 hours to avoid a falsely elevated serum digoxin level, the nurse should wait at least 8 hours after administering oral digoxin and at least 6 hours after administering IV digoxin to draw a blood sample.

178.) Methylergonovine (Methergine) is prescribed for a client with postpartum hemorrhage caused by uterine atony. Before administering the medication, the nurse checks which of the following as the important client parameter? 1. Temperature 2. Lochial flow 3. Urine output 4. Blood pressure

4. Blood pressure Rationale: Methylergonovine is an ergot alkaloid used for postpartum hemorrhage. It stimulates contraction of the uterus and causes arterial vasoconstriction. Ergot alkaloids are avoided in clients with significant cardiovascular disease, peripheral disease, hypertension, eclampsia, or preeclampsia. These conditions are worsened by the vasoconstrictive effects of the ergot alkaloids. The nurse would check the client's blood pressure before administering the medication and would follow agency protocols regarding withholding of the medication. Options 1, 2, and 3 are items that are checked in the postpartum period, but they are unrelated to the use of this medication.

Which client is at greatest risk for respiratory depression while receiving opioids for analgesia? 1. Elderly client with chronic pain who has a hip fracture 2. Client with a heroin addiction and back pain 3. Young female client with advanced multiple myeloma 4. Child with an arm fracture and cystic fibrosis

4. Child with an arm fracture and cystic At greatest risk are elderly clients, opiate-naive clients, and those with underlying pulmonary disease. The child has two of the three risk factors.

19.) Tamoxifen is prescribed for the client with metastatic breast carcinoma. The nurse understands that the primary action of this medication is to: 1. Increase DNA and RNA synthesis. 2. Promote the biosynthesis of nucleic acids. 3. Increase estrogen concentration and estrogen response. 4. Compete with estradiol for binding to estrogen in tissues containing high concentrations of receptors.

4. Compete with estradiol for binding to estrogen in tissues containing high concentrations of receptors. Rationale: Tamoxifen is an antineoplastic medication that competes with estradiol for binding to estrogen in tissues containing high concentrations of receptors. Tamoxifen is used to treat metastatic breast carcinoma in women and men. Tamoxifen is also effective in delaying the recurrence of cancer following mastectomy. Tamoxifen reduces DNA synthesis and estrogen response.

217.) A health care provider prescribes auranofin (Ridaura) for a client with rheumatoid arthritis. Which of the following would indicate to the nurse that the client is experiencing toxicity related to the medication? 1. Joint pain 2. Constipation 3. Ringing in the ears 4. Complaints of a metallic taste in the mouth

4. Complaints of a metallic taste in the mouth Rationale: Ridaura is the one gold preparation that is given orally rather than by injection. Gastrointestinal reactions including diarrhea, abdominal pain, nausea, and loss of appetite are common early in therapy, but these usually subside in the first 3 months of therapy. Early symptoms of toxicity include a rash, purple blotches, pruritus, mouth lesions, and a metallic taste in the mouth.

157.) A client receiving nitrofurantoin (Macrodantin) calls the health care provider's office complaining of side effects related to the medication. Which side effect indicates the need to stop treatment with this medication? 1. Nausea 2. Diarrhea 3. Anorexia 4. Cough and chest pain

4. Cough and chest pain Rationale: Gastrointestinal (GI) effects are the most frequent adverse reactions to this medication and can be minimized by administering the medication with milk or meals. Pulmonary reactions, manifested as dyspnea, chest pain, chills, fever, cough, and the presence of alveolar infiltrates on the x-ray, would indicate the need to stop the treatment. These symptoms resolve in 2 to 4 days following discontinuation of this medication. **Eliminate options 1, 2, and 3 because they are similar GI-related side effects. Also, use the ABCs— airway, breathing, and circulation**

132.) The client with non-Hodgkin's lymphoma is receiving daunorubicin (DaunoXome). Which of the following would indicate to the nurse that the client is experiencing a toxic effect related to the medication? 1. Fever 2. Diarrhea 3. Complaints of nausea and vomiting 4. Crackles on auscultation of the lungs

4. Crackles on auscultation of the lungs Rationale: Cardiotoxicity noted by abnormal electrocardiographic findings or cardiomyopathy manifested as congestive heart failure is a toxic effect of daunorubicin. Bone marrow depression is also a toxic effect. Nausea and vomiting are frequent side effects associated with the medication that begins a few hours after administration and lasts 24 to 48 hours. Fever is a frequent side effect, and diarrhea can occur occasionally. The other options, however, are not toxic effects. **keep in mind that the question is asking about a toxic effect and think: ABCs—airway, breathing, and circulation**

A client with hepatitis C develops liver failure and GI hemorrhage. The blood products that would most likely bring about hemostasis in the client care: 1. Whole blood and albumin. 2. Platelets and packed red blood cells. 3. Fresh frozen plasma and whole blood. 4. Cryoprecipitate and fresh frozen plasma.

4. Cryoprecipitate and fresh frozen plasma. the liver is vital in the synthesis of clotting factors, so when its diseased or dysfunctional, as in hepatitis C, bleeding occurs. Treatment consists of administering blood products that aid clotting. These include fresh frozen plasma containing fibrinogen and cryoprecipitate, which have most of the CLOTTING FACTORS. Although administering whole blood, albumin, packed cells will contribute to hemostasis, those products aren't specifically used to treat hemostasis. Platelets are helpful, but the best answer is cryoprecipitate and fresh frozen plasma.

117.) A nurse has given the client taking ethambutol (Myambutol) information about the medication. The nurse determines that the client understands the instructions if the client immediately reports: 1. Impaired sense of hearing 2. Distressing gastrointestinal side effects 3. Orange-red discoloration of body secretions 4. Difficulty discriminating the color red from green

4. Difficulty discriminating the color red from green Rationale: Ethambutol causes optic neuritis, which decreases visual acuity and the ability to discriminate between the colors red and green. This poses a potential safety hazard when driving a motor vehicle. The client is taught to report this symptom immediately. The client is also taught to take the medication with food if gastrointestinal upset occurs. Impaired hearing results from antitubercular therapy with streptomycin. Orange-red discoloration of secretions occurs with rifampin (Rifadin).

190.) A child is hospitalized with a diagnosis of lead poisoning. The nurse assisting in caring for the child would prepare to assist in administering which of the following medications? 1. Activated charcoal 2. Sodium bicarbonate 3. Syrup of ipecac syrup 4. Dimercaprol (BAL in Oil)

4. Dimercaprol (BAL in Oil) Rationale: Dimercaprol is a chelating agent that is administered to remove lead from the circulating blood and from some tissues and organs for excretion in the urine. Sodium bicarbonate may be used in salicylate poisoning. Syrup of ipecac is used in the hospital setting in poisonings to induce vomiting. Activated charcoal is used to decrease absorption in certain poisoning situations. Note that dimercaprol is prepared with peanut oil, and hence should be avoided by clients with known or suspected peanut allergy.

203.) A nurse is preparing to give the postcraniotomy client medication for incisional pain. The family asks the nurse why the client is receiving codeine sulfate and not "something stronger." In formulating a response, the nurse incorporates the understanding that codeine: 1. Is one of the strongest opioid analgesics available 2. Cannot lead to physical or psychological dependence 3. Does not cause gastrointestinal upset or constipation as do other opioids 4. Does not alter respirations or mask neurological signs as do other opioids

4. Does not alter respirations or mask neurological signs as do other opioids Rationale: Codeine sulfate is the opioid analgesic often used for clients after craniotomy. It is frequently combined with a nonopioid analgesic such as acetaminophen for added effect. It does not alter the respiratory rate or mask neurological signs as do other opioids. Side effects of codeine include gastrointestinal upset and constipation. The medication can lead to physical and psychological dependence with chronic use. It is not the strongest opioid analgesic available.

159.) A nurse is caring for a client receiving morphine sulfate subcutaneously for pain. Because morphine sulfate has been prescribed for this client, which nursing action would be included in the plan of care? 1. Encourage fluid intake. 2. Monitor the client's temperature. 3. Maintain the client in a supine position. 4. Encourage the client to cough and deep breathe.

4. Encourage the client to cough and deep breathe. Rationale: Morphine sulfate suppresses the cough reflex. Clients need to be encouraged to cough and deep breathe to prevent pneumonia. **ABCs—airway, breathing, and circulation**

223.) A client with a psychotic disorder is being treated with haloperidol (Haldol). Which of the following would indicate the presence of a toxic effect of this medication? 1. Nausea 2. Hypotension 3. Blurred vision 4. Excessive salivation

4. Excessive salivation Rationale: Toxic effects include extrapyramidal symptoms (EPS) noted as marked drowsiness and lethargy, excessive salivation, and a fixed stare. Akathisia, acute dystonias, and tardive dyskinesia are also signs of toxicity. Hypotension, nausea, and blurred vision are occasional side effects.

231.) A client admitted to the hospital gives the nurse a bottle of clomipramine (Anafranil). The nurse notes that the medication has not been taken by the client in 2 months. What behaviors observed in the client would validate noncompliance with this medication? 1. Complaints of hunger 2. Complaints of insomnia 3. A pulse rate less than 60 beats per minute 4. Frequent handwashing with hot, soapy water

4. Frequent handwashing with hot, soapy water Rationale: Clomipramine is commonly used in the treatment of obsessive-compulsive disorder. Handwashing is a common obsessive-compulsive behavior. Weight gain is a common side effect of this medication. Tachycardia and sedation are side effects. Insomnia may occur but is seldom a side effect.

After intentionally taking an overdose of hydrocodone (vicodine), a client is admitted to the emergency department. Activated charcoal is prescribed. Before administering the drug, the nurse should ensure that the client: 1. Is able to follow commands. 2. Has nasogastric (NG) tube in place. 3. Has an advance directive on file. 4. Has audible bowel sounds.

4. Has audible bowel sounds. activated charcoal binds to the ingested drug and is eliminated in the stool. therefore, the client should have audible sounds before the drug is given. being able to follow commands isn't required.

192.) A nurse is collecting medication information from a client, and the client states that she is taking garlic as an herbal supplement. The nurse understands that the client is most likely treating which of the following conditions? 1. Eczema 2. Insomnia 3. Migraines 4. Hyperlipidemia

4. Hyperlipidemia Rationale: Garlic is an herbal supplement that is used to treat hyperlipidemia and hypertension. An herbal supplement that may be used to treat eczema is evening primrose. Insomnia has been treated with both valerian root and chamomile. Migraines have been treated with feverfew.

73.) A client with myasthenia gravis is suspected of having cholinergic crisis. Which of the following indicate that this crisis exists? 1. Ataxia 2. Mouth sores 3. Hypotension 4. Hypertension

4. Hypertension Rationale: Cholinergic crisis occurs as a result of an overdose of medication. Indications of cholinergic crisis include gastrointestinal disturbances, nausea, vomiting, diarrhea, abdominal cramps, increased salivation and tearing, miosis, hypertension, sweating, and increased bronchial secretions.

76.) Carbidopa-levodopa (Sinemet) is prescribed for a client with Parkinson's disease, and the nurse monitors the client for adverse reactions to the medication. Which of the following indicates that the client is experiencing an adverse reaction? 1. Pruritus 2. Tachycardia 3. Hypertension 4. Impaired voluntary movements

4. Impaired voluntary movements Rationale: Dyskinesia and impaired voluntary movement may occur with high levodopa dosages. Nausea, anorexia, dizziness, orthostatic hypotension, bradycardia, and akinesia (the temporary muscle weakness that lasts 1 minute to 1 hour, also known as the "on-off phenomenon") are frequent side effects of the medication.

212.) Mannitol (Osmitrol) is being administered to a client with increased intracranial pressure following a head injury. The nurse assisting in caring for the client knows that which of the following indicates the therapeutic action of this medication? 1. Prevents the filtration of sodium and water through the kidneys 2. Prevents the filtration of sodium and potassium through the kidneys 3. Decreases water loss by promoting the reabsorption of sodium and water in the loop of Henle 4. Induces diuresis by raising the osmotic pressure of glomerular filtrate, thereby inhibiting tubular reabsorption of water and solutes

4. Induces diuresis by raising the osmotic pressure of glomerular filtrate, thereby inhibiting tubular reabsorption of water and solutes Rationale: Mannitol is an osmotic diuretic that induces diuresis by raising the osmotic pressure of glomerular filtrate, thereby inhibiting tubular reabsorption of water and solutes. It is used to reduce intracranial pressure in the client with head trauma.

225.) A nursing student is assigned to care for a client with a diagnosis of schizophrenia. Haloperidol (Haldol) is prescribed for the client, and the nursing instructor asks the student to describe the action of the medication. Which statement by the nursing student indicates an understanding of the action of this medication? 1. It is a serotonin reuptake blocker. 2. It inhibits the breakdown of released acetylcholine. 3. It blocks the uptake of norepinephrine and serotonin. 4. It blocks the binding of dopamine to the postsynaptic dopamine receptors in the brain.

4. It blocks the binding of dopamine to the postsynaptic dopamine receptors in the brain. Rationale: Haloperidol acts by blocking the binding of dopamine to the postsynaptic dopamine receptors in the brain. Imipramine hydrochloride (Tofranil) blocks the reuptake of norepinephrine and serotonin. Donepezil hydrochloride (Aricept) inhibits the breakdown of released acetylcholine. Fluoxetine hydrochloride (Prozac) is a potent serotonin reuptake blocker.

200.) A client is seen in the clinic for complaints of skin itchiness that has been persistent over the past several weeks. Following data collection, it has been determined that the client has scabies. Lindane is prescribed, and the nurse is asked to provide instructions to the client regarding the use of the medication. The nurse tells the client to: 1. Apply a thick layer of cream to the entire body. 2. Apply the cream as prescribed for 2 days in a row. 3. Apply to the entire body and scalp, excluding the face. 4. Leave the cream on for 8 to 12 hours and then remove by washing.

4. Leave the cream on for 8 to 12 hours and then remove by washing. Rationale: Lindane is applied in a thin layer to the entire body below the head. No more than 30 g (1 oz) should be used. The medication is removed by washing 8 to 12 hours later. Usually, only one application is required.

A client with gastrointestinal hypermotility has a prescription to receive atropine sulfate. The nurse should withhold the medication and question the prescription if the client has a history of which disease process? 1. Biliary colic 2. Sinus bradycardia 3. Peptic ulcer disease 4. Narrow-angle glaucoma

4. Narrow-angle glaucoma Atropine sulfate can cause a blockade of muscarinic receptors on the iris sphincter, producing mydriasis (dilation of the pupils). It also produces cycloplegia (relaxation of the ciliary muscles). It is contraindicated in clients with narrow-angle glaucoma. The other options are therapeutic reasons for using the medication. Atropine is a mydriatics contraindicated in clients with glaucoma because of the risk of (IOP). Increased intraocular pressure.

143.) A client has just taken a dose of trimethobenzamide (Tigan). The nurse plans to monitor this client for relief of: 1. Heartburn 2. Constipation 3. Abdominal pain 4. Nausea and vomiting

4. Nausea and vomiting Rationale: Trimethobenzamide is an antiemetic agent used in the treatment of nausea and vomiting. The other options are incorrect.

39.) The client with a gastric ulcer has a prescription for sucralfate (Carafate), 1 g by mouth four times daily. The nurse schedules the medication for which times? 1. With meals and at bedtime 2. Every 6 hours around the clock 3. One hour after meals and at bedtime 4. One hour before meals and at bedtime

4. One hour before meals and at bedtime Rationale: Sucralfate is a gastric protectant. The medication should be scheduled for administration 1 hour before meals and at bedtime. The medication is timed to allow it to form a protective coating over the ulcer before food intake stimulates gastric acid production and mechanical irritation. The other options are incorrect.

2.) Oral iron supplements are prescribed for a 6-year-old child with iron deficiency anemia. The nurse instructs the mother to administer the iron with which best food item? 1. Milk 2. Water 3. Apple juice 4. Orange juice

4. Orange juice Rationale: Vitamin C increases the absorption of iron by the body. The mother should be instructed to administer the medication with a citrus fruit or a juice that is high in vitamin C. Milk may affect absorption of the iron. Water will not assist in absorption. Orange juice contains a greater amount of vitamin C than apple juice.

15.) The client with small cell lung cancer is being treated with etoposide (VePesid). The nurse who is assisting in caring for the client during its administration understands that which side effect is specifically associated with this medication? 1. Alopecia 2. Chest pain 3. Pulmonary fibrosis 4. Orthostatic hypotension

4. Orthostatic hypotension Rationale: A side effect specific to etoposide is orthostatic hypotension. The client's blood pressure is monitored during the infusion. Hair loss occurs with nearly all the antineoplastic medications. Chest pain and pulmonary fibrosis are unrelated to this medication.

184.) A nurse preparing a client for surgery reviews the client's medication record. The client is to be nothing per mouth (NPO) after midnight. Which of the following medications, if noted on the client's record, should the nurse question? 1. Cyclobenzaprine (Flexeril) 2. Alendronate (Fosamax) 3. Allopurinol (Zyloprim) 4. Prednisone

4. Prednisone Rationale: Prednisone is a corticosteroid that can cause adrenal atrophy, which reduces the body's ability to withstand stress. Before and during surgery, dosages may be temporarily increased. Cyclobenzaprine is a skeletal muscle relaxant. Alendronate is a bone-resorption inhibitor. Allopurinol is an antigout medication.

189.) Prostaglandin E1 is prescribed for a child with transposition of the great arteries. The mother of the child asks the nurse why the child needs the medication. The nurse tells the mother that the medication: 1. Prevents hypercyanotic (blue or tet) spells 2. Maintains an adequate hormone level 3. Maintains the position of the great arteries 4. Provides adequate oxygen saturation and maintains cardiac output

4. Provides adequate oxygen saturation and maintains cardiac output Rationale: A child with transposition of the great arteries may receive prostaglandin E1 temporarily to increase blood mixing if systemic and pulmonary mixing are inadequate to maintain adequate cardiac output. Options 1, 2, and 3 are incorrect. In addition, hypercyanotic spells occur in tetralogy of Fallot. **Use the ABCs—airway, breathing, and circulation—to answer the question. The correct option addresses circulation**

144.) A client is taking docusate sodium (Colace). The nurse monitors which of the following to determine whether the client is having a therapeutic effect from this medication? 1. Abdominal pain 2. Reduction in steatorrhea 3. Hematest-negative stools 4. Regular bowel movements

4. Regular bowel movements Rationale: Docusate sodium is a stool softener that promotes the absorption of water into the stool, producing a softer consistency of stool. The intended effect is relief or prevention of constipation. The medication does not relieve abdominal pain, stop gastrointestinal (GI) bleeding, or decrease the amount of fat in the stools.

166.) Alendronate (Fosamax) is prescribed for a client with osteoporosis. The client taking this medication is instructed to: 1. Take the medication at bedtime. 2. Take the medication in the morning with breakfast. 3. Lie down for 30 minutes after taking the medication. 4. Take the medication with a full glass of water after rising in the morning.

4. Take the medication with a full glass of water after rising in the morning. Rationale: Precautions need to be taken with administration of alendronate to prevent gastrointestinal side effects (especially esophageal irritation) and to increase absorption of the medication. The medication needs to be taken with a full glass of water after rising in the morning. The client should not eat or drink anything for 30 minutes following administration and should not lie down after taking the medication.

67.) Phenazopyridine hydrochloride (Pyridium) is prescribed for a client for symptomatic relief of pain resulting from a lower urinary tract infection. The nurse reinforces to the client: 1. To take the medication at bedtime 2. To take the medication before meals 3. To discontinue the medication if a headache occurs 4. That a reddish orange discoloration of the urine may occur

4. That a reddish orange discoloration of the urine may occur Rationale: The nurse should instruct the client that a reddish-orange discoloration of urine may occur. The nurse also should instruct the client that this discoloration can stain fabric. The medication should be taken after meals to reduce the possibility of gastrointestinal upset. A headache is an occasional side effect of the medication and does not warrant discontinuation of the medication.

177.) A nurse is assisting in caring for a pregnant client who is receiving intravenous magnesium sulfate for the management of preeclampsia and notes that the client's deep tendon reflexes are absent. On the basis of this data, the nurse reports the finding and makes which determination? 1. The magnesium sulfate is effective. 2. The infusion rate needs to be increased. 3. The client is experiencing cerebral edema. 4. The client is experiencing magnesium toxicity.

4. The client is experiencing magnesium toxicity. Rationale: Magnesium toxicity can occur as a result of magnesium sulfate therapy. Signs of magnesium sulfate toxicity relate to the central nervous system depressant effects of the medication and include respiratory depression; loss of deep tendon reflexes; sudden decrease in fetal heart rate or maternal heart rate, or both; and sudden drop in blood pressure. Hyperreflexia indicates increased cerebral edema. An absence of reflexes indicates magnesium toxicity. The therapeutic serum level of magnesium for a client receiving magnesium sulfate ranges from 4 to 7.5 mEq/L (5 to 8 mg/dL).

232.) A client in the mental health unit is administered haloperidol (Haldol). The nurse would check which of the following to determine medication effectiveness? 1. The client's vital signs 2. The client's nutritional intake 3. The physical safety of other unit clients 4. The client's orientation and delusional status

4. The client's orientation and delusional status Rationale: Haloperidol is used to treat clients exhibiting psychotic features. Therefore, to determine medication effectiveness, the nurse would check the client's orientation and delusional status. Vital signs are routine and not specific to this situation. The physical safety of other clients is not a direct assessment of this client. Monitoring nutritional intake is not related to this situation.

160.) Meperidine hydrochloride (Demerol) is prescribed for the client with pain. Which of the following would the nurse monitor for as a side effect of this medication? 1. Diarrhea 2. Bradycardia 3. Hypertension 4. Urinary retention

4. Urinary retention Rationale: Meperidine hydrochloride (Demerol) is an opioid analgesic. Side effects of this medication include respiratory depression, orthostatic hypotension, tachycardia, drowsiness and mental clouding, constipation, and urinary retention.

140.) The client has a new prescription for metoclopramide (Reglan). On review of the chart, the nurse identifies that this medication can be safely administered with which condition? 1. Intestinal obstruction 2. Peptic ulcer with melena 3. Diverticulitis with perforation 4. Vomiting following cancer chemotherapy

4. Vomiting following cancer chemotherapy Rationale: Metoclopramide is a gastrointestinal (GI) stimulant and antiemetic. Because it is a GI stimulant, it is contraindicated with GI obstruction, hemorrhage, or perforation. It is used in the treatment of emesis after surgery, chemotherapy, and radiation.

180.) A health care provider (HCP) writes a prescription for digoxin (Lanoxin), 0.25 mg daily. The nurse teaches the client about the medication and tells the client that it is important to: 1. Count the radial and carotid pulses every morning. 2. Check the blood pressure every morning and evening. 3. Stop taking the medication if the pulse is higher than 100 beats per minute. 4. Withhold the medication and call the HCP if the pulse is less than 60 beats per minute.

4. Withhold the medication and call the HCP if the pulse is less than 60 beats per minute. Rationale: An important component of taking this medication is monitoring the pulse rate; however, it is not necessary for the client to take both the radial and carotid pulses. It is not necessary for the client to check the blood pressure every morning and evening because the medication does not directly affect blood pressure. It is most important for the client to know the guidelines related to withholding the medication and calling the HCP. The client should not stop taking a medication.

185.) Which of the following herbal therapies would be prescribed for its use as an antispasmodic? Select all that apply. 1.Aloe 2.Kava 3.Ginger 4.Chamomile 5.Peppermint oil

4.Chamomile 5.Peppermint oil Rationale: Chamomile has a mild sedative effect and acts as an antispasmodic and anti-inflammatory. Peppermint oil acts as an antispasmodic and is used for irritable bowel syndrome. Topical aloe promotes wound healing. Aloe taken orally acts as a laxative. Kava has an anxiolytic, sedative, and analgesic effect. Ginger is effective in relieving nausea.

An HDL of ____ or higher reduces heart disease risk

60mg/dL

Autonomic nervous system

A branch of the peripheral nervous system that controls autonomic bodily functions. Consists of the sympathetic nervous system and the parasympathetic nervous system.

Benzodiazepines

A chemical category of drugs most frequently prescribed as anxiolytic drugs and less frequently as sedative-hypnotic agents.

Alpha-adrenergic receptors

A class of adrenergic receptors that are further subdivided into alpha1 and alpha2-adrenergic receptors

Barbiturates

A class of drugs that are chemical derivatives of barbituric acid. Used to induce sedation. Can produce many unwanted adverse effects. Are physiologically habit forming, and have a low therapeutic index.

Carbonic Anhydrase Inhibitors CAI

A diuretic that can induce metabolic/respiratory acidosis Elevate blood glucose

Atony

A lack of normal muscle tone

Autoinduction

A metabolic process in which a drug stimulates the production of enzymes that enhance its own metabolism over time, which leads to a reduction in therapeutic drug concentrations

Autoinduction

A metabolic process in which a drug stimulates the production of enzymes that enhance its own metabolism over time, which leads to a reduction in therapeutic drug concentrations.

Cholinergic receptor

A nerve receptor that is stimulated by acetylcholine.

Electroencephalogram (EEG)

A recording of the electrical activity that arises from spontaneous currents in nerve cells in the brain, derived from electrodes placed on the outer skull.

Status epilepticus

A seizure disorder characterized by generalized tonic-clonic convulsions that occur repeatedly; considered a medical emergency.

Dopaminergic receptor

A third type of adrenergic receptor (in addition to alpha and beta-adrenergic receptors) located in various tissues and organs and activated by the binding of the neurotransmitter dopamine, which can be either endogenous or a synthetic drug form.

Muscle Relaxants

A variety of conditions such as trauma, inflammation, anxiety, and pain can be associated with acute muscle spasms. Group of compounds that act predominantly within the CNS to relieve pain associated with skeletal muscle spasms.

A nurse if preparing a teaching plan for a client who was prescribed enalapril maleate (Vasotec) for the treatment of hypertension. Which instructions should the nurse include in the teaching plan? SATA A. Instruct the client to avoid salt substitutes. B. Tell the client that light-headedness is a common adverse effect that does not need to be reported C. Inform the client of a potential sore throat for the first few days of therapy D. Advise the client to report facial swelling or difficulty breathing immediately E. Tell the client that blood tests will be necessary every 3 weeks for 2 months and periodically after that F. Advise the client not to change the position suddenly to minimize orthostatic hypotension

A. Instruct the client to avoid salt substitutes D. Advise the client to report facial swelling or difficulty breathing immediately F. Advise the client not to change the position suddenly to minimize orthostatic hypotension. The nurse should tell the client to avoid salt substitutes because they may contain potassium, which can cause light-headedness and syncope. Facial swelling or difficulty breathing should be reported immediately because they may be signs of angio edema, which would require discontinuation of the drug. The client should also be advised to change the position slowly to minimize orthostatic hypotension. The nurse should tell the client to report light-headedness, especially during the first few days of therapy, so dosage adjustments can be made. The client should also report signs of infection, such as sore throat and fever, because the drug may decrease the white blood cell (WBC) count. Because this effect is generally seen within 3 months, the WBC count and differential should be monitored periodically.

What antihypertensive is known for its benefits in protecting kidneys and preventing diabetic neuropathy

ACE Inhibitor

captopril

ACE inhibitor, antihypertensive avoid foods high in potassium e.g. broccoli, bananas, and salt substitutes

ACE Inhibitors Mechanism/Therapeutic Effects

ACE inhibitors prevent vasoconstriction and the retention of sodium and water. Preload and afterload are decreased. The therapeutic effects of ACE inhibitors are decreased blood pressure and diuresis.

Pharmacokinetics/Dosing of Aminoglutethimide

Absorbed orally half-life of 13 hours, which is reduced to 7 hours after chronic therapy Time to peak concentration is 1.5 hours, w/ adrenal function suppression occurring w/i 3 to 5 days of therapy. Metabolized in the liver and excreted by the kidneys. Adult oral dosage is 250 mg two or three times daily for approx. 14 days. Maintenance dosage is 250 mg q6hr. Pediatric dosage unestablished.

Expectorant

Acetylcysteine (Mucomyst) Increase fluid intake and watch for GI upset

Loop Diuretics

Act on loop of henle: Furosemide

Individuals taking metronidazole (Flagyl) must avoid which of the following substances or suffer severe headache with nausea and vomiting?

Alcohol

Acetaminophen Interactions

Alcohol hepatotoxic drugs phenytoin barbiturates warfarin isoniazid rifampin beta blockers anticholinergic drugs

Interactions with benzodiazepines in anxiolytic treatment

Alcohol and CNS depressants, when combined with benzos, can result in additive CNS depression and even death. More likely to occur in patients with renal and/or hepatic compromise.

What type of antacid causes constipation

Aluminum (Maalox, Amphojel)/Calcium containing (Tums)

Both calcium and magnesium based antacids are more likely to accumulate to toxic levels in patients with renal disease, an alternative would be

Aluminum containing antacids (Mylanta/Maalox/Amphojel)

Drug interactions with Aminoglutethimide

Aminoglutethimide may interfere with the effect of warfarin.

Class III antidysrhythmic drugs

Amiodarone

Tricyclic (TCAs)

Amitriptyline (Elavil) adverse effects: dry mouth, tachycardia, constipation, retention, reflux, orthostatic hypotension Administer at night Contraindicated with cardiac disease

Gabapentin (Neurontin)

An anti epileptic also used to treat neuropathic pain

Atropine

An anticholinergic drug given preoperatively to dry secretions

Doxapram (Dopram)

Analeptic that is commonly used in conjunction with supportive measures in cases of respiratory depression that involve anesthetics or drugs of abuse and in COPD-associated hypercapnia. Deep tendon reflexes, in addition to vitals and heart rhythm, are monitored to prevent OD.

Treatment for Acute Pain

Analgesics (opioids), NSAIDS, COX-2 inhibitors, and non-narcotic agents (tramadol)

Antidiarrheal drug that acts by decreasing peristalsis and muscular tone of the intestine, thus slowing the movement of substances through the GI tract

Anticholenergics (severe)

Anticholinergic antiparkinson agents

Anticholinergic antiparkinson agents or acetylcholine antagonists block the muscarinic acetylcholine receptors and cholinergic nerve activity. Activation of muscarinic receptors has an excitatory effect, opposite to that of dopaminergic activation, so suppression of the effects of acetylcholine compensates for a lack of dopamine in Parkinson Ex: Diphenhydramine

Anticholinergic bronchodilators

Anticholinergic bronchodilators (or muscarinic receptor antagonists) block the parasympathetic nerve reflexes that cause the airways to constrict, so allow the air passages to remain open. Muscarinic receptor antagonists bind to muscarinic receptors and inhibit acetylcholine mediated bronchospasm. Anticholinergic bronchodilators are used more to treat chronic obstructive pulmonary disease than to treat asthma. Ex: Atrovent, Spiriva

Interactions w/ cholinergic drugs

Anticholinergics (atropine) Antihistamines Sympathomimetics may antagonize cholinergic drugs and lead to a reduced response to them. Other cholinergic drugs may have additive effects.

What type of drugs can be given to relieve neuropathic pain?

Anticonvulsants

Interactions with antipsychotics

Antihypertensives may have additional hypotensive effects and CNS depressants may have additive CNS depressant effects when taken with antipsychotics. Grapefruit juice can enhance the effects of clozapine.

Adverse effects of fibric acid derivatives

Antilipemic: Gemfibrozil (Lopid): Diarrhea, HA, blurred vision

Aminoglutethimide (Cytadren)

Antirenal or adrenal steroid inhibitor that inhibits or suppresses adrenal cortex function. It inhibits the enzyme conversion of cholesterol or pregnenolone (hormone precursor), thereby blocking the synthesis of adrenal steroids. May also have other suppression effects in the synthesis and metabolism of the steroids. It also inhibits estrogen production from androgens by blocking an enzyme in there peripheral tissue and may also enhance the metabolism of estrone; thus it is investigation ally used to treat breast cancer.

What antiplatelet is recommended for stroke prevention?

Aspirin

Valproic Acid and Derivative

Aspirin may increase amount of free drug and result in hepatotoxicity

NSAIDs

Aspirin, ibuprofen, naproxen, cyclooxygenase-2 inhibitor celcoxib, etc. Nonopioid analgesics. Indications: Management of pain, esp. pain associated w. inflammatory conditions such as arthritis bc they have significant anti-inflammatory effects in a edition to their analgesic effects.

Nursing Implications of ACE Inhibitors

Assess blood pressure, apical pulse, and respiratory status prior to administration. Withhold medication if serum potassium >5 mEq/L and notify the health care provider. Monitor for angioedema and notify the health care provider immediately if it occurs. Monitor sodium and fluid volume status. Monitor dietary and fluid intake because ACE inhibitors can cause anorexia secondary to impaired taste.

Adverse effects of analeptics

At higher dosages, analeptics stimulate the vagal, vasomotor, and respiratory centers of the medulla in the brainstem, as well as increasing blood flow to skeletal muscles. Vagal effects include: stimulation of gastric secretions diarrhea reflex tachycardia Vasomotor effects include: flushing sweating Respiratory effects include: elevated resp rate Skeletal muscle effects include: muscular tension tremors Neurologic effects include: reduced deep tendon reflexes

Risperidone

Atypical antipsychotic, even more effective than clozapine at the serotonin receptors. Also has a high affinity for alpha1- and alpha2-adrenergic receptors and histamine H1 receptors. -Indicated for schizophrenia, including negative symptoms, and causes minimal extrapyramidal adverse effects at therapeutic dosages of 1 to 6 mg/day. -Contraindicated in cases of allergy. -Adverse effects include elevated prolactin levels, abnormal dreams, insomnia, dizziness, headache. -Drug interactions include: CNS depressants, antihypertensives.

oxycodone hydrochloride

Available in immediate-release and sustained-release.

A nurse is caring for a neonate with a suspected diaphragmatic hernia. The nurse should question an order for : A. Chest xray B. Mask ventilation C. Placement of an orogastric tube D. Immediate endotracheal intubation

B. Mask ventilation The nurse should question an order for mask ventilation because this procedure may introduce air into the neonate's GI track. An emergency chest xray can help diagnose diagphramatic hernia. An orogastric tube is appropriate because it decompresses the bowel and stomach within the neonate's chest. Intubation is needed to ventilate the neonate.

A 65-year-old patient with pneumonia is receiving GARAMYCIN. It would be moSt important for a nurse to monitor which of the following laboratory values in this patient?

BUN and creatinine nephrotoxic; will see proteinuria, oliguria, hematuria, thirst, increased BUN, decreased creatine clearance

An antibiotic that inhibits protein synthesis in a microorganism but does not kill the organism is known as what type of agent?

Bacteriostatic

Which of the following is a characteristic of tetracycline?

Bacteriostatic

Adverse effects of antiepileptics

Barbiturates (phenobarbital, primidone): - dizziness, drowsiness, lethargy, paradoxical restlessness Hydantoins (phenytoin, fosphenytoin): - nystagmus, ataxia, drowsiness, rash, gingival hyperplasia, thrombocytopenia, agranulocytosis, hepatitis Iminostilbenes (carbamasepine, oxcarbazepine): - Nausea, headache, dizziness, unusual eye movements, visual change, behavioral changes, rash, abd pain, abnormal gait. Valproic acid and derivatives: - dizziness, drowsiness, GI upset, weight gain, hepatotoxy, pancreastitis

Interactions with antiepileptics

Barbiturates: BBs, corticosteroids, oral contraceptives, dihydropyridine, CCBs, metronidazole, quinidine, theophylline, alcohol. Hydantoins: amiodarone, benzos, azole antifungals, isoniazid, PPIs, sulfonamide antibiotics, SSRIs, carbamazepine, cyclosporine, loop diuretics, meperidine, methadone, rifampin, quinidein, questiapine, theophylline, zonisamide, warfarin. IIminostilbenes: Azole antifungals, diltiazem, isoniazid, macrolides, protease inhibitor antiretrovirals, SSRIs, valproid acid, verapamil, barbiturates, hydantions, rifampin, succinimides, theophylline, acetaminophen.

Atomoxetine

Being used to treat ADHD, works in the CNS by selective inhibition of norepinephrine reuptake.

Vascular Pain

Believed to originate from the vascular per perivascular tissues and is thought to account for a large percentage of migraine headaches.

Indications for anxiolytic use

Benzodiazepines are indicated for: Anxiety Ethanol withdrawal (alcohol withdrawal) Insomnia and muscle spasms seizure disorders Adjuncts in anesthesia Adjunct therapy for depression

Class II antidysrhythmic drugs

Beta blockers

Dobutamine (Dobutrex)

Beta1-selective vasoactive adrenergic drug that is structurally similar to the naturally occurring catecholamine dopamine. Through stimulation of the beta1 receptors on heart muscle, it increases cardiac output by increasing the stroke volume, especially in patients in patients with heart failure.

Direct-acting cholinergic agonists

Bind directly to cholinergic receptors and activate them. bethanechol carbachol pilocarpine succinylcholine

Reversible cholinesterase inhibitors

Bind to cholinesterase for a short period of time.

Direct-acting sympathomimetic

Binds directly to the receptor and causes a physiologic response. Ex. Epinephrine

During the client's dialysis, the nurse observes that the solution draining from the abdomen is consistently blood tinged. The client has a permanent peritoneal catheter in place. Which interpretation of this observation would be correct?

Bleeding indicates abdominal blood vessel damage Because the client has a permanent catheter in place, blood tinged drainage should not occur. Persistent blood tinged drainage could indicate damage to the abdominal vessels, and the physician should be notified. The bleeding is originating in the peritoneal cavity, not the kidneys. Too rapid infusion of the dialysate can cause pain

Cholinergic-blocking drugs

Block the action of acetylcholine and substances similar to acetylcholine at receptor sites in the synapse. (cholinergic blockers, anticholinergics, parasympatholytics, antimuscarinic drugs)

Your patient calls after taking an antibiotic for 8 days. He reports that his sinus infection is much better, but now he has bruises on his arms and legs and a sore throat. You suspect:

Bone Marrow Toxicity

Mixed-acting sympathomimetic

Both directly stimulates the receptor by binding to it and indirectly stimulates the receptor by causing the release of the neurotransmitter stored in vesicles at the nerve endings. Ex. Ephedrine

Symptoms of digoxin toxicity

Bradycardia, HA, dizziness, confusion, nausea, blurred vision (Yellow)

Respiratory indications for adrenergic drugs

Bronchodilators are adrenergic drugs that have an affinity for the adrenergic receptors located in the respiratory system. Tend to preferentially stimulate the beta2-adrenergic receptors and cause bronchodilation. Of the two subtypes, these drugs are attracted more to the beta2-adrenergic receptor receptors located on the bronchial, uterine, and vascular smooth muscles as opposed to the beta1-adrenergic receptors located on the heart. Helpful for treating conditions such as asthma and bronchitis.

Laxative that absorb water into the intestine, increasing the volume and distending the bowel

Bulk-Forming

Norepinephrine Dopamine Reuptake Inhibitor (NDRIs)

Bupropion (Wellbutrin/Zyban)

Client newly diagnosed with diabetes is started on ACARBOSE. The nurse should tell the client that the medication should be taken A 30 mins after meals B 1 hour before meals C with the first bite of a meal D daily at bedtime

C with the first bite of a meal comes in a tablet helps treat type 2 diabetes

"The nurse is reviewing the medication record of a female client with acute gastritis. Which medication, if noted on the client's record, would the nurse question? A. Digoxin (Lanoxin) B. Furosemide (Lasix) C. Indomethacin (Indocin) D. Propranolol hydrochloride (Inderal)"

C. Indomethacin (Indocin) is an NSAID and can cause ulceration of the esophagus, stomach, or small stomach, or small intestine. indomethacin is contraindicated in a client with gastrointestinal disorders.

Class IV antidysrhythmic drugs

CCBs: Verapamil, diltaizim

What drug is indicated in vasospastic angina?

CCBs: Verapamil, diltiazem, amlodipine

a pt admitted to the hospital after a severe hypersensitivity reaction to PENICILLIN. which of the following classes of antibiotics should the pt avoid in the future?

CEPHALOSPORIN and CARBAPENEMS

You are checking medication orders that were received by telephone for a client with rheumatoid arthritis who was admitted with METHOTREXATE toxicity. Which order is most important to clarify with the physician? 1. Administer chlorambucil (Leukeran) 4 mg PO daily 2. Infuse normal saline at 250 mL/hr for 4 hours 3. Administer folic acid (Folacin) 2000 mcg PO daily 4. Give cyanocobalamin (vitamin B12) 10,000 mcg PO

CHLORAMBUCIL is an antineoplastic drug used to treat cancer.

Mechanism of action of barbiturates

CNS depressants that act primarily on the brainstem in an area called the reticular formation. Sedative and hypnotic effete are dose related, and the act by reducing the nerve impulses traveling to the area of the brain called the cerebral cortex. Ability to inhibit nerve impulse transmission is due in part to their ability to potentiate the action of the inhibitory neurotransmitter GABA, which is found in high concentrations in the CNS. Also raise the seizure threshold and can be used to treat seizures.

Caffeine

CNS stimulant tat can be found in OTC drugs and combo prescription drugs, also food and beverages. Contraindicated in patients with: Known hypersensitivity Used with caution in patients who have a history of peptic ulcers or cardiac dysrhythmias or who have recently experienced an MI. Available in oral and injectable forms

Sympathomimetic Drugs

CNS stimulants such as noradrenergic drugs (and, to a lesser degree, dopaminergic drugs) whose actions resemble or mimic those of the sympathetic nervous system.

the tx of choice for Systemic Lupus Erythematosus

CORTICOSTEROIDS

What type of antacid causes kidney stones

Calcium (tums)

Cancer Pain

Can be acute or chronic or both. Most often results from pressure of the tumor mass against nerves, organs, or tissues. Other causes of cancer pain include hypoxia from blockage of blood supply to an organ, metastases, pathologic fractures, muscle spasms and adverse effects of radiation surgery, and chemo.

Indications for barbiturate use

Can be used as hypnotics, sedatives, and anticonvulsants and also for anesthesia during surgical procedures. NO LONGER RECOMMENDED FOR SLEEP INDUCTION.

First line anti epileptics

Carbamazepin (tergetrol) Pehnobarbital Phenytoin Valproic acid

What patient need to avoid caffeine?

Cardiac Dysrhythmias

Adverse Effects of lithium

Cardiac dysrhythmias drowsiness slurred speech epilepsy-type seizures chroeoathetotic movements ataxia hypotension

A1 blockers

Cardura, Minipress, Flomax, and Hytrin used for HTN, BPH, and concurrently with digoxin in HF

A2 blocker

Catapress (clonidine) decrease blood pressure, given after other drugs fail due to sedation

Nicotinic receptors

Cholinergic receptors located in the ganglia (where presynaptic and postsynaptic nerve fibers meet) of both the parasympathetic nervous system and the sympathetic nervous system; so named bc they can be stimulated by the alkaloid nicotine.

Muscarinic receptors

Cholinergic receptors that are located post-synaptically in the effector organs such as smooth muscle, cardiac muscle, and glands supplied by parasympathetic fibers.

Serotonin Receptor Agonists

Class of CNS stimulants used to treat migraine headaches; work by stimulating 5-hydroxytryptamine 1 receptors in the brain and are sometimes referred to as selective serotonin receptor agonists or triptans.

Modafinil

Classified as an analeptic. Promotes wakefulness like the amphetamines and phenidates. Lacks sympathomimetic properties, however, and appears to work primarily by reducing GABA-mediated neurotransmission in the brian.

Sedative-Hypnotic Benzodiazepines

Clonazepam (Klonopin) Diazepam (Valium) Alprazolam (Xanax) Lorazepam (Ativan) Temazepam (Restoril) Midazolam (Versed) Triazolam (Halcion)

Mild Agonists

Codeine, and hydrocodone.

Indications for use of drugs such as fentanyl, sufentanil, and alfentanil

Commonly used in combo w/ anesthetics during surgery; relieve pain and maintain balanced state of anesthesia.

Drug Interactions of Fludrocortisone

Concurrent use of P450 enzyme inducers (rifampin, phenobarbital, phenytoin, and carbamazepine) may result in increased metabolism and reduced efficacy of fludrocortisone. Concurrent use of digoxin: hypokalemic effect of fludrocortisone may potentiate the risk for cardiac dysrhythmias or dig tox. Concurrent use of hepatic enzyme inducers: Effectiveness of fludro. may be decreased, and a dosage requirement may be required. Concurrent used of potassium-depleting diuretics or hypokalemic-inducing meds may produce severe hypokalemia. In type IV renal tubular acidosis: Concurrent use of sodium-containing foods or meds w/ fludro. may result in HTN, hypernatremia, and edema.

Cataplexy

Condition characterized by abrupt attacks of muscular weakness and hypotonia triggered by an emotional stimulus such as joy, laughter, anger, fear, or surprise. Often associated with narcolepsy.

Potential Opioid Adverse Effects and Their Management (1 of 3)

Constipation: Opioids decrease GI tract peristalsis bc of their CNS depression. Stool becomes excessively dehydrate bc it remains in the GI tract longer. May be managed w/ increased fluid intake, stool softeners such as docusate sodium, or the use of stimulants such as bisacodyl or senna. Agents such as lactulose, sorbitol, and polyethylene glycol (Miralax) have proven effective. N/V: Opioids decrease GI tract peristalsis, and some also stimulate the vomiting center in the CNS, so nausea and vomiting are often experienced. May be managed with the use of anti emetics such as phenothiazines

Miosis

Contraction of the pupil.

Adenosine

Converts to sinus rhythm

Glucocorticoids and the CNS

Corticosteroids affect mood and behavior and possibly cause neuronal or brain excitability. Some persons report euphoria, insomnia, anxiety, depression, or increased motor activity, or the may become psychotic.

Indications for use of TCAs

Currently most commonly used to treat neuropathic pain syndromes and insomnia. Use as an antidepressant is rare. Imipramine is used as an adjunct in the treatment of childhood enuresis (bedwetting), and clomipramine is useful in the treatment of OCD. Bc TCAs tend to increase appetite leading to weight gain, they are sometimes used to treat anorexia nervosa.

Which food is capable of producing a hypertensive crisis when taken with Parnate(tranylcypromine) A cottage cheese B processed cheese C cream cheese D cheddar cheese

D cheddar cheese You can not eat aged cheese, and cheddar is one.

A student nurse is teaching a patient about how to take antacids. Which of these statement made by the patient ensures the patient teaching has been effective: A. "I should take this medication at the same time I take my other medications." B. "I need to drink a full glass of water when taking this medication." C. "I can take this medication for as long as I need it." D. "I need to take this medication 1 hour after or 2 hours before taking any other medication."

D. "I need to take this medication 1 hour after or 2 hours before taking any other medication."

a client is taking CARBAMAZEPINE. For which of the following potential complications should the nurse be monitoring the client? 1. acute respiratory distress syndrome 2. diplopia 3. elevated levels of phenytoin 4. leukocytosis

DIPLOPIA complications of carbamazepine include diplopia, dizziness, ataxia, and rash.

Smoking has been shown to ____ the effectiveness of H2 antagonists like Cimetidine, famotidine, rantitidine)

Decrease

Mechanism of action and effects of anxiolytics

Decrease anxiety by reducing overactivity in the CNS. Benzodiazepines exert their effects by depressing activity in the areas of the brain called the brainstem and the limbic system. Believed to increase the action of GABA, which is an inhibitory neurotransmitter in the brain that blocks nerve transmission in the CNS.

Diuretics

Decrease the plasma and extracellular fluid volumes results in reduced preload (HCTZ)

Eszopiclone (Lunesta)

Designed to provide a full 8 hours of sleep. Considered a short- to intermediate-acting agent. Patients should allot 8 hours of sleep time and should avoid taking hypnotics when they must awaken in less than 6-8 hours.

Antitussives

Dextromethorphan Reduces nonproductive cough Adverse Effects: GI upset, Drowsiness, consitpation

First choice for status epilepticus

Diazepam (valium)

Other antidyrhythmic drugs:

Digoxin, Adenosine

Mydriasis

Dilation of the pupil of the eye caused by contraction of the dilator muscle of the iris.

A pt has been taking alosteron (Lotonex) for 3 weeks for IBS, today she reports constipation. What is the priority action?

Discontinue the drug immediately.

Alzheimer's disease

Disease of the brain that is characterized by progressive mental deterioration manifested by confusion, disorientation, and loss of memory, ability to calculate, and visual-spatial orientation.

What actions is appropriate with the administration of acetylcysteine as part of treatment for an acetaminophen overdose?

Disguise the flavor with soda or flavored water

Local Anesthesia

Does not involve paralysis of respiratory function but does involve eliminations of pain sensation in the tissues innervated by anesthetized nerves.

Contraindications fo anorexiants

Drug allergy Any severe cardiovascular disease uncontrolled HTN Hyperthyroidism glaucoma mental agitation history of drug abuse eating disorders use of MAOIs w/i previous 14 days. Orlistat is contraindicated in cases of chronic malbsorption syndrome (e.g. Crohn's disease, colitis, short bowel syndrome) or cholestasis.

Anesthetics

Drugs that depress the CNS

Sedatives

Drugs that have an inhibitory effect on the CNS to the degree that they reduce nervousness, excitability, and irritability w/o causing sleep.

Parasympathomimetics

Drugs that mimic the parasympathetic nervous system; also referred to as cholinergic agonist drugs.

Stress Effects of Glucocorticoids

During stressful situations (e.g. injury, major surgery), corticosteroids are suddenly released or are necessary to help maintain homeostasis. Sudden release is believed to be a protective mechanism. Hypotension and shock may occur w/o steroid admin. During stress, epinephrine and norepinephrine are released from the adrenal medulla, and these catecholamines have a synergistic action with the corticosteroids.

relative refractory period

During the remainder of phase 3 and until the return to the RMP (phase 4), the cardiac cell can be depolarized again if it receives a powerful enough impulse (i.e. drugs, pacemaker).

lithium

EARLY signs of toxicity include: -nausea and vomiting -slurred speech -muscle weakness

The antidote for PENICILLIN?

EPINEPHRINE

Adverse Effects of Opioids

Effects include central nervous system depression, respiratory depression, hypotension, flushing, palpitations, constipation, nausea and vomiting, urinary retention, itching, rash, and biliary tract spasm.

ACE Inhibitors Adverse Effects

Effects may include fatigue; dizziness; mood changes; headache; dry, nonproductive cough; first-dose hypotension; loss of taste; proteinuria; hyperkalemia; rash; pruritus; anemia; neutropenia; thrombocytosis; and agranulocytosis.

A type of laxative that softens the stool

Emollient

Carbonic Anhydrase Inhibitors (CAIs)

Enzyme inhibiting Diuretic: Acetazolamide, for glaucoma

Acetylcholinesterase

Enzyme responsible for the breakdown of acetylcholine (also referred to as cholinesterase).

Primary epilepsy

Epilepsy in which there is no identifiable cause. Also known as idiopathic.

Adverse effects of muscle relaxants

Euphoria lightheadedness dizziness drowsiness fatigue confusion muscle weakness All experienced early in treatment. Less common: diarrhea GI upset headache slurred speech muscle stiffness constipation sexual difficulties in males hyptension tachycardia weight gain

The antidote for BENZODIAZEPINE is

FLUMAZENIL

Cyclobenzaprine (Flexeril) increases the patients risk for

Falls: r/t altered sensorium

Cardiovascular indications for adrenergics

Final group of adrenergic agents is used to support the cardiovascular system during cardiac failure or shock. These drugs are referred to as vasoactive sympathomimetics, vasoconstrictive drugs (vasopressive drugs, pressor drugs, and pressors), inotropes, or cardioselective sympathomimetics.

Mechanism/Therapeutic Effects Hydantoins (Antiepileptic)

First, they increase the threshold of activity in the area of the brain called the motor cortex, making it more difficult for a nerve to be excited. Second, they act to depress or limit the spread of a seizure discharge from its origin. Third, they can decrease the speed of nerve impulse conduction within a given neuron. The major therapeutic indication for AEDs is the prevention or control of seizure activity.

Effects of antiepileptics

First, they increase the threshold of activity in the area of the brain called the motor cortex; in other words, they make it more difficult for a nerve to be excited, or reduce the nerve's response to incoming electrical or chemical stimulation. Second, they act to limit the spread of a seizure discharge from its origin by suppressing the transmission of impulses from one nerve to the next. Third, they can decrease the speed of nerve impulse conduction within a given neuron. Some drugs may indirectly affect seizure foci (locations) in the brain by altering blood supply to these areas. Some drugs work by enhancing the effects of the inhibitory neurotransmitter GABA, which plays a role in regulating neuron excitability. Low GABA levels are associated with seizures. OVERALL, antiepileptics stabilize neurons and keep them from becoming hyper excited and generating excessive nerve impulses to adjacent neurons.

Antidote for Benzodiazepine

Flumazenil

Opioid Analgesic Assessment

Focus assessment on vitals; allergies, resp disorders, reps function, presence of head injury, neuro status w/ attention to level of consciousness or alertness and the level of sedation, sensory and motor function, GI tract functioning.

A pt with migraines is being evaluated. One potential tx is ergotamine tabs. What medical condition is a contraindication?

HTN Glaucoma Coronary Heart Disease Peripheral Vascular Disease

Darbepoetin (Arnesp), a erythropoiesis stimulating agent, is contraindicated when?

HTN, Hbg >10

Glucocorticoids in carb and protein metabolism

Help to maintain BG levels and glycogen content of liver and muscle. Fcilitate protein breakdown in muscle and extra hepatic tissues that least to increased plasma levels of amino acids. Can produce hyperglycemia and glycosuria. Effects can aggravate diabetes, bring on latent diabetes, and cause insulin resistance. Inhibition of protein synthesis can delay wound healing and cause muscle wasting and osteoporosis. May inhibit growth in young persons.

Drugs known as H2 Blockers that reduce acid secretion in the stomach

Histamine type 2 receptor antagonists: Cimetidine (tagamet) Ranitidine (Zantac) and Famotidine (Pepcid)

Vasodilators

Hydralazine, diazoxide, and nitroprusside *first line for hypertensive emergency*

SSRI Contraindications

Hypersensitivity, renal dysfunction, hepatic dysfunction, and pregnancy and lactation

Adverse affect of Venofer (Iron sucrose)

Hypotension

During a procedure, the nurse is monitoring a patient who has received Dexmedetomidine (Precedex) for moderate sedation. the nurse will observe for which potential adverse effect?

Hypotension Bradycardia Nausea

the nurse is caring for a client in the ICU. hemodynamic monitoring is accomplished via a Swan-ganz catheter. The nurse is aware that this type of monitoring will provide which of the following information?

INDIRECTLY MEASURES THE PRESSURE IN THE VENTRICLES

Correct administration of Adenosine (adenocard)

IV 6mg bolus over 1-2 sec; second rapid bolus of 12 mg as needed *very fast*

Toxicity and management of alpha blocker OD

In acute oral OD, activated charcoal is administered to bind w/ drug and remove it from the stomach and the circulation. W/ OD of both oral and injectable forms, symptomatic and supportive measures are to be instituted as needed. BP support w/ admin of fluids, volume expanders, and pressers and the admin of anticonvulsants such as diazepam for the control of seizures are examples of such measures.

Adjunct Anesthetics

Include: Neuromuscular Blocking Drugs (NMBDs) Sedative-hypnotics or anxiolytics Benzodiazepines Barbiturates Opioid analgesics anticholinergics antiemetics

Positive chronotropic effects

Increase in heart rate

Positive dromotropic effects

Increase in the conduction of cardiac electrical impulses through the atrioventricular node, which results in the transfer of nerve action potential from the atria to the ventricles. Ultimately leads to a systolic heartbeat (ventricular contractions).

Interactions with TCAs

Increased anticholinergic effects are seen when TCAs are taken with anticholinergics and phenothiazines. When MAOIs are taken with TCAs, the result may be increased therapeutic and toxic effects, including hyper pyretic crisis (excessive fever).

SSRI Drug Interactions

Increased drug effects occur when SSRIs are given with other strongly protein-bound drugs (warfarin and phenytoin). SSRIs prolong the actions of drugs metabolized by the cytochrome P-450 system (carbamazepine). There is increased risk for toxicity with monoamine oxidase inhibitors and tricyclic antidepressants.

ACE Inhibitors Drug Interactions

Increased effects occur with other antihypertensive drugs and diuretics. Decreased effects occur with aspirin and NSAIDs. Risk for hyperkalemia occurs if given with potassium supplements or potassium-sparing diuretics. Risk for lithium toxicity occurs if administered concurrently with lithium.

Midazolam (Versed)

Indications: Most commonly used preop and for moderate sedation. Useful for this due to its ability to cause amnesia and anxiolysis as well as sedation. Normally given by injection to adults, however a liquid oral dosage form is also available for children.

Diazepam (Valium)

Indications: anxiety, procedural sedation, anesthesia adjunct, anticonvulsant therapy, and skeletal muscle relaxation following orthopedic injury or surgery. Available in oral, rectal, and injectable forms.

Inotropic Drugs

Influence the force of muscular contractions, particularly contraction of the heart muscle.

Fosphenytoin (Cerebyx)

Injectable prodrug of phenytoin that was developed in an attempt to overcome some of the chemical disadvantages of phenytoin injection. Can be given IM or IV, by IV push or continuous infusion, w/o causing burning on injection. Given at a rate of 150 mg PE/min or less to avoid hypotension or cardiorespiratory depression.

Temazepam (Restoril)

Intermediate-acting benzo, normally induces sleep within 20-40 minutes. Long onset of action, so it is recommended that patients take it about 1 hour prior to going to bed.

Indications for topical nasal decongestants

Intranasal application of certain adrenergics can cause the constriction of dilated arterioles and a reduction in nasal blood flow, which thus decreases congestion. Work by stimulating alpha1-adrenergic receptors and have little or no effect on beta-adrenergic receptors.

General Anesthesia

Involves complete loss of consciousness, loss of body reflexes, elimination of pain and other sensations throughout the entire body, and skeletal and moth muscle paralysis, including respiratory muscles.

MAOIS

Isocarboxazid (Marplan) *do not take with SSRIs May cause hypertensive crisis if people eat foods containing tyramine

Ciprofloxacin (Cipro) is a quinolone, which acts by inhibiting DNA gyrase. Which of the following statements about ciprofloxacin is true?

It should not be used in children. Ciprofloxacin is teratogenic and is not approved for pregnant women or children. Antacids and other agents that contain divalent minerals can prevent ciprofloxacin from being absorbed. First-generation quinolones have little efficacy against the Gram-positive organisms that cause community-acquired pneumonia.

Contraindications to the use of anesthetic drugs

Known allergy Depending on the drug: Pregnancy Narrow-angle glaucoma Known susceptibility to malignant hyperthermia

Beta-adrenergic receptors

Located on postsynaptic cells that are stimulated by specific autonomic nerve fibers. Beta1-adrenergic receptors are located primarily in the heart, whereas beta2-adrenergic receptors are located in the smooth muscle fibers of the bronchioles, arterioles, and visceral organs.

What type of diuretic is useful when rapid diuresis is desired?

Loop (furosemide)

tranylcypromine

MAOI do NOT drink wine, beer

phenelzine sulfate

MAOI avoid age cheese leads to HTN crisis

Treatment resistant depression may recieve

MAOIs or methylphenidate

Anticoagulant therapy is appropriate for which conditions

MI, unstable angina, atrial fibrillation, mechanical heart valve, surgery or prolonged immobilization

The client with chronic renal failure tells the nurse he takes magnesium hydroxide (milk of magnesium) at home for constipation. The nurse suggests that the client switch to psyllium hydrophilic mucilloid (Metamucil) because:

MOM can cause magnesium toxicity Magnesium is normally excreted by the kidneys. When the kidneys fail, magnesium can accumulate and cause severe neurologic problems.

Treatment for Septic Shock

May consist of: - Volume replacement - Antibiotics - Surgery (if abscessed or necrotic) - Vasoconstricting agents (dopamine, norepiniphrine, or levarterenol) - Diuretics - Glucocorticoids (steroid) (somewhat controversial)

Toxicity and Management of OD of benzodiazepines

May result in one or all of the following: Somnolence Confusion Coma Diminished reflexes Flumazenil, a benzo antidote, can be used to acutely reverse the sedative effects of benzodiazepines by antagonizing the action of benzodiazepines on the CNS by directly competing with the benzodiazepine for binding at the receptors.

Third-generation cephalosporins are similar to first-generation cephalosporins with regard to:

Mechanism of antimicrobial action All cephalosporins, regardless of generation, are bacteriocidal, because they alter the bacterial peptidoglycan layer (cell wall). Efficacy against Gram-positive bacteria and gram-negative bacteria varies among third-generation cephalosporins. Third-generation, but not first-generation, cephalosporins have good penetration into the CNS.

Class I antidysrhythmic drugs

Membrane stabilizaing (sodium channel blocker) quinidine, procainaminie, phenytoin, lidocaine

Meperidine-like drugs

Meperidine, fentanyl, remifentanil, sufentanil, alfentanil

Dopamine

Metabolic precursor of norepinephrine, which is also a neurotransmitter in the sympathetic nervous system.

Methadone-like drugs

Methadone

Feverfew is used for

Migraine Headache

Morphine-like drugs

Morphine, heroin, hydromorphone, oxymorphone, levorphanol, codeine, hydrocodone, oxycodone

Strong Agonists

Morphine, hydromorphone, levorphanol, oxycodone, oxymorphone, meperidine, fentanyl, and methadone. Meperidine is not recommended for long-term use bc of the accumulation of a neurotoxic metabolite, normeperidine.

Adverse effects of benzodiazepines in psychotherapeutic treatment

Most common adverse effects is over expression of their therapeutic effects, in particular CNS depression. Can also cause hypotension, and paradoxical reactions, including hyperactivity and aggressive behavior (relatively uncommon). Rebound disinhibition can occur in elderly patients upon tapering of doses or dc of benzodiazepines. Al benzos are potentially habit forming and addictive.

Receptors to which opioids bind to relieve pain

Mu Kappa Delta - Mu is the most important

Guaifenesin (Mucinex)

Mucolytic

Toxicity and Management of Overdose

Naloxone and naltrexone are opioid antagonists, and they bind to and occupy all of the receptor sites (mu, kappa, delta). They are competitive antagonists with a strong affinity for these binding sites. - Reverse respiratory depression. - Used in the management of both opioid OD and opioid addiction. - Effects of naloxone are short lived and usually last about 1 hour. W/ long-acting opioids, respiratory depressant effects may reappear, and naloxone may need to be redosed.

One common use of adjuvant drugs is in the treatment of...?

Neuropathic pain. Opioids are not completely effective in such cases.

What are the 3 classes of meds used to treat angina

Nitrates, Beta Blockers, and CCBs

An 18 year old basketball player fell and twisted his ankle during a game. Which type of analgesic is he likely to receive?

Non opioid analgesic such as indomethacin

Adjuvant Drug Therapy

Nonspecific immunostimulant that enhances overall immune function, rather than stimulating the function of a specific immune system cell or cytokine through specific chemical reactions. Assist primary drugs in relieving pain. This approach allows the use of smaller dosages of opioids and reduces some of the adverse effects that are seen with higher dosages of opioids such as: Respiratory depression Constipation Urinary retention

Atomoxetine

Nonstimulant drug that is also used to treat ADHD. Noepinephrine reuptake inhibitor Associated w/ low incidence of insomnia and has low abuse potential.

Mechanism of action of antiepileptics

Not known with certainty, evidence indicates that they alter the movement of sodium, potassium, calcium, and magnesium ions. These changes in the movement of ions result in more stabilized and less excitable cell membranes.

Interactions with barbiturates

Notorious enzyme inducers. Stimulate action of enzymes in the liver that are responsible for the metabolism or breakdown of may drugs. By stimulating action of these enzymes, they cause many drugs to be metabolized more quickly, which usually shortens their duration of action. Additive CNS depression occurs with the coadministration of barbiturates with alcohol, antihistamines, benzodiazepines, opioids, and tranquilizers. Drugs most likely to have marked interactions w/ barbiturates include: MAOIs tricyclic antidepressants anticoagulants glucocorticoids oral contraceptives

Toxicity and management of barbiturate overdose

OD of barbiturates produces CNS depression ranging from sleep to profound coma and death. Respiratory depression progresses to Cheyne-Stokes respirations, hypoventilation, and cyanosis. Patients often have cold clammy skin or are hypothermic, and later can exhibit fever, areflexia, tachycardia, and hypotension. Treatment is mainly symptomatic and supportive. Mainstays of therapy are maintenance of an adequate airway, assisted ventilation, and oxygen administration if needed, along with fluid and pressor support as indicated. Highly metabolized by the liver, so in an OD, the amount of barbiturate may overwhelm the liver's ability to metabolize it. Activated charcoal

The physician has ordered orlistat (Xenical). What condition is this used to treat?

Obesity

Indications for anorexiant use

Obesity; however effects are often minimal w/o accompanying behavior modifications involving diet and exercise. Most commonly used in higher risk patients, which include obese patients with a BMI of 30 or higher, or patients with a BMI of 27 who are also hypertensive or have high cholesterol or diabetes.

Febrile Seizures

Occur in children 6 months to 5 years of age, and by definition are caused by fever. Antipyretic drugs are normally adequate for acute treatment.

What is a secondary use of anticonvulsants

Peripheral Neuropathy pain

Beta Blockers

Peripherally acting propanolol, metoprolol, atenolol

Antidepressant drugs

Pharmacologic treatment of choice for major depressive disorders. Very effective in treating depression, also: dysthymia (chronic low-grade depression) schoziphrenia (adjunctive) eating disorders personality disorders Some antidepressants are also used in the treatment of: migraines chronic pain syndromes sleep disorders premenstrual syndrome hot flashes associated w/ menopause

A barbiturate used primarily to control tonic-clonic and partial seizures

Phenobarbital

Generic name of a first line antiepileptic drug which can cause gingival hyperplasia with long term use

Phenytoin

A pt that has been taking a benzodiazepine for 5 weeks has been instructed to stop the medication. Which instruction will the nurse provide?

Plan a gradual reduction in dosage. Abrupt cessation may cause rebound insomnia

Adverse effects of antipsychotics

Possible hematologic effects include agranulocytosis (lack of granulocytes in the blood) and hemolytic anemia. CNS effects include drowsiness, neuroleptic malignant syndrome (potentially life-threatening adverse effect that may include high fever, unstable BP, and myoglobinemia), extrapyramidal symptoms, and tar dive dyskinesia. Extrapyramidal symptoms are involuntary motor symptoms similar to those associated w/ Parkinson's disease. This durg-induced state is known as pseudoparkinsonism and is characterized by symptoms such as akathisia and acute dyskinesia. Cardiovascular effects include postural hypotension, ECG changes, notably prolonged QT interval. Phenothiazines and haloperidol can also augment prolactin release, which can result in swelling of the breasts and milk secretion in worn taking these drugs. Gynecomastia can also occur in male patients. Adverse endocrine effects include insulin resistance, weight gain, changes in serum lipid levels, metabolic syndrome.

Glucocorticoids maintain normal BP

Potentiate vasoconstrictor action of norepinephrine. When glucocorticoids are absent, vasoconstriction action of catecholamines is diminished, and BP falls

Adverse effects of BBs

Primarily extensions of their pharmacologic activity. Sudden withdrawal may exacerbate: underlying angina precipitate an MI cause rebound HTN Also delay the recovery from hypoglycemia in patients with DMI Nonselective BBs can interfere with normal responses to hypoglycemia, such as tremor, tachycardia, and nervousness, in essence masking the S/S of hypoglycemia.

Amphetamines and related stimulants

Prinicple drugs used to treat ADHD and narcolepsy are amphetamines and non amphetamine stimulants. Atomoxetine, a non stimulant drug, is also used for ADHD.

Another word for intestinal flora modifier

Probiotic

Lisdexamfetamine (Vyanse)

Prodrug for dextroamphetamine, meaning it is converted int he body to dextroamphetamine.

Parietal Cells

Produce HCL

What is the antidote for heparin

Protamine sulfate IV

A client is receiving heparin sodium (Hepalean) therapy for a pulmonary embolism. Which antidote does the nurse confirm is available on the unit? 1. Vitamin K 2. Protamine sulfate 3. Antihemophilic factor 4.Aminocaproic acid

Protamine sulfate. Protamine sulfate is the antidote for heparin. Vitamin K is the antidote for warfarin. Antihemophilic factor and aminocaproic acid are both antidotes for fibrinolytic therapy that attempts to break up established clots

Phenytoin interacts with

Proton Pump inhibitors Anti coagulants Benzodiazepans Amioderone Sulfonamide antibiotics SSRIs

Indications for the treatment of Antipsychotics

Psychotic illness, most commonly schizophrenia. anxiety mood disorders Certain antipsychotics are used as antiemetics. They block serotonin receptors and dopamine receptors in the chemoreceptor trigger zone in the brain and inhibit neurotransmission in the vagus nerve in the GI tract. Additional blocking of dopamine receptors in the brainstem reticular system also allows atypical drugs to have anxiolytic or anti anxiety effects.

Your patient has been hospitalized after a seizure in which she fell and hit her head on a coffee table. On admission, she was also found to have a UTI. Which antibiotic class is known to reduce seizure threshold and should not be used to treat the UTI in your patient?

Quinolones

Adverse Effects of Acetaminophen

Rash N/V blood disorders or dyscrasias (e.g. anemias) nephrotoxicities hepatotoxicity - can be reversed with acetylcysteine

182.) A client with angina pectoris is experiencing chest pain that radiates down the left arm. The nurse administers a sublingual nitroglycerin tablet to the client. The client's pain is unrelieved, and the nurse determines that the client needs another nitroglycerin tablet. Which of the following vital signs is most important for the nurse to check before administering the medication? 1. Temperature 2. Respirations 3. Blood pressure 4. Radial pulse rate

Rationale: Nitroglycerin acts directly on the smooth muscle of the blood vessels, causing relaxation and dilation. As a result, hypotension can occur. The nurse would check the client's blood pressure before administering the second nitroglycerin tablet. Although the respirations and apical pulse may be checked, these vital signs are not affected as a result of this medication. The temperature also is not associated with the administration of this medication.

134.) A nurse reinforces instructions to a client who is taking levothyroxine (Synthroid). The nurse tells the client to take the medication: 1. With food 2. At lunchtime 3. On an empty stomach 4. At bedtime with a snack

Rationale: Oral doses of levothyroxine (Synthroid) should be taken on an empty stomach to enhance absorption. Dosing should be done in the morning before breakfast. **Note that options 1, 2, and 4 are comparable or alike in that these options address administering the medication with food.**

206.) A client is receiving baclofen (Lioresal) for muscle spasms caused by a spinal cord injury. The nurse monitors the client, knowing that which of the following is a side effect of this medication? 1. Muscle pain 2. Hypertension 3. Slurred speech 4. Photosensitivity

Rationale: Side effects of baclofen include drowsiness, dizziness, weakness, and nausea. Occasional side effects include headache, paresthesia of the hands and feet, constipation or diarrhea, anorexia, hypotension, confusion, and nasal congestion. Paradoxical central nervous system excitement and restlessness can occur, along with slurred speech, tremor, dry mouth, nocturia, and impotence. **Option 3 is most closely associated with a neurological disorder**

Adrenergic receptors

Receptor sites for the sympathetic neurotransmitters norepinephrine and epinephrine.

Caffeine Citrate

Recommended for neonatal apnea, including cases not responsive to other methylxanthines such as theophylline.

Parasympatholytics

Reduce the activity of the parasympathetic nervous system; also called anticholinergics.

Equianalgesia

Refers to the ability to provide equivalent pain relief by calculating dosages of different drugs and/or routes of administration that provide comparable analgesia.

LETROZOLE is prescribed for postmenopausal client with advanced breast cancer. The nurse provides instructions to the client regarding the medication and tells the client that the frequent side effect is:

SKELETAL PAIN LETROZOLE is an aromatase inhibitor that is used to treat advanced breast cancer is postmenopausal women whose disease has progressed after anti estrogen therapy. The frequent s/e include nausea, headache, fatigue, constipation, vomiting, and dyspnea

What is the first line treatment for depression?

SSRIs

Treatment for anxiety disorders include

SSRIs and benzodiazipine (increased risk for falls in elderly)

Chief Cells

Secrete Pepsinogen (enzyme)

The nurse is administering phenobarbital (Luminal) and will monitor the patient for which adverse effect

Sedation

Potential Opioid Adverse Effects and Their Management (2 of 3)

Sedation and Mental Clouding: Any change in mental status should always be evaluated to ensure that causes other than drug-related CNS depression are ruled out. May be managed with a decrease in the dosage of opioids or change in drug used. Prescriber may also order various CNS stimulants. Respiratory Depression: Long-term opioid use is generally associated with tolerance to respiratory depression. For severe depression, opioid antagonists (naloxone) may be used to improve respiratory status and, if titrated in small amounts, respiratory depression may be reversed without analgesia reversal.

Intermediate acting barbiturates

Sedations and control of convulsive conditions

Tonic-clonic seizures

Seizures involving initial muscular contraction throughout the body (tonic phase), progressing to alternating contraction and relaxation (clonic phase).

Zolpidem (Sonata)

Short-acting nonbenzo hypnotic. Relatively short half-life and its lack of active metabolites contribute to a lower incidence of daytime sleepiness compared with benzo hypnotics. Ambien CR is a longer-acting form with two separate drug reservoirs. One releases zolpidem faster than the other to induce hypnosis more rapidly. The second reservoir also releases zolpidem but does so more slowly throughout the night to help maintain sleep. Sleep walking is a concern with this dosage form.

Zaleplon (Sonata)

Short-acting nonbenzodiazepine hypnotic. Has a very short half-life, so patients whose sleep difficulties include early awakenings can take a dose in the middle of the night as long as it is at least 4 hours before they must arise.

Balanced Anesthesia

Simultaneous useo f both general anesthetics and adjuncts.

Superficial Pain

Skin and mucous membranes

Bradykinesia

Slowness of movement; classic symptom of parkinson's.

A highly soluble antacid form with a quick onset but short duration of action

Sodium bicarbonate

Potassium Sparing diuretics

Spironolactone

Opioid Tolerance

State of adaptation in which exposure to a drug causes change in drug receptors that result in reduced drug effects over time. Can occur in as little as one week.

Adrenergic agonists

Stimulate and mimic the actions of the sympathetic nervous system. Also called SYMPATHOMIMETICS.

Pregabalin (Lyrica)

Structurally r/t the inhibitory neurotransmitter GABA. Does not bind to GABA receptors but rather to the alpha2-delta receptor sites, which affect calcium channels in CNS tissues. Mech of action still not fully understood. Indicated as adjunct therapy for partial seizures, though it is most commonly used for neuropathic pain and postherpetic neuralgia. Adverse drug reactions are primarily CNS related. No clinically significant drug interactions are listed.

Ramelteon (Rozerem)

Structurally similar to the hormone melatonin, which is believed to regulate circadian rhythms in the body. Works as an agonist at melatonin receptors in the CNS; technically not a CNS depressant. Primarily indicated for patients who have difficulty with sleep onset rather than sleep maintenance. Contraindicated in cases of severe liver dysfunction, and is best avoided in patients receiving fluvoxamine, fluconazole, or ketoconazole, all of which can impeded its metabolism. Rifampin can reduce efficacy by speeding its metabolism via the induction of hepatic enzymes.

Topiramate (Topomax)

Structurally unique drugs chemically related to fructose. Indicated as adjunct therapy for partial and secondarily generalized seizures, for generalized tonic-clonc seizures, and for drop attacks in Lennox-Gastaut syndrome. Mech of action unknown. Common adverse effects are primarily CNS related. Angle closure glaucoma can ale occur, and the patient must immediately report any visual changes. Common drug interactions involve chiefly other anti epileptic drugs and oral contraceptives.

Anticonvulsants

Substances or procedures that prevent or reduce the severity of epileptic or other convulsive seizures.

Catecholamines

Substances that can produce a sympathomimetic response. Either endogenous catecholamines (epinephrine, no epinephrine, and dopamine) or synthetic catecholamine drugs (dobutamine).

A pt has been taking antiepileptic drugs for a year. The nurse is reviewing his recent history and will monitor for which condition that may develop during this time?

Suidical Thoughts or behavior

Cinchonism

Symptoms: tinnitus, hearing loss, blurred vision, GI upset Adverse effect of class 1a antidystrrhythmic Quinidine

Narcolepsy

Syndrome characterized by sudden sleep attacks, cataplexy, sleep paralysis, and visual or auditory hallucinations at the onset of sleep.

Contraindications to corticosteroid use

Systemic infections (AIDS, HIV, chickenpox, measles, fungal infection, Strongyloides, TB) Ocular herpes DM may be exacerbated Reactivation of active or latent esophagitis, gastritis, or PUD may be masked, bleed or perf may occur w/o warning. Monitor carefully those clients for whom edema may be hazardous (cardiac disease, HF, HTN, renal function impairment) Myasthenic crisis my be induced if administered to client w/ myasthenia gravis. May aggravate acute psychosis. Use with caution in pregnant women. If administered IA, joint should not be infected, bleeding, fracture or have had recent surgery.

Mr. Jones, age 37, is HIV-positive, and ciprofloxacin (a quinolone) has been ordered for him. In addition to making sure he drinks 8 full glasses of water a day to prevent crystalluria, you are aware that you need to be observant for the possible development of:

TSS Quinolones can cause a toxic shock-like syndrome with a diffuse, nonpuritic rash, fever, and hypotension in patients who are HIV positive.

vitamin B6

TX tuberculosis

Patient Teaching for ACE Inhibitors

Take medication exactly as prescribed. Do not stop taking it abruptly. Do not use with potassium supplements or increased dietary intake of potassium. Move and change positions slowly to prevent postural hypotension. This medication may cause an irritating dry cough. Impaired taste may be an adverse effect that may last up to 2 to 3 months after the drug has been discontinued. Monitor blood pressure and understand which parameters to report.

The physician orders sucralfate 1 g PO bid for a client taking digoxin 0.25 mg daily. The client asks the nurse if both pills can be taken together at breakfast so that the client doesn't forget to take them. The nurse should advise the client to take which of the following actions?

Take the SUCRALFATE 1 hour before breakfast and the DIGOXIN 1 hour after breakfast. SUCRALFATE best results on empty stomach, medications should be separated by 2 hours for maximum absorption

Which antibiotic should be avoided in children because it causes permanent staining of the teeth?

Tetracycline

Dronabinol (Marinol)

Tetrahydrocannabionoid derived from marijuana

Adverse Effects of AEDs

The most common effects are lethargy, abnormal movements, mental confusion, and cognitive changes. These drugs can also cause bone marrow suppression, exfoliative dermatitis, lupus erythematosus, Stevens-Johnson syndrome, and neuropathies. At toxic levels, they can cause nystagmus, ataxia, dysarthria, and encephalopathy. Long-term therapy can cause gingival hyperplasia, acne, hirsutism, osteoporosis, and hypertrophy of subcutaneous facial tissue resulting in an appearance known as Dilantin facies.

Acetylcholine

The neurotransmitter responsible for transmission of nerve impulses to effector cells in the parasympathetic nervous system.

Balanced Anesthesia

The practice of using combinations of different drug classes rather than a single drug

Synaptic cleft

The space either between two adjacent nerve cell membranes or between a nerve cell membrane and an effector organ cell membrane (also called a synapse).

Why can't you give folic acid to treat anemia without determining the underlying cause?

The symptoms of pernicious anemia may be masked, delaying treatment

Toxicity and management of cholinergic drug OD

There is little systemic absorption of the topically administered drugs and therefore little systemic toxicity. -When administered locally in the eye, they can cause temporary ocular changes such as transient blurring and dimming of vision. -Systemic toxicity with topically applied cholinergics is seen most commonly wen longer-acting drugs are given repeatedly over a long period. This can result in overstimulation of the parasympathetic nervous system and all the attendant responses. -Treatment is generally symptomatic and supportive, and the admin of a reversal drug is rarely required. -Likelihood of toxicity is greater for cholinergics that are given orally or IV. The most severe consequence of an OD is a cholinergic crisis. Symptoms include: circulatory collapse hypotension bloody diarrhea shock cardiac arrest Early signs include: abd cramps salivation flushing of skin N/V orthostatic hypotension Can be reversed promptly by the admin of atropine. Severe cardiovascular reactions or bronchoconstriction may be alleviated by epinephrine. ***SLUDGE Salivation, Lacrimation, Urinary incontinence, Diarrhea, GI cramps, Emesis

SSRI Adverse Effects

They are generally well tolerated. The most common adverse effects include insomnia, weight gain, and sexual dysfunction. The most potentially serious adverse effect is the occurrence of serotonin syndrome (elevated or diminished blood pressure, palpitations, fever, confusion, mania, and [in the most severe cases] seizures and coma). Serotonin syndrome is usually self-limited with discontinuation of medication.

Deep Pain

Tissues below skin level

The nurse is reviewing a pts medication administration record. What is a common use for doxapram (Dopram)?

To treat drug-induced respiratory depression (COPD)

Toxicity and management of adrenergic OD

Toxic effects of adrenergic drugs are an extension of their common adverse effects (e.g. seizures from excessive CNS stimulation, hypotension or hypertension, dysrhythmias, palpitations, nervousness, dizziness, fatigue, malaise, insomnia, headache, tremor, dry mouth, and nausea). In the acute setting seizures can be effectively managed w/ diazepam. Intracranial bleeding can also occur, often as the result of an extreme elevation in blood pressure. Such elevated BP poses the risk of hemorrhage not only in the brain but elsewhere in the body as well. Best treatment is to lower the BP using a rapid-acting sympatholytic drug.

Other Opioids

Tramadol, tapentadol

Serotonin Antagonists and Reuptake inhibitors

Trazodone (Desyrel) for sedation

Indications for the use of aminoglutethimide

Treatment of Cushing's syndrome associated with adrenal carcinoma Ectopic adrenocorticotropic hormone tumors Adrenal gland hyperplasia

Indications for acetaminophen

Treatment of mild to moderate pain and fever. Appropriate substitute for aspirin bc of its analgesic and antipyretic properties. Antipyretic drug of choice in children and adolescents with flu syndromes, bc of the use of aspirin in these populations is associated with a condition known as Reye's syndrome.

Mechanism of action and effects of serotonin receptor agonists

Triptans stimulate 5-HT receptors in cerebral arteries, causing vasoconstriction and normally reducing or eliminating headache symptoms. Also reduce production of inflammatory neuropeptides.

Adverse effects of anorexiants

W/ exception of diethyproprion, anorexiants may raise BP and cause heart palpitations and even dysrhythmias at higher dosages. May also cause: anxiety agitation dizziness headache Most common adverse effects of orlistat include: headache URI GI distress, including fecal incontinence and oily stools.

Interactions with muscle relaxants

When coadministered with other depressant drugs such as alcohol and benzos, caution should be used to avoid OD. Mental confusion, anxiety, tremors, and additive hypoglycemic activity have also been reported.

Adverse Effects of amphetamine and non amphetamine stimulants

Wide range of effects, most often arise when administered at high doses. These drugs tend to speed up body systems. For example, effects on the cardiovascular system include angina, anxiety, insomnia, headache, tremor, blurred vision, increased metabolic rate, GI distress, dry mouth, mania, psychoses, or aggression. Common adverse effects associated with atomoxetine include: he ache, abdominal pain, vomiting, anorexia, and cough.

A client is prescribed metaproterenol (Alupent) via a metered-dose inhaler, two puffs every 4 hours. The nurse instructs the client to report adverse effects. Which of the following are potential adverse effects of metaproterenol? a) Irregular heartbeat. b) Constipation. c) Pedal edema. d) Decreased pulse rate.

a) Irregular heartbeat. Metaproterenol (Alupent) may cause irregular heartbeat, tachycardia, or anginal pain because of its adrenergic effect on beta-adrenergic receptors in the heart. It is not recommended for use in clients with known cardiac disorders. Metaproterenol does not cause constipation, pedal edema, or bradycardia. bronchodilator

D5NS, steroid in high dose is given to tx

addisonian crisis

When a second drug is given with a primary analgesic to enhance the analgesic effect it is being used as an

adjuvant

hospital for IV antibiotic tx of an infection. the client has a known hx of ALCOHOL ABUSE, on the evening of the second day, the client becomes CONFUSED, AGITATED and TACHYCARDIC. which of the following is an appropriate action by the nurse ?

administer CHLORDIAZEPOXIDE (librium) as prescribed it is often used for clients who are withdrawing from alcohol b/c it reduces symptoms and can help prevent seizure and delirium tremens.

A client receives MIDAZOLAM, 2 mg I.V., as sedation before bronchoscopy. Five minutes after he receives the drug, his respiratory rate drops to 4 breaths/minute. What is the most appropriate action by the nurse? 1. administer naloxone 2. administer protamine sulfate 3. administer phentolamine 4 administer flumazenil

administer FLUMAZENIL reverse the effects of benzodiazepines

Common bronchodilators classified as predominantly beta2-selective adrenergics

albuterol ephedrine formoterol levalbuterol metaproterenol pirbuterol salmeterol terbutaline

A patient taking phenobarbital must not ingest

alcohol

A patient will be receiving the barbituarate phenobarbital as a part of treatment for seizures. the nurse assesses the pts current list of medications, which are known to cause interactions?

alcohol antihistamines benzodiazepines, opioids tranquilizers anticoagulants MAOIs TCAs Oral Contraceptives Glucocorticoids

Contraindications to BB use

allergies uncompensated HF cardiogenic shock heart block or bradycardia prenancy severe pulmonary disease Raynaud's

Contraindications to the use of cholinergic drugs

allergy GI or genitourinary (GU) tract obstruction bradycardia defects in cardiac impulse conduction hyperthyroidism epilepsy hypotension COPD Parkinson's disease is listed as a precaution to these drugs; however, rivastigmine is used in patients w/ Parkinson's disease who also have dementia.

Contraindications to anticholinergic use

allergy angle-closure glaucoma acute asthma or other resp distress myasthenia gravis acute cardiovascular instability GI or GU tract obstruction or other acute GI/GU illness

Contraindications for the use of Antipsychotics

allergy comatose state significant CNS depression brain damage liver or kidney disease blood dyscrasias uncontrolled epilepsy

Contraindications to alpha-blocking drugs

allergy peripheral vascular disease hepatic and renal disease coronary artery disease peptic ulcer sepsis

Contraindications to adrenergic use

allergy severe hypertension

Ultrashort acting barbiturates

anesthesia for short surgical procedures, anesthesia induction, control of convulsions, and reduction of intracranial pressure in neurosurgical patients.

lidocaine

anesthetic, anti-arrhythmics; tx ventricular tachycardia

epoetin

anti-anemics

chlordiazepoxide

anti-anxiety sedative/hypnotics

phenytoin

anti-arrhythmic anti-convulsant, at risk for impaired liver function

ethosuximide

anti-convulsant s/e nausea

amitriptyline

anti-depressant do NOT drink alcohol

hydralazine hydrocholirde

anti-hypertensive arm when client has autonomic dysreflexia

doxycycline

anti-infective

trifluoperazine

anti-psychotic do NOT breast feed

impaired hearing results from

anti-tubercular drugs

nitroglycerin patch

antianginal

fluphenazine (prolixin)

antipsychotic given to schizophrenics

risperidone

antipsychotic, mood stabilizers

acyclovir

antiviral tx herpes take with food

Negative Symptoms of Schizophrenia

apathy social withdrawal blunted affect poverty of speech catatonia

aspirin

assess for tinnitus =toxicity

the nurse devises a teaching plan for the client with aplastic anemia. Which of the following is the most important concept to teach for health promotion and maintenance ?

avoid exposure to others with acute infections. clients with aplastic anemia are severely immunocompromised and at risk for infection and possible death r/t bone marrow suppression and pancytopenia.

triazolam overdose benzodiazepine ending lam or pam

benzodiazepine antidote: FLUMAZENIL

atenolol and captopril contraindicated

betablockers and ace inhibitors are contraindicated with african americans

alendronate contraindicated with

bone resorption inhibitors, contraindicated ECT, sit for 30 minutes when taking alendronate lead to irritation and ulcers

the nurse cares for a client receiving QUINIDINE . it is most important for the nurse to include which information in the client's teaching plan?

call your health care provider if you have ringing in your ears. quinidine (antiarrhythmic) causes tinnitus which can lead to permanent hearing loss.

Patients taking sulfonamides should be advised to drink a lot of liquids because sulfonamides:

can crystallize in concentrated urine, causing renal damage.

Ace Inhibitors

captopril, enalapril, monopril, lisonopril, ramipril *First line* Used for HTN and HF: prevents sodium reabsorbtion and decreases vascular resistance Used in diabetes to reduce GFR and prevent progression of neuropathy Interacts with NSAIDS :( Beware of hyperkalemia Dry cough

Beta Blockers are used with caution in diabetics due to

causing hyper/hypoglycemia

Myoclonic seizures

characterized by brief muscular jerks, but not as extreme as in other subtypes.

magnesium sulfate

check deep tendon reflex

The nurse is explaining the proper use of IPECAC to a group of parents. Which of the following accidental poisonings is the treatment appropriate? A. oven cleaner B. drain cleaner C. kerosene d. chewable vitamins

chewable vitamins POISONING WITH VITAMINS IS THE ONLY CASE IN WHICH IT IS SAFE TO INDUCE VOMITING WITH IPECAC. THE OTHER 3 CHOICES ARE TOO CORROSIVE WHICH WILL DAMAGE YOUR ESOPHAGUS, THROAT, ETC.

Drugs that stimulate the parasympathetic nervous system

cholinergic drugs, cholinergic agonists, and parasympathomimetics.

bethanechol

cholinergic; tx urinary retention

Anticholinerics, Antihistamins, and benzodiazepines can all cause

confusion

propanolol

contraindicated with asthma can cause bronchospasm

pancrealipase

contraindicated with milk

indomethacin (NSAID)

contraindicated with peptic ulcer disease

hyrdrocortisone

corticosteroid; causes HTN, tx addisonian crisis

a pt is receiving TOBRAMYCIN for lower respiratory tract infection. the nurse should monitor for potential side effects, paying close attention to

creatinine the most common s/e of tobramycin are nephrotoxicity , neurotoxicity, and ototoxicity

function of morphine sulfate

decreases blood return to the right side of the heart and decreases peripheral resistance

Side effects of Anti-parkinson drugs

delirium, orthostatic hypotension

the nurse is preparing to begin DOPAMINE infusion on a client before beginning the infusion the nurse should

determine the patency of the iv line extravasation occurs, there is sloughing of the surrounding skin and tissue; patent IV line is essential to prevent serious side effects

Thiazide diuretics interact with

digitalis, hypoglycemics, and corticosteroids

Common vasoactive adrenergics

dobutamine dopamine ephedrine epinephrine fenoldopam midodrine norepinephrine phenylephrine

Milrinone (Primacor) is used in HF, but can cause

dysrhythmias

Glucocorticoids often administered for their anti-inflammatory activity

e.g. control of asthma, inflammation, swelling, and edema. - Dexamethasone (Decadron) represents an agent with almost pure glucocorticoid activity is used to treat a number of inflammatory states. - Most corticosteroids used in clinical practice (prednisone, prednisolone, hydrocortisone) possess both glucocorticoid and mineralocoritcoid effects, and result in a wide array of short and long-term adverse effects.

furosemide

eat cantaloupe contains potassium

Nasal decongestant adrenergics

ephedrine naphazoline oxymetazoline phenylephrine tetrahydrozoline

Long acting barbiturate

epileptic seizure prophylaxis

Ophthalmic adrenergics

epinephrine naphazoline phenylephrine tetrahydrozoline

PROPANOLOL is also used for

essential tremors given for akinesia

diabetics should NOT have

garlic leads to hypoglycemia

digoxin toxicity

greater than 2 ng /ml

Positive Symptoms of Schizophrenia

hallucinations delusions conceptual disorganizations

propanolol is CONTRAINDICATED

heart failure and pulmonary edema

a nurse is reviewing the physician's orders for a client with systemic lupus erythematosus (SLE). The nurse determines that the medication most appropriate for the treatment plan is:

hydroxychloroquine is used in tx of SLE to prevent inflammation

sodium polystyrene sulfonate

hypokalemic, kayexalate contraindicated with orange juice

non promptly treated status epilepticus

hypotension hypoxia brain damage death Drugs used to treat: diazepam fosphenytion lorasepam phenobarbital phenytoin

ginkgo contraindicated with

ibuprofen increases RBC for bleeding

when caring for a client with diabetes who is schedule for a radiographic study requiring contrast, the nurse should expect the physician :

implement ACETYLCYSTEINE therapy

fludorcortisone

increase dose if pt loses too much weight. this med is suppose to retain na+ and h20

Action of Amphetamines and phenidates

increase the effects of norepinephrine and dopamine in CNS synapses by increasing their release and blocking their reuptake. As result, both neurotransmitters are in contact with their receptors longer, which lengthens their duration of action.

Rifampin is known to stimulate the hepatic metabolism of many other drugs. This is called enzyme

induction

Ezetimibe (Zetia) works by

inhibiting absorption of cholesterol in the small intestine *never give with statins

from which drug class should be adm. to the client with heart failure to MAXIMIZE cardiac performance?

inotropics

Adverse effects of 2nd gen antidepressants

insomnia weight gain sexual dysfunction (in SSRI use, primarily r/t inability to achieve orgasm). serotonin syndrome

PENTOXIFYLLINE (blood viscosity reducing agent and increases the flexibility of RBCs) is used to tx

intermittent claudication

Action potential duration

interval between phases 0 and 4 of action potential.

Absence seizures

involve a brief loss of awareness that commonly occurs with repetitive spasmodic eye blinking for up to 30 seconds. Primarily occurs in childhood and rarely after 14 years of age.

Atonic seizures (drop attacks)

involve sudden global muscle weakness and syncope.

The nurse is giving IV phenytoin (Dilantin). Which guidelines will the nurse follow for administration?

it is very irritating to veins and must be administered slowly, diluted in NS, using a filter needle followed by a flush of NS to avoid local venous irritation, it must be given IV push not continuously.

Drugs for adjunctive therapy for refractory partial onset seizures

lamotrigine, topiramate, gabapentin, oxcarbazepine, zonisamide, levetiracetam, and tiagabine.

psyllium

laxative it is the safest

succimer is the antidote for

lead toxicity

Antilipimic drugs are contraindicated in patients that have

liver disease

laxatives include

magnesium hydroxide bisacodyl

Which drug does the CDC now recommend for the treatment of Clostridium difficile infection?

metronidazole (Flagyl)

lithium carbonate

mood stabilizer tx bipolar; client must stay hydrated

antidepressant drugs chew on gum or hard candy to prevent

mouth dryness

HMG- CoA cause

muscle pain that can lead to rhabdomyelysis

carbidopa/ levodopa

neurological med, tx parkinson disease

Your patient is a sexually active 15-year-old girl who suffers from recurrent urinary tract infections. Which drug should NOT be given to this patient as prophylaxis for UTI?

norfloxacin Women should be informed that antibiotics can reduce the efficacy of contraceptive steroids. Norfloxacin and other quinolone antibiotics are teratogenic and should not be used by sexually active women who may become pregnant.

phencyclidine aka angel dust. client on this will show:

nystagmus

In giving patients an antibiotic, the nurse must:

observe the patient for 15 to 20 minutes after administration of the drug.

A superinfection is an infection that:

occurs when normal body flora are altered by an antibiotic.

Central Pain

occurs with trauma, tumor, inflammation, or disease affecting CNS tissues

percocet

opioid analgesic, given to hemophiliacs

butorphanol tartrate

opioid analgesics, assess respiratory depression

naloxone hydrochloride

opioid antagonist; works as a blocking drug for opioids, so it does not produce analgesia or resp. depression. Drug of choice for complete or partial reversal of resp. depression, also indicated in cases of acute opioid OD. Primary adverse effect is opioid withdrawal syndrome, which can occur with the abrupt overreversal in opioid-tolerant patients. Only available in injectable dosage forms. Contraindicated in hypersensitivity cases.

Aminoglycosides such as gentamycin may cause:

ototoxicity and nephrotoxicity

Rest and Digest

parasympathetic nervous system

A client on mechanical ventilator begins to fight the ventilator. Which med will be ordered for the client? A pavulon(neuromuscular blocking agent) B versed (benzodiazepine) C atarax(anti-anxiety) D sublimaze( opioid)

pavulon (neuromuscular blocking agent)

absolute or effective refractory period

period between phase 0 and midway through phase 3. Cardiac cell cannot be restimulated to depolarize and generate another action potential.

Drugs for partial onset seizures

phenobarbital, phenytoin, primidone, carbamazepine, and valproic acid are equally effective.

unsteady gait, stiff muscles, and EYES moving RAPILY SIDE TO SIDE UP AND DOWN =

phenycyclidine intoxication aka angel dust

Noncatecholamine adrenergic drugs

phenylephrine, metaproterenol, and albuterol. - Structurally dissimilar to the endogenous catecholamines and have a longer duration of action than either the endogenous or synthetic catecholamines; show similar patterns of activity.

scabies

private room

the nurse cares for a client receiving LITHIUM therapy. it is most important for the nurse to include which when instructing the client?

regular diet with normal sodium and adequate fluid intake.

orange-red discoloration of secretion occurs with

rifampin anti-tuberculars

Short acting barbiturates

sedation and control of convulsive conditions.

A hypnotic causes

sleep

Referred Paine

spinal cord

cutaneous anthrax which precaution do you follow?

standard precaution

dexamethasone

steroid suppress your immune system

Trimethoprim is given most commonly in combination with which other drug to increase the bacteriocidal potential?

sulfamethoxazole

Fight or Flight

sympathetic nervous system

pt is taking ALENDRONATE SODIUM (fosamax). when planning client teaching for this med. which should the nurse include ?

take ALENDRONATE SODIUM with a full glass of water on an empty stomach in an upright position is recommend to decrease GI upset

PHENYTOIN

take vitamin D include milk, cantaloupe, kale

metformin

take with meals

prednisone

take with meals

acetylsalicylic acid contraindicated

tartrazine if allergic

an anxious client has been ordered the following preoperative medications IM: MEPERIDINE, PROMETHAZINE, DIAZEPAM, which technique should be used in the administration of these medication?

the DIAZEPAM should be placed in a separate syringe diazepam is not compatible with most medications and should be administered separately.

Mabel G., age 61, started taking calcium carbonate (Tums) as a calcium supplement last year. Yesterday, enteric-coated erythromycin tablets were prescribed for the treatment of Mycoplasma pneumoniae. You tell her not to take Tums within 2 hours of the erythromycin because:

the Tums will dissolve the enteric coating on the erythromycin in the stomach, and she will experience gastric distress.

Optimal antihypertensive drugs for african americans are

thiazides and Calcium Chanel Blockers

the nurse is aware that antihypertensives should be cautiously in clients already taking which other drug?

thioridazine (antipsychotic)

levothyoxine

thyroid hormone, take in the AM to prevent insomnia

levothyroxine (t4)

thyroid hormone, take in the morning on an empty stomach

gentamicin sulfate

toxicity= hearing loss

when adm platelets do NOT use

transfusion set

Heparin is used for

treatment and prevention (hospital prophylaxis)

hydroxyzine

treats anxiety

carbamazepine

tx bipolar disorder, seizures, trigeminal neuralgia, diabetic neuropathy. contraindicated hormonal contraceptives

colchicine, allopurinol, and NSAID Tx

tx gout

pemoline (cylert)

used for ADHD

naloxone

used to reverse narcotic depression; expect nausea and vomiting, restlessness, abdominal cramping

a low serum POTASSIUM level increases the risk of

ventricular tachycardia encourage client to eat potassium-rich foods

albumin

volume expanders tx=ascites

Nursing Implications of AEDs

¬ Assess complete blood count, serum chemistry, and drug levels prior to administration. Know that therapeutic drug levels are usually 10 to 20 mcg/mL. ¬ Know that 150 mg of fosphenytoin yield 100 mg of phenytoin and that the concentration and infusion rate of fosphenytoin would be expressed as a phenytoin equivalent (PE). ¬ Administer phenytoin no faster than 50 mg/min and fosphenytoin at 150 mg PE/min to avoid hypotension or cardiorespiratory depression. ¬ Dilute phenytoin only with normal saline solution. Flush intravenous lines with saline before and after administration. ¬ Monitor the patient for ataxia and dizziness after an infusion. ¬ Do not confuse fosphenytoin (Cerebyx) with Celebrex. ¬ Administer oral dosage forms of the drug with food. Instruct the patient not to open, chew, or break capsules; however, the non-sustained-release tablets may be chewed.

Indications for use of drugs such as morphine, meperidine, hydromorphone, and oxycodone

Often used to control postop and other types of pain. IV morphine and hydromorphone are often first-line analgesics in the immediate postop setting.

REM Sleep

One of the stages of the sleep cycle. Some of the characteristics of REM sleep are rapid movement of the eyes, vivid dreams, and irregular breathing.

Parasympathetic Nervous System

-Branch of autonomic nervous system with functions opposite those of the sympathetic nervous system. -Acetylcholine -Cholinergic receptors -Nicotinic receptors -Muscarinic receptors

Adverse effects of beta-adrenergics

-CNS: mild tremors headache nervousness dizziness -cardiovascular: increased HR palpitations fluctuations in BP -Other: sweating N/V muscle cramps

Adverse effects of CCBs

-Cardiovascular: Hypotension, palpitations, tachy or brady -GI: Constipation, nausea -Other: Dyspnea, rash, flushing, peripheral edema

Which drugs are approved for treating obesity?

Only amphetamines approved: Benzphetamine Methamphetamine Orlistat is a related non stimulant drug used to treat obesity. Works locally in the small and large intestines inhibiting caloric intake from fatty foods.

Opioid Analgesics

- Originated from the opium poppy plant. - Very strong pain relievers. - Morphine and Codeine are the only useful pain-relieving drugs from poppy.

Cholinergic drugs (AKA cholinergic agonists, parasympathomimetics)

-Mimic the effects of acetylcholine -Can stimulate receptors either directly or indirectly

Indications for Indirect-acting cholinergic drugs

-Work by increasing acetylcholine concentrations at the receptor sites, which leads to stimulation of the effector cells. -Cause skeletal muscle contraction and are used for the diagnosis and treatment of myasthenia gravis. -Their ability to inhibit acetylcholinesterase also makes them useful for the reversal of neuromuscular blockade produced either by neuromuscular blocking drugs or by anticholinergic poisoning. For this reason, indirect-acting drug physostigmine is considered the antidote for anticholinergic poisoning as well as poisoning by irreversible cholinesterase inhibitors such as organophosphates and carbonates, common classes of insecticides. -Also used to treat Alzheimer's disease. Cholinergic drugs increase concentrations of acetylcholine in the brain by inhibiting cholinesterase; helps to enhance and maintain memory and learning capabilities. Donepezil, glantamine, rivastigmine. Although their therapeutic efficacy is often limited, these drugs may enhance a patient's mental status enough to cause a noticeable, if temporary, improvement in the quality of life for patients as well as caregivers and family members. The most commonly used of these meds at this time is donepezil. Patient response to these drugs is highly variable. Memantine is also used to treat Alzheimer's disease, but it is not a cholinesterase inhibitor.

179.) A nurse provides medication instructions to a client who had a kidney transplant about therapy with cyclosporine (Sandimmune). Which statement by the client indicates a need for further instruction? 1. "I need to obtain a yearly influenza vaccine." 2. "I need to have dental checkups every 3 months." 3. "I need to self-monitor my blood pressure at home." 4. "I need to call the health care provider (HCP) if my urine volume decreases or my urine becomes cloudy."

1. "I need to obtain a yearly influenza vaccine." Rationale: Cyclosporine is an immunosuppressant medication. Because of the medication's effects, the client should not receive any vaccinations without first consulting the HCP. The client should report decreased urine output or cloudy urine, which could indicate kidney rejection or infection, respectively. The client must be able to self-monitor blood pressure to check for the side effect of hypertension. The client needs meticulous oral care and dental cleaning every 3 months to help prevent gingival hyperplasia.

The nurse administers racemic epinephrine (Racepinephrine) to an 8 year old boy. Ten minutes after administration, The nurse should be alert for: 1. Respiratory distress 2. Profound tachcardia 3. Signs of improved oxygenation 4. Diminished cyanosis

1. Respiratory distress a rebound effect from racemic epinephrine can occur up to 4 hours after treatment with signs of respiratory distress. Tachycardia may initially follow treatment with racemic epinephrine as well as improvement in client status.

A patient has taken an overdose of ASPIRIN. Which of the following should a nurse most closely monitor for during acute management of this patient?

Onset of pulmonary edema Aspirin overdose can lead to metabolic acidosis and cause pulmonary edema development.

Hormonal effects of the antipsychotic medications include which of the following? 1. Retrograde ejaculation and gynecomastia 2. Dysmenorrhea and increased vaginal bleeding 3. Polydipsia and dysmenorrhea 4. Akinesia and dysphasia.

1. Retrograde ejaculation and gynecomastia decreased libido,retrograde ejaculation, gynecomastia are all Hormonal effects that can occur with antipsychotic medications. Reassure the client that the effects can be reversed or that changing medication may be possible. Polydipsia, akinesia, and dysphasia aren't hormonal effects.

202.) A nurse is collecting data from a client about medications being taken, and the client tells the nurse that he is taking herbal supplements for the treatment of varicose veins. The nurse understands that the client is most likely taking which of the following? 1. Bilberry 2. Ginseng 3. Feverfew 4. Evening primrose

1. Bilberry Rationale: Bilberry is an herbal supplement that has been used to treat varicose veins. This supplement has also been used to treat cataracts, retinopathy, diabetes mellitus, and peripheral vascular disease. Ginseng has been used to improve memory performance and decrease blood glucose levels in type 2 diabetes mellitus. Feverfew is used to prevent migraine headaches and to treat rheumatoid arthritis. Evening primrose is used to treat eczema and skin irritation.

156.) A nurse is reviewing the laboratory results for a client receiving tacrolimus (Prograf). Which laboratory result would indicate to the nurse that the client is experiencing an adverse effect of the medication? 1. Blood glucose of 200 mg/dL 2. Potassium level of 3.8 mEq/L 3. Platelet count of 300,000 cells/mm3 4. White blood cell count of 6000 cells/mm3

1. Blood glucose of 200 mg/dL Rationale: A blood glucose level of 200 mg/dL is elevated above the normal range of 70 to 110 mg/dL and suggests an adverse effect. Other adverse effects include neurotoxicity evidenced by headache, tremor, insomnia; gastrointestinal (GI) effects such as diarrhea, nausea, and vomiting; hypertension; and hyperkalemia.

Opioid Mechanism/Therapeutic Effects

Opioid analgesics bind to opioid receptors in the brain, blocking the transmission of pain messages and causing a reduction in pain sensation. They also cause euphoria and sedation. Full agonists cause the strongest analgesic effect but also have the strongest adverse reactions. Partial or mixed agonists are weaker analgesics but also cause fewer adverse effects.

32.) Desmopressin acetate (DDAVP) is prescribed for the treatment of diabetes insipidus. The nurse monitors the client after medication administration for which therapeutic response? 1. Decreased urinary output 2. Decreased blood pressure 3. Decreased peripheral edema 4. Decreased blood glucose level

1. Decreased urinary output Rationale: Desmopressin promotes renal conservation of water. The hormone carries out this action by acting on the collecting ducts of the kidney to increase their permeability to water, which results in increased water reabsorption. The therapeutic effect of this medication would be manifested by a decreased urine output. Options 2, 3, and 4 are unrelated to the effects of this medication.

105.) A nurse is collecting data from a client and the client's spouse reports that the client is taking donepezil hydrochloride (Aricept). Which disorder would the nurse suspect that this client may have based on the use of this medication? 1. Dementia 2. Schizophrenia 3. Seizure disorder 4. Obsessive-compulsive disorder

1. Dementia Rationale: Donepezil hydrochloride is a cholinergic agent used in the treatment of mild to moderate dementia of the Alzheimer type. It enhances cholinergic functions by increasing the concentration of acetylcholine. It slows the progression of Alzheimer's disease. Options 2, 3, and 4 are incorrect.

193.) Sodium hypochlorite (Dakin's solution) is prescribed for a client with a leg wound containing purulent drainage. The nurse is assisting in developing a plan of care for the client and includes which of the following in the plan? 1. Ensure that the solution is freshly prepared before use. 2. Soak a sterile dressing with solution and pack into the wound. 3. Allow the solution to remain in the wound following irrigation. 4. Apply the solution to the wound and on normal skin tissue surrounding the wound.

1. Ensure that the solution is freshly prepared before use. Rationale: Dakin solution is a chloride solution that is used for irrigating and cleaning necrotic or purulent wounds. It can be used for packing necrotic wounds. It cannot be used to pack purulent wounds because the solution is inactivated by copious pus. It should not come into contact with healing or normal tissue, and it should be rinsed off immediately if used for irrigation. Solutions are unstable and the nurse must ensure that the solution has been prepared fresh before use. **Eliminate options 2 and 3 first because they are comparable or alike. It makes sense to ensure that the solution is freshly prepared; therefore, select option 1**

112.) A hospitalized client is started on phenelzine sulfate (Nardil) for the treatment of depression. The nurse instructs the client to avoid consuming which foods while taking this medication? Select all that apply. 1. Figs 2. Yogurt 3. Crackers 4. Aged cheese 5 Tossed salad 6. Oatmeal cookies

1. Figs 2. Yogurt 4. Aged cheese Rationale: Phenelzine sulfate (Nardil) is a monoamine oxidase inhibitor(MAOI). The client should avoid taking in foods that are high in tyramine. Use of these foods could trigger a potentially fatal hypertensive crisis. Some foods to avoid include yogurt, aged cheeses, smoked or processed meats, red wines, and fruits such as avocados, raisins, and figs.

158.) A client with chronic renal failure is receiving epoetin alfa (Epogen, Procrit). Which laboratory result would indicate a therapeutic effect of the medication? 1. Hematocrit of 32% 2. Platelet count of 400,000 cells/mm3 3. White blood cell count of 6000 cells/mm3 4. Blood urea nitrogen (BUN) level of 15 mg/dL

1. Hematocrit of 32% Rationale: Epoetin alfa is used to reverse anemia associated with chronic renal failure. A therapeutic effect is seen when the hematocrit is between 30% and 33%. The laboratory tests noted in the other options are unrelated to the use of this medication.

167.) A nurse prepares to reinforce instructions to a client who is taking allopurinol (Zyloprim). The nurse plans to include which of the following in the instructions? 1. Instruct the client to drink 3000 mL of fluid per day. 2. Instruct the client to take the medication on an empty stomach. 3. Inform the client that the effect of the medication will occur immediately. 4. Instruct the client that, if swelling of the lips occurs, this is a normal expected response.

1. Instruct the client to drink 3000 mL of fluid per day. Rationale: Allopurinol (Zyloprim) is an antigout medication used to decrease uric acid levels. Clients taking allopurinol are encouraged to drink 3000 mL of fluid a day. A full therapeutic effect may take 1 week or longer. Allopurinol is to be given with or immediately following meals or milk to prevent gastrointestinal irritation. If the client develops a rash, irritation of the eyes, or swelling of the lips or mouth, he or she should contact the health care provider because this may indicate hypersensitivity.

12.) A nurse is caring for a client who is receiving an intravenous (IV) infusion of an antineoplastic medication. During the infusion, the client complains of pain at the insertion site. During an inspection of the site, the nurse notes redness and swelling and that the rate of infusion of the medication has slowed. The nurse should take which appropriate action? 1. Notify the registered nurse. 2. Administer pain medication to reduce the discomfort. 3. Apply ice and maintain the infusion rate, as prescribed. 4. Elevate the extremity of the IV site, and slow the infusion.

1. Notify the registered nurse. Rationale: When antineoplastic medications (Chemotheraputic Agents) are administered via IV, great care must be taken to prevent the medication from escaping into the tissues surrounding the injection site, because pain, tissue damage, and necrosis can result. The nurse monitors for signs of extravasation, such as redness or swelling at the insertion site and a decreased infusion rate. If extravasation occurs, the registered nurse needs to be notified; he or she will then contact the health care provider.

18.) The nurse is reviewing the history and physical examination of a client who will be receiving asparaginase (Elspar), an antineoplastic agent. The nurse consults with the registered nurse regarding the administration of the medication if which of the following is documented in the client's history? 1. Pancreatitis 2. Diabetes mellitus 3. Myocardial infarction 4. Chronic obstructive pulmonary disease

1. Pancreatitis Rationale: Asparaginase (Elspar) is contraindicated if hypersensitivity exists, in pancreatitis, or if the client has a history of pancreatitis. The medication impairs pancreatic function and pancreatic function tests should be performed before therapy begins and when a week or more has elapsed between administration of the doses. The client needs to be monitored for signs of pancreatitis, which include nausea, vomiting, and abdominal pain. The conditions noted in options 2, 3, and 4 are not contraindicated with this medication.

90.) A nurse is reviewing the record of a client who has been prescribed baclofen (Lioresal). Which of the following disorders, if noted in the client's history, would alert the nurse to contact the health care provider? 1. Seizure disorders 2. Hyperthyroidism 3. Diabetes mellitus 4. Coronary artery disease

1. Seizure disorders Rationale: Clients with seizure disorders may have a lowered seizure threshold when baclofen is administered. Concurrent therapy may require an increase in the anticonvulsive medication. The disorders in options 2, 3, and 4 are not a concern when the client is taking baclofen.

92.) In monitoring a client's response to disease-modifying antirheumatic drugs (DMARDs), which findings would the nurse interpret as acceptable responses? Select all that apply. 1. Symptom control during periods of emotional stress 2. Normal white blood cell counts, platelet, and neutrophil counts 3. Radiological findings that show nonprogression of joint degeneration 4. An increased range of motion in the affected joints 3 months into therapy 5. Inflammation and irritation at the injection site 3 days after injection is given 6. A low-grade temperature upon rising in the morning that remains throughout the day

1. Symptom control during periods of emotional stress 2. Normal white blood cell counts, platelet, and neutrophil counts 3. Radiological findings that show nonprogression of joint degeneration 4. An increased range of motion in the affected joints 3 months into therapy Rationale: Because emotional stress frequently exacerbates the symptoms of rheumatoid arthritis, the absence of symptoms is a positive finding. DMARDs are given to slow progression of joint degeneration. In addition, the improvement in the range of motion after 3 months of therapy with normal blood work is a positive finding. Temperature elevation and inflammation and irritation at the medication injection site could indicate signs of infection.

21.) A nurse is assisting with caring for a client with cancer who is receiving cisplatin. Select the adverse effects that the nurse monitors for that are associated with this medication. Select all that apply. 1. Tinnitus 2. Ototoxicity 3. Hyperkalemia 4. Hypercalcemia 5. Nephrotoxicity 6. Hypomagnesemia

1. Tinnitus 2. Ototoxicity 5. Nephrotoxicity 6. Hypomagnesemia Rationale: Cisplatin is an alkylating medication. Alkylating medications are cell cycle phase-nonspecific medications that affect the synthesis of DNA by causing the cross-linking of DNA to inhibit cell reproduction. Cisplatin may cause ototoxicity, tinnitus, hypokalemia, hypocalcemia, hypomagnesemia, and nephrotoxicity. Amifostine (Ethyol) may be administered before cisplatin to reduce the potential for renal toxicity.

79.) Ibuprofen (Advil) is prescribed for a client. The nurse tells the client to take the medication: 1. With 8 oz of milk 2. In the morning after arising 3. 60 minutes before breakfast 4. At bedtime on an empty stomach

1. With 8 oz of milk Rationale: Ibuprofen is a nonsteroidal anti-inflammatory drug (NSAID). NSAIDs should be given with milk or food to prevent gastrointestinal irritation. Options 2, 3, and 4 are incorrect.

183.) A client who received a kidney transplant is taking azathioprine (Imuran), and the nurse provides instructions about the medication. Which statement by the client indicates a need for further instructions? 1. "I need to watch for signs of infection." 2. "I need to discontinue the medication after 14 days of use." 3. "I can take the medication with meals to minimize nausea." 4. "I need to call the health care provider (HCP) if more than one dose is missed."

2. "I need to discontinue the medication after 14 days of use." Rationale: Azathioprine is an immunosuppressant medication that is taken for life. Because of the effects of the medication, the client must watch for signs of infection, which are reported immediately to the HCP. The client should also call the HCP if more than one dose is missed. The medication may be taken with meals to minimize nausea.

86.) A nurse is reinforcing discharge instructions to a client receiving baclofen (Lioresal). Which of the following would the nurse include in the instructions? 1. Restrict fluid intake. 2. Avoid the use of alcohol. 3. Stop the medication if diarrhea occurs. 4. Notify the health care provider if fatigue occurs.

2. Avoid the use of alcohol. Rationale: Baclofen is a central nervous system (CNS) depressant. The client should be cautioned against the use of alcohol and other CNS depressants, because baclofen potentiates the depressant activity of these agents. Constipation rather than diarrhea is an adverse effect of baclofen. It is not necessary to restrict fluids, but the client should be warned that urinary retention can occur. Fatigue is related to a CNS effect that is most intense during the early phase of therapy and diminishes with continued medication use. It is not necessary that the client notify the health care provider if fatigue occurs.

215.) A client with rheumatoid arthritis is taking acetylsalicylic acid (aspirin) on a daily basis. Which medication dose should the nurse expect the client to be taking? 1. 1 g daily 2. 4 g daily 3. 325 mg daily 4. 1000 mg daily

2. 4 g daily Rationale: Aspirin may be used to treat the client with rheumatoid arthritis. It may also be used to reduce the risk of recurrent transient ischemic attack (TIA) or brain attack (stroke) or reduce the risk of myocardial infarction (MI) in clients with unstable angina or a history of a previous MI. The normal dose for clients being treated with aspirin to decrease thrombosis and MI is 300 to 325 mg/day. Clients being treated to prevent TIAs are usually prescribed 1.3 g/day in two to four divided doses. Clients with rheumatoid arthritis are treated with 3.6 to 5.4 g/day in divided doses. **Eliminate options 1 and 4 because they are alike**

29.) A client is taking Humulin NPH insulin daily every morning. The nurse reinforces instructions for the client and tells the client that the most likely time for a hypoglycemic reaction to occur is: 1. 2 to 4 hours after administration 2. 4 to 12 hours after administration 3. 16 to 18 hours after administration 4. 18 to 24 hours after administration

2. 4 to 12 hours after administration Rationale: Humulin NPH is an intermediate-acting insulin. The onset of action is 1.5 hours, it peaks in 4 to 12 hours, and its duration of action is 24 hours. Hypoglycemic reactions most likely occur during peak time.

123.) A nurse is planning to administer amlodipine (Norvasc) to a client. The nurse plans to check which of the following before giving the medication? 1. Respiratory rate 2. Blood pressure and heart rate 3. Heart rate and respiratory rate 4. Level of consciousness and blood pressure

2. Blood pressure and heart rate Rationale: Amlodipine is a calcium channel blocker. This medication decreases the rate and force of cardiac contraction. Before administering a calcium channel blocking agent, the nurse should check the blood pressure and heart rate, which could both decrease in response to the action of this medication. This action will help to prevent or identify early problems related to decreased cardiac contractility, heart rate, and conduction. **amlodipine is a calcium channel blocker, and this group of medications decreases the rate and force of cardiac contraction. This in turn lowers the pulse rate and blood pressure.**

methadone hydrochloride

Opioid of choice for detox treatment of opioid addicts in methadone maintenance programs. Indications: Chronic (e.g. neuropathic) and cancer-related pain. - Bound in tissues of liver, kidneys, and brain. W/ repeated doses, it accumulates in these tissues and is slowly released, thus allowing for 24-hour dosing. Eliminated through the liver, making it safer for patients w/ renal impairment. May cause cardia dysrhythmias. Available in oral and injectable forms.

Vascular Pain

Originate from vascular or peripheral tissue (migraines)

Partial onset seizures

Originate in a more localized region of the brain (also called focal seizures).

84.) Baclofen (Lioresal) is prescribed for the client with multiple sclerosis. The nurse assists in planning care, knowing that the primary therapeutic effect of this medication is which of the following? 1. Increased muscle tone 2. Decreased muscle spasms 3. Increased range of motion 4. Decreased local pain and tenderness

2. Decreased muscle spasms Rationale: Baclofen is a skeletal muscle relaxant and central nervous system depressant and acts at the spinal cord level to decrease the frequency and amplitude of muscle spasms in clients with spinal cord injuries or diseases and in clients with multiple sclerosis. Options 1, 3, and 4 are incorrect.

209.) A client with multiple sclerosis is receiving diazepam (Valium), a centrally acting skeletal muscle relaxant. Which of the following would indicate that the client is experiencing a side effect related to this medication? 1. Headache 2. Drowsiness 3. Urinary retention 4. Increased salivation

2. Drowsiness Rationale: Incoordination and drowsiness are common side effects resulting from this medication. Options 1, 3, and 4 are incorrect.

133.) A nurse is monitoring a client receiving desmopressin acetate (DDAVP) for adverse effects to the medication. Which of the following indicates the presence of an adverse effect? 1. Insomnia 2. Drowsiness 3. Weight loss 4. Increased urination

2. Drowsiness Rationale: Water intoxication (overhydration) or hyponatremia is an adverse effect to desmopressin. Early signs include drowsiness, listlessness, and headache. Decreased urination, rapid weight gain, confusion, seizures, and coma also may occur in overhydration. **Recall that this medication is used to treat diabetes insipidus to eliminate weight loss and increased urination.**

The hcp orders EXENATIDE (Byetta) for a client with type 1 DM who takes insulin. The nurse plans to take which appropriate intervention? 1. Administer the med within 60 minutes before the morning meal and eve meal. 2. Hold the medication and call the hcp questioning the order for the client. 3. Monitor the client for gastrointestinal side effects after administering the med. 4. Withdraw the insulin from the penlet into an insulin syringe to prepare for administration.

2. Hold the medication and call the hcp questioning the order for the client. this medication is only used for patients with type 2 DM

Generalized onset seizures (Grand mal)

Originate simultaneously in both cerebral hemispheres.

162.) Carbamazepine (Tegretol) is prescribed for a client with a diagnosis of psychomotor seizures. The nurse reviews the client's health history, knowing that this medication is contraindicated if which of the following disorders is present? 1. Headaches 2. Liver disease 3. Hypothyroidism 4. Diabetes mellitus

2. Liver disease Rationale: Carbamazepine (Tegretol) is contraindicated in liver disease, and liver function tests are routinely prescribed for baseline purposes and are monitored during therapy. It is also contraindicated if the client has a history of blood dyscrasias. It is not contraindicated in the conditions noted in the incorrect options.

89.) A nurse is reviewing the laboratory studies on a client receiving dantrolene sodium (Dantrium). Which laboratory test would identify an adverse effect associated with the administration of this medication? 1. Creatinine 2. Liver function tests 3. Blood urea nitrogen 4. Hematological function tests

2. Liver function tests Rationale: Dose-related liver damage is the most serious adverse effect of dantrolene. To reduce the risk of liver damage, liver function tests should be performed before treatment and periodically throughout the treatment course. It is administered in the lowest effective dosage for the shortest time necessary. **Eliminate options 1 and 3 because these tests both assess kidney function.**

Visceral Pain

Originates from organs and smooth muscle

Visceral Pain

Originates from organs and smooth muscles. Usually require opioids.

165.) The client has been on treatment for rheumatoid arthritis for 3 weeks. During the administration of etanercept (Enbrel), it is most important for the nurse to assess: 1. The injection site for itching and edema 2. The white blood cell counts and platelet counts 3. Whether the client is experiencing fatigue and joint pain 4. A metallic taste in the mouth and a loss of appetite

2. The white blood cell counts and platelet counts Rationale: Infection and pancytopenia are adverse effects of etanercept (Enbrel). Laboratory studies are performed before and during treatment. The appearance of abnormal white blood cell counts and abnormal platelet counts can alert the nurse to a potential life-threatening infection. Injection site itching is a common occurrence following administration of the medication. In early treatment, residual fatigue and joint pain may still be apparent. A metallic taste and loss of appetite are not common signs of side effects of this medication.

Somatic Pain

Originates from skeletal muscles, ligaments, and joints. Usually responds better to nonopioid analgesics such as NSAIDs.

Superficial Pain

Originates from skin and mucous membranes. Usually requires opioids.

What type of diuretic can be used to reduce ICP and early renal failure?

Osmotic :mannitol (filter needle)

50.) A nurse has given a client taking ethambutol (Myambutol) information about the medication. The nurse determines that the client understands the instructions if the client states that he or she will immediately report: 1. Impaired sense of hearing 2. Problems with visual acuity 3. Gastrointestinal (GI) side effects 4. Orange-red discoloration of body secretions

2. Problems with visual acuity Rationale: Ethambutol causes optic neuritis, which decreases visual acuity and the ability to discriminate between the colors red and green. This poses a potential safety hazard when a client is driving a motor vehicle. The client is taught to report this symptom immediately. The client is also taught to take the medication with food if GI upset occurs. Impaired hearing results from antitubercular therapy with streptomycin. Orange-red discoloration of secretions occurs with rifampin (Rifadin).

Gingival hyperplasia

Overgrowth of gum tissue and often a side effects of phenytoin.

25.) A home care nurse visits a client recently diagnosed with diabetes mellitus who is taking Humulin NPH insulin daily. The client asks the nurse how to store the unopened vials of insulin. The nurse tells the client to: 1. Freeze the insulin. 2. Refrigerate the insulin. 3. Store the insulin in a dark, dry place. 4. Keep the insulin at room temperature.

2. Refrigerate the insulin. Rationale: Insulin in unopened vials should be stored under refrigeration until needed. Vials should not be frozen. When stored unopened under refrigeration, insulin can be used up to the expiration date on the vial. Options 1, 3, and 4 are incorrect.

168.) Colcrys (colchicine) is prescribed for a client with a diagnosis of gout. The nurse reviews the client's medical history in the health record, knowing that the medication would be contraindicated in which disorder? 1. Myxedema 2. Renal failure 3. Hypothyroidism 4. Diabetes mellitus

2. Renal failure Rationale: Colchicine is contraindicated in clients with severe gastrointestinal, renal, hepatic or cardiac disorders, or with blood dyscrasias. Clients with impaired renal function may exhibit myopathy and neuropathy manifested as generalized weakness. This medication should be used with caution in clients with impaired hepatic function, older clients, and debilitated clients. **Note that options 1, 3, and 4 are all endocrine-related disorders: Myxedema=Hypothyroidism**

114.) A postoperative client requests medication for flatulence (gas pains). Which medication from the following PRN list should the nurse administer to this client? 1. Ondansetron (Zofran) 2. Simethicone (Mylicon) 3. Acetaminophen (Tylenol) 4. Magnesium hydroxide (milk of magnesia, MOM)

2. Simethicone (Mylicon) Rationale: Simethicone is an antiflatulent used in the relief of pain caused by excessive gas in the gastrointestinal tract. Ondansetron is used to treat postoperative nausea and vomiting. Acetaminophen is a nonopioid analgesic. Magnesium hydroxide is an antacid and laxative.

______ is injected into the tissues to reverse the damaging effects of a DOPAMINE infiltration.

PHENTOLAMINE

A client with bipolar disorder is being treated with LITHIUM for the 1st time. The nurse should observe the client for which common adverse effect of LITHIUM?

POLYURIA commonly occurs early in the treatment with lithium and could result in fluid volume deficit.

First line therapy for GERD that has not responded to medical treatment?

PPI

42.) A client with a peptic ulcer is diagnosed with a Helicobacter pylori infection. The nurse is reinforcing teaching for the client about the medications prescribed, including clarithromycin (Biaxin), esomeprazole (Nexium), and amoxicillin (Amoxil). Which statement by the client indicates the best understanding of the medication regimen? 1. "My ulcer will heal because these medications will kill the bacteria." 2. "These medications are only taken when I have pain from my ulcer." 3. "The medications will kill the bacteria and stop the acid production." 4. "These medications will coat the ulcer and decrease the acid production in my stomach."

3. "The medications will kill the bacteria and stop the acid production." Rationale: Triple therapy for Helicobacter pylori infection usually includes two antibacterial drugs and a proton pump inhibitor. Clarithromycin and amoxicillin are antibacterials. Esomeprazole is a proton pump inhibitor. These medications will kill the bacteria and decrease acid production.

141.) The nurse has reinforced instructions to a client who has been prescribed cholestyramine (Questran). Which statement by the client indicates a need for further instructions? 1. "I will continue taking vitamin supplements." 2. "This medication will help lower my cholesterol." 3. "This medication should only be taken with water." 4. "A high-fiber diet is important while taking this medication."

3. "This medication should only be taken with water." Rationale: Cholestyramine (Questran) is a bile acid sequestrant used to lower the cholesterol level, and client compliance is a problem because of its taste and palatability. The use of flavored products or fruit juices can improve the taste. Some side effects of bile acid sequestrants include constipation and decreased vitamin absorption. **Note the closed-ended word "only" in option 3**

Sucralfate (carafate)

PPI used for mucosal protectant for ulcers and PUD

Drugs that block all acid secretion in the stomach

PPI: Proton Pump Inhibitor Omeprazole (Prilosec) Lansoprazole (Prevacid) Pantoprazole (Protonix)

120.) A client is taking lansoprazole (Prevacid) for the chronic management of Zollinger-Ellison syndrome. The nurse advises the client to take which of the following products if needed for a headache? 1. Naprosyn (Aleve) 2. Ibuprofen (Advil) 3. Acetaminophen (Tylenol) 4. Acetylsalicylic acid (aspirin)

3. Acetaminophen (Tylenol) Rationale: Zollinger-Ellison syndrome is a hypersecretory condition of the stomach. The client should avoid taking medications that are irritating to the stomach lining. Irritants would include aspirin and nonsteroidal antiinflammatory drugs (ibuprofen). The client should be advised to take acetaminophen for headache. **Remember that options that are comparable or alike are not likely to be correct. With this in mind, eliminate options 1 and 2 first.**

207.) A client is suspected of having myasthenia gravis, and the health care provider administers edrophonium (Enlon) to determine the diagnosis. After administration of this medication, which of the following would indicate the presence of myasthenia gravis? 1. Joint pain 2. A decrease in muscle strength 3. An increase in muscle strength 4. Feelings of faintness, dizziness, hypotension, and signs of flushing in the client

3. An increase in muscle strength Rationale: Edrophonium is a short-acting acetylcholinesterase inhibitor used as a diagnostic agent. When a client with suspected myasthenia gravis is given the medication intravenously, an increase in muscle strength would be seen in 1 to 3 minutes. If no response occurs, another dose is given over the next 2 minutes, and muscle strength is again tested. If no increase in muscle strength occurs with this higher dose, the muscle weakness is not caused by myasthenia gravis. Clients receiving injections of this medication commonly demonstrate a drop of blood pressure, feel faint and dizzy, and are flushed.

139.) Prednisone is prescribed for a client with diabetes mellitus who is taking Humulin neutral protamine Hagedorn (NPH) insulin daily. Which of the following prescription changes does the nurse anticipate during therapy with the prednisone? 1. An additional dose of prednisone daily 2. A decreased amount of daily Humulin NPH insulin 3. An increased amount of daily Humulin NPH insulin 4. The addition of an oral hypoglycemic medication daily

3. An increased amount of daily Humulin NPH insulin Rationale: Glucocorticoids can elevate blood glucose levels. Clients with diabetes mellitus may need their dosages of insulin or oral hypoglycemic medications increased during glucocorticoid therapy. Therefore the other options are incorrect.

245.) A client taking carbamazepine (Tegretol) asks the nurse what to do if he misses one dose. The nurse responds that the carbamazepine should be: 1. Withheld until the next scheduled dose 2. Withheld and the health care provider is notified immediately 3. Taken as long as it is not immediately before the next dose 4. Withheld until the next scheduled dose, which should then be doubled

3. Taken as long as it is not immediately before the next dose Rationale: Carbamazepine is an anticonvulsant that should be taken around the clock, precisely as directed. If a dose is omitted, the client should take the dose as soon as it is remembered, as long as it is not immediately before the next dose. The medication should not be double dosed. If more than one dose is omitted, the client should call the health care provider.

244.) A client has a prescription for valproic acid (Depakene) orally once daily. The nurse plans to: 1. Administer the medication with an antacid. 2. Administer the medication with a carbonated beverage. 3. Ensure that the medication is administered at the same time each day. 4. Ensure that the medication is administered 2 hours before breakfast only, when the client's stomach is empty.

3. Ensure that the medication is administered at the same time each day. Rationale: Valproic acid is an anticonvulsant, antimanic, and antimigraine medication. It may be administered with or without food. It should not be taken with an antacid or carbonated beverage because these products will affect medication absorption. The medication is administered at the same time each day to maintain therapeutic serum levels. **Use general pharmacology guidelines to assist in eliminating options 1 and 2. Eliminate option 4 because of the closed-ended word "only."**

a clinic nurse is reviewing prescriptions for a 5 y/o who has a new dx of generalized contact dermatitis. which medication prescription should the nurse question?

PREDNISONE would not be the 1st line of treatment for contact dermatitis. this med could be used later on. expected med for contact dermatitis includes: -diphenhydramine -calamine lotion -hydrocortisone cream

150.) A client complaining of not feeling well is seen in a clinic. The client is taking several medications for the control of heart disease and hypertension. These medications include a β-blocker, digoxin (Lanoxin), and a diuretic. A tentative diagnosis of digoxin toxicity is made. Which of the following assessment data would support this diagnosis? 1. Dyspnea, edema, and palpitations 2. Chest pain, hypotension, and paresthesia 3. Double vision, loss of appetite, and nausea 4. Constipation, dry mouth, and sleep disorder

3. Double vision, loss of appetite, and nausea Rationale: Double vision, loss of appetite, and nausea are signs of digoxin toxicity. Additional signs of digoxin toxicity include bradycardia, difficulty reading, visual alterations such as green and yellow vision or seeing spots or halos, confusion, vomiting, diarrhea, decreased libido, and impotence. **gastrointestinal (GI) and visual disturbances occur with digoxin toxicity**

85.) A nurse is monitoring a client receiving baclofen (Lioresal) for side effects related to the medication. Which of the following would indicate that the client is experiencing a side effect? 1. Polyuria 2. Diarrhea 3. Drowsiness 4. Muscular excitability

3. Drowsiness Rationale: Baclofen is a central nervous system (CNS) depressant and frequently causes drowsiness, dizziness, weakness, and fatigue. It can also cause nausea, constipation, and urinary retention. Clients should be warned about the possible reactions. Options 1, 2, and 4 are not side effects.

138.) A daily dose of prednisone is prescribed for a client. A nurse reinforces instructions to the client regarding administration of the medication and instructs the client that the best time to take this medication is: 1. At noon 2. At bedtime 3. Early morning 4. Anytime, at the same time, each day

3. Early morning Rationale: Corticosteroids (glucocorticoids) should be administered before 9:00 AM. Administration at this time helps minimize adrenal insufficiency and mimics the burst of glucocorticoids released naturally by the adrenal glands each morning. **Note the suffix "-sone," and recall that medication names that end with these letters are corticosteroids.**

a client has been on long term PREDNISONE therapy. The nurse should instruct the client to consume a diet hight in which of the following? SATA 1. carbohydrate 2. protein 3. trans fat 4. potassium 5. calcium 6. vitamin D

PROTEIN, POTASSIUM, CALCIUM, VITAMIN D

97.) Amikacin (Amikin) is prescribed for a client with a bacterial infection. The client is instructed to contact the health care provider (HCP) immediately if which of the following occurs? 1. Nausea 2. Lethargy 3. Hearing loss 4. Muscle aches

3. Hearing loss Rationale: Amikacin (Amikin) is an aminoglycoside. Adverse effects of aminoglycosides include ototoxicity (hearing problems), confusion, disorientation, gastrointestinal irritation, palpitations, blood pressure changes, nephrotoxicity, and hypersensitivity. The nurse instructs the client to report hearing loss to the HCP immediately. Lethargy and muscle aches are not associated with the use of this medication. It is not necessary to contact the HCP immediately if nausea occurs. If nausea persists or results in vomiting, the HCP should be notified. **(most aminoglycoside medication names end in the letters -cin)**

What is one disadvantage of using the rectal route? 1. It can cause orthostatic hypotension. 2. It can cause hypersensitivity to the drug. 3. It can result in incomplete drug absorption. 4. It can cause rectal tears.

3. It can result in incomplete drug absorption.incomplete drug absorption is a disadvantage of rectal drug administration. The drug itself, not the way in which it is administered, may cause orthostatic hypotension or hypersensitivity reactions. If inserted properly, drugs won't cause rectal tears.

210.) Dantrolene (Dantrium) is prescribed for a client with a spinal cord injury for discomfort resulting from spasticity. The nurse tells the client about the importance of follow-up and the need for which blood study? 1. Creatinine level 2. Sedimentation rate 3. Liver function studies 4. White blood cell count

3. Liver function studies Rationale: Dantrolene can cause liver damage, and the nurse should monitor liver function studies. Baseline liver function studies are done before therapy starts, and regular liver function studies are performed throughout therapy. Dantrolene is discontinued if no relief of spasticity is achieved in 6 weeks.

Which of the following nursing assessments is a LATE symptom of polycythemia vera?(Polycythemia vera is a bone marrow disease that leads to an abnormal increase in the number of blood cells. The red blood cells are mostly affected)

PRURITUS is a late symptom that results from abnormal histamine metabolism. Headache and dizziness are early symptoms from engorged veins. Shortness of breath is an early symptom from congested mucous membrane and ineffective gas exchange.

Breakthrough Pain

Pain that often occurs between doses of pain meds. This is bc the analgesic effects wear off as the drug is metabolized and eliminated from the body. Treatment with PRN doses of immediate-release dosage forms given between scheduled doses of extended-release dosage forms is often helpful in these cases. **Chewing or crushing of any extended-release opioid drug can cause over sedation, respiratory depression, and even death due to rapid drug absorption.

SNRIs are used for

Pain, diabetic neuropathy

229.) A client who is taking lithium carbonate (Lithobid) is scheduled for surgery. The nurse informs the client that: 1. The medication will be discontinued a week before the surgery and resumed 1 week postoperatively. 2. The medication is to be taken until the day of surgery and resumed by injection immediately postoperatively. 3. The medication will be discontinued 1 to 2 days before the surgery and resumed as soon as full oral intake is allowed. 4. The medication will be discontinued several days before surgery and resumed by injection in the immediate postoperative period.

3. The medication will be discontinued 1 to 2 days before the surgery and resumed as soon as full oral intake is allowed. Rationale: The client who is on lithium carbonate must be off the medication for 1 to 2 days before a scheduled surgical procedure and can resume the medication when full oral intake is prescribed after the surgery. **lithium carbonate is an oral medication and is not given as an injection**

after sustaining a closed head injury, a client is prescribed PHENYTOIN 100mg IV every 8 hrs for seizure prophylaxis. which nursing intervention are necessary when administering PHENYTOIN? 1. administer phenytoin through any peripheral IV site 2. mix IV doses in solutions containing dextrose 5% in water 3. administer an IV bolus no faster than 50 mg/minute 4. monitor ECG, BP, and respiratory status continuously when administering phenytoin IV 5. do not use an inline filter when administering the drug. 6. keep in mind that early toxicity may cause drowsiness, n&v, nystagmus, ataxia, dysarthria, tremor and slurred speech

3. administer an IV bolus no faster than 50 mg/minute 4. monitor ECG, BP, and respiratory status continuously when administering phenytoin IV 6. keep in mind that early toxicity may cause drowsiness, n&v, nystagmus, ataxia, dysarthria, tremor and slurred speech phenytoin should not be administered by IV push in veins on the back of the hand; larger veins are needed to prevent discoloration associated with purple glove syndrome. use of inline filter is recommend.

a nurse is providing medication instructions to a client with HIV who will be taking SAQUINAVIR. the nurse instructs the client to take the medication: 1. on an empty stomach 2. 2 hours before breakfast 3. within 2 hours after a full meal 4. at bedtime

3. within 2 hours after a full meal SAQUINAVIR is an antiviral medication. It is administered within 2 hrs after a full meal. If the medication is taken without food in the stomach, it may result in no antiviral activity.

124.) A client with chronic renal failure is receiving ferrous sulfate (Feosol). The nurse monitors the client for which common side effect associated with this medication? 1. Diarrhea 2. Weakness 3. Headache 4. Constipation

4. Constipation Rationale: Feosol is an iron supplement used to treat anemia. Constipation is a frequent and uncomfortable side effect associated with the administration of oral iron supplements. Stool softeners are often prescribed to prevent constipation. **Focus on the name of the medication. Recalling that oral iron can cause constipation will easily direct you to the correct option.**

118.) A nurse is caring for an older client with a diagnosis of myasthenia gravis and has reinforced self-care instructions. Which statement by the client indicates that further teaching is necessary? 1. "I rest each afternoon after my walk." 2. "I cough and deep breathe many times during the day." 3. "If I get abdominal cramps and diarrhea, I should call my doctor." 4. "I can change the time of my medication on the mornings that I feel strong."

4. "I can change the time of my medication on the mornings that I feel strong." Rationale: The client with myasthenia gravis should be taught that timing of anticholinesterase medication is critical. It is important to instruct the client to administer the medication on time to maintain a chemical balance at the neuromuscular junction. If not given on time, the client may become too weak to swallow. Options 1, 2, and 3 include the necessary information that the client needs to understand to maintain health with this neurological degenerative disease.

62.) A client is on nicotinic acid (niacin) for hyperlipidemia and the nurse provides instructions to the client about the medication. Which statement by the client would indicate an understanding of the instructions? 1. "It is not necessary to avoid the use of alcohol." 2. "The medication should be taken with meals to decrease flushing." 3. "Clay-colored stools are a common side effect and should not be of concern." 4. "Ibuprofen (Motrin) taken 30 minutes before the nicotinic acid should decrease the flushing."

4. "Ibuprofen (Motrin) taken 30 minutes before the nicotinic acid should decrease the flushing." Rationale: Flushing is a side effect of this medication. Aspirin or a nonsteroidal anti-inflammatory drug can be taken 30 minutes before taking the medication to decrease flushing. Alcohol consumption needs to be avoided because it will enhance this side effect. The medication should be taken with meals, this will decrease gastrointestinal upset. Taking the medication with meals has no effect on the flushing. Clay-colored stools are a sign of hepatic dysfunction and should be immediately reported to the health care provider (HCP).

218.) A film-coated form of diflunisal has been prescribed for a client for the treatment of chronic rheumatoid arthritis. The client calls the clinic nurse because of difficulty swallowing the tablets. Which initial instruction should the nurse provide to the client? 1. "Crush the tablets and mix them with food." 2. "Notify the health care provider for a medication change." 3. "Open the tablet and mix the contents with food." 4. "Swallow the tablets with large amounts of water or milk."

4. "Swallow the tablets with large amounts of water or milk." Rationale: Diflunisal may be given with water, milk, or meals. The tablets should not be crushed or broken open. Taking the medication with a large amount of water or milk should be tried before contacting the health care provider.

198.) Coal tar has been prescribed for a client with a diagnosis of psoriasis, and the nurse provides instructions to the client about the medication. Which statement by the client indicates a need for further instructions? 1. "The medication can cause phototoxicity." 2. "The medication has an unpleasant odor." 3. "The medication can stain the skin and hair." 4. "The medication can cause systemic effects."

4. "The medication can cause systemic effects." Rationale: Coal tar is used to treat psoriasis and other chronic disorders of the skin. It suppresses DNA synthesis, mitotic activity, and cell proliferation. It has an unpleasant odor, can frequently stain the skin and hair, and can cause phototoxicity. Systemic toxicity does not occur. **The name of the medication will assist in eliminating options 2 and 3**

242.) A client who was started on anticonvulsant therapy with clonazepam (Klonopin) tells the nurse of increasing clumsiness and unsteadiness since starting the medication. The client is visibly upset by these manifestations and asks the nurse what to do. The nurse's response is based on the understanding that these symptoms: 1. Usually occur if the client takes the medication with food 2. Are probably the result of an interaction with another medication 3. Indicate that the client is experiencing a severe untoward reaction to the medication 4. Are worse during initial therapy and decrease or disappear with long-term use

4. Are worse during initial therapy and decrease or disappear with long-term use Rationale: Drowsiness, unsteadiness, and clumsiness are expected effects of the medication during early therapy. They are dose related and usually diminish or disappear altogether with continued use of the medication. It does not indicate that a severe side effect is occurring. It is also unrelated to interaction with another medication. The client is encouraged to take this medication with food to minimize gastrointestinal upset. **Eliminate options 2 and 3 first because they are comparable or alike and because of the word "severe" in option 3**

155.) Mycophenolate mofetil (CellCept) is prescribed for a client as prophylaxis for organ rejection following an allogeneic renal transplant. Which of the following instructions does the nurse reinforce regarding administration of this medication? 1. Administer following meals. 2. Take the medication with a magnesium-type antacid. 3. Open the capsule and mix with food for administration. 4. Contact the health care provider (HCP) if a sore throat occurs.

4. Contact the health care provider (HCP) if a sore throat occurs. Rationale: Mycophenolate mofetil should be administered on an empty stomach. The capsules should not be opened or crushed. The client should contact the HCP if unusual bleeding or bruising, sore throat, mouth sores, abdominal pain, or fever occurs because these are adverse effects of the medication. Antacids containing magnesium and aluminum may decrease the absorption of the medication and therefore should not be taken with the medication. The medication may be given in combination with corticosteroids and cyclosporine. **neutropenia can occur with this medication**

94.) The client with acquired immunodeficiency syndrome has begun therapy with zidovudine (Retrovir, Azidothymidine, AZT, ZDV). The nurse carefully monitors which of the following laboratory results during treatment with this medication? 1. Blood culture 2. Blood glucose level 3. Blood urea nitrogen 4. Complete blood count

4. Complete blood count Rationale: A common side effect of therapy with zidovudine is leukopenia and anemia. The nurse monitors the complete blood count results for these changes. Options 1, 2, and 3 are unrelated to the use of this medication.

Carbamazepine (Tegretol) (Iminostilbene)

Second most commonly prescribed anti epileptic drug in the US, after phenytoin. Considered a first-line treatment for partial seizures and generalized tonic-clonic seizures. May actually WORSEN myoclonic or absence seizures. Contraindicated in both of these as well as in cases of known drug allergy and bone marrow depression. Associated with autoinduction (process in which, over time, a drug stimulates the production of enzymes that enhance its own metabolism, which leads to lower than expected drug concentrations) of hepatic enzymes.

Which classes of drugs are used as adjunctive drugs with anesthesia?

Sedative Hypnotics/Anxiolytics (Propofol) Benzodiazepines (Diazepam, midazolam) Barbituates (tiopental) Opioid Analgesics ( Fentanyl, morphine) Anticholinergics (Atropine) Antiemetics (Ondansteron)

SSRI Mechanism/Therapeutic Effects

Selective serotonin reuptake inhibitors (SSRIs) inhibit the reuptake of serotonin into presynaptic terminals (nerve endings) and thus increase the levels of serotonin available for neurotransmission at the postsynaptic nerve endings. They also demonstrate weak inhibition of norepinephrine and dopamine reuptake. Increased levels of these neurotransmitters are responsible for the improvement of the symptoms of depression. SSRIs are the first-line treatment for major depression and are also indicated for the treatment of obsessive-compulsive disorder, anxiety disorders, panic disorders, and compulsive eating disorders such as bulimia nervosa.

Antiemetic Chemotherapy drugs

Serotonin Blockers Tetrahydrocannabinoids

Antimigraine Drugs

Serotonin receptor agonists (-triptans) - work by stimulating serotonin receptors in brain. Should be used cautiously in patients with severe cardiovascular disease (esp. angina). Ergot alkaloids: Obtained from a fungus and cause vasoconstriction of dilated blood vessels in the brain and of the carotid arteries. Contraindicated in patients with PVD, CAD, sepsis, impaired renal or hepatic function, and severe HTN.

Indications for direct-acting cholinergic drugs

Carbachol Pilocarpine Echothiophate -Used topically to reduce intraocular pressure in patients w/ glaucoma or in those undergoing ocular surgery. -Poorly absorbed orally, limiting them to topical use. -Bethanechol affects the detrusor muscle of the urinary bladder and also the moth muscle of the GI tract. Causes increased bladder and GI tract tone and motility, which increases the movement of contents through these areas. Also causes the sphincters in the bladder and the GI tract to relax, allowing them to empty. Also used to treat atony of the bladder and the GI tract. -Cevimeline is used to treat xerostomia resulting from a disorder known as Sjogren's syndrome. -Oral pilocarpine can also be used for this purpose. -Another direct-acting cholinergic is succinylcholine, which is used as a neuromuscular blocker in general anesthesia.

A nurse admits a 23-year-old client to the emergency department in a status epilepticus state. The nurse anticipates the following drug will be needed: 1. Phenytoin (Dilantin) 2. Phenobarbital 3. Valproic acid 4. Diazepam (Valium)

4. Diazepam (Valium) Valium is the drug of choice to terminate seizures of status epileptics (1). Dilantin is used to treat seizure activity on a daily basis (2). Phenobarbital is used to treat seizure activity on daily basis (3). Valproic acid is used for absence seizures

236.) A client is being treated for depression with amitriptyline hydrochloride. During the initial phases of treatment, the most important nursing intervention is: 1. Prescribing the client a tyramine-free diet 2. Checking the client for anticholinergic effects 3. Monitoring blood levels frequently because there is a narrow range between therapeutic and toxic blood levels of this medication 4. Getting baseline postural blood pressures before administering the medication and each time the medication is administered

4. Getting baseline postural blood pressures before administering the medication and each time the medication is administered Rationale: Amitriptyline hydrochloride is a tricyclic antidepressant often used to treat depression. It causes orthostatic changes and can produce hypotension and tachycardia. This can be frightening to the client and dangerous because it can result in dizziness and client falls. The client must be instructed to move slowly from a lying to a sitting to a standing position to avoid injury if these effects are experienced. The client may also experience sedation, dry mouth, constipation, blurred vision, and other anticholinergic effects, but these are transient and will diminish with time.

196.) A client has been prescribed amikacin (Amikin). Which of the following priority baseline functions should be monitored? 1. Apical pulse 2. Liver function 3. Blood pressure 4. Hearing acuity

4. Hearing acuity Rationale: Amikacin (Amikin) is an antibiotic. This medication can cause ototoxicity and nephrotoxicity; therefore, hearing acuity tests and kidney function studies should be performed before the initiation of therapy. Apical pulse, liver function studies, and blood pressure are not specifically related to the use of this medication.

36.) The client has a PRN prescription for ondansetron (Zofran). For which condition should this medication be administered to the postoperative client? 1. Paralytic ileus 2. Incisional pain 3. Urinary retention 4. Nausea and vomiting

4. Nausea and vomiting Rationale: Ondansetron is an antiemetic used to treat postoperative nausea and vomiting, as well as nausea and vomiting associated with chemotherapy. The other options are incorrect.

Indirect-acting sympathomimetic

Causes the release of the catecholamine from the storage sites (vesicles) in the nerve endings; it then binds to the receptors and causes a physiologic response. Ex. Amphetamine and other related anorexiants.

175.) A nurse notes that a client is receiving lamivudine (Epivir). The nurse determines that this medication has been prescribed to treat which of the following? 1. Pancreatitis 2. Pharyngitis 3. Tonic-clonic seizures 4. Human immunodeficiency virus (HIV) infection

4. Human immunodeficiency virus (HIV) infection Rationale: Lamivudine is a nucleoside reverse transcriptase inhibitor and antiviral medication. It slows HIV replication and reduces the progression of HIV infection. It also is used to treat chronic hepatitis B and is used for prophylaxis in health care workers at risk of acquiring HIV after occupational exposure to the virus. **Note the letters "-vir" in the trade name for this medication**

119.) A client with diabetes mellitus who has been controlled with daily insulin has been placed on atenolol (Tenormin) for the control of angina pectoris. Because of the effects of atenolol, the nurse determines that which of the following is the most reliable indicator of hypoglycemia? 1. Sweating 2. Tachycardia 3. Nervousness 4. Low blood glucose level

4. Low blood glucose level Rationale: β-Adrenergic blocking agents, such as atenolol, inhibit the appearance of signs and symptoms of acute hypoglycemia, which would include nervousness, increased heart rate, and sweating. Therefore, the client receiving this medication should adhere to the therapeutic regimen and monitor blood glucose levels carefully. Option 4 is the most reliable indicator of hypoglycemia.

Common SSRIs

Celexa (citalopram) Escitalopram (lexapro) Fluoxetine (Prozac) Fluvoxamine (Luvox) Paraxetine (Paxil) Sertaline (Zoloft)

17.) The client with ovarian cancer is being treated with vincristine (Oncovin). The nurse monitors the client, knowing that which of the following indicates a side effect specific to this medication? 1. Diarrhea 2. Hair loss 3. Chest pain 4. Numbness and tingling in the fingers and toes

4. Numbness and tingling in the fingers and toes Rationale: A side effect specific to vincristine is peripheral neuropathy, which occurs in almost every client. Peripheral neuropathy can be manifested as numbness and tingling in the fingers and toes. Depression of the Achilles tendon reflex may be the first clinical sign indicating peripheral neuropathy. Constipation rather than diarrhea is most likely to occur with this medication, although diarrhea may occur occasionally. Hair loss occurs with nearly all the antineoplastic medications. Chest pain is unrelated to this medication.

13.) The client with squamous cell carcinoma of the larynx is receiving bleomycin intravenously. The nurse caring for the client anticipates that which diagnostic study will be prescribed? 1. Echocardiography 2. Electrocardiography 3. Cervical radiography 4. Pulmonary function studies

4. Pulmonary function studies Rationale: Bleomycin is an antineoplastic medication (Chemotheraputic Agents) that can cause interstitial pneumonitis, which can progress to pulmonary fibrosis. Pulmonary function studies along with hematological, hepatic, and renal function tests need to be monitored. The nurse needs to monitor lung sounds for dyspnea and crackles, which indicate pulmonary toxicity. The medication needs to be discontinued immediately if pulmonary toxicity occurs. Options 1, 2, and 3 are unrelated to the specific use of this medication.

Cholinergic crisis

Severe muscle weakness and respiratory paralysis due to excessive acetylcholine; often seen in patients w/ myasthenia gravis as an adverse effect of drugs used to treat the disorder.

70.) Oxybutynin chloride (Ditropan XL) is prescribed for a client with neurogenic bladder. Which sign would indicate a possible toxic effect related to this medication? 1. Pallor 2. Drowsiness 3. Bradycardia 4. Restlessness

4. Restlessness Rationale: Toxicity (overdosage) of this medication produces central nervous system excitation, such as nervousness, restlessness, hallucinations, and irritability. Other signs of toxicity include hypotension or hypertension, confusion, tachycardia, flushed or red face, and signs of respiratory depression. Drowsiness is a frequent side effect of the medication but does not indicate overdosage.

66.) Trimethoprim-sulfamethoxazole (TMP-SMZ) is prescribed for a client. A nurse should instruct the client to report which symptom if it developed during the course of this medication therapy? 1. Nausea 2. Diarrhea 3. Headache 4. Sore throat

4. Sore throat Rationale: Clients taking trimethoprim-sulfamethoxazole (TMP-SMZ) should be informed about early signs of blood disorders that can occur from this medication. These include sore throat, fever, and pallor, and the client should be instructed to notify the health care provider if these symptoms occur. The other options do not require health care provider notification.

44.) A client is receiving acetylcysteine (Mucomyst), 20% solution diluted in 0.9% normal saline by nebulizer. The nurse should have which item available for possible use after giving this medication? 1. Ambu bag 2. Intubation tray 3. Nasogastric tube 4. Suction equipment

4. Suction equipment Rationale: Acetylcysteine can be given orally or by nasogastric tube to treat acetaminophen overdose, or it may be given by inhalation for use as a mucolytic. The nurse administering this medication as a mucolytic should have suction equipment available in case the client cannot manage to clear the increased volume of liquefied secretions.

99.) The client with acquired immunodeficiency syndrome and Pneumocystis jiroveci infection has been receiving pentamidine isethionate (Pentam 300). The client develops a temperature of 101° F. The nurse does further monitoring of the client, knowing that this sign would most likely indicate: 1. The dose of the medication is too low. 2. The client is experiencing toxic effects of the medication. 3. The client has developed inadequacy of thermoregulation. 4. The result of another infection caused by leukopenic effects of the medication.

4. The result of another infection caused by leukopenic effects of the medication. Rationale: Frequent side effects of this medication include leukopenia, thrombocytopenia, and anemia. The client should be monitored routinely for signs and symptoms of infection. Options 1, 2, and 3 are inaccurate interpretations.

Mechanism of action and effects of antipsychotics

All antipsychotics block dopamine receptors in the brain, which decreases the dopamine concentration in the CNS. The conventional phenothiazines block the dopamine receptors postsynaptically in certain areas of the CNS, such as the limbic system and the basal ganglia. These are the ares associated with emotions, cognitive function, and motor function. Receptor blocking produces a tranquilizing effect in psychotic patients. Both the therapeutic and toxic effects of these drugs are the direct result of of the dopamine blockade in these areas.

The nurse in the health care clinic receives a telephone call from the mother of a child who reports that an insect has somehow flown into the child's ear. The mother reports that the child is complaining of a buzzing sound in the ear. Which priority instruction should the nurse provide to the mother? 1. Irrigate the ear. 2. Report to the clinic immediately. 3. Use a tweezers to try to remove the insect. 4. Use a flashlight to coax the insect out of the ear.

4. Use a flashlight to coax the insect out of the ear. Insects that make their way into an ear often can be coaxed out using a flashlight or a humming noise. If this is unsuccessful, then the insect must be killed before removal. Mineral oil or diluted alcohol is instilled into the ear to suffocate the insect, which is then removed by means of an ear forceps. The mother should be instructed not to irrigate the ear or attempt to remove the insect by using tweezers because this could damage the ear. If the mother is unsuccessful in coaxing the insect out of the ear, she should be instructed to report to the clinic or the hospital emergency department.

104.) Disulfiram (Antabuse) is prescribed for a client who is seen in the psychiatric health care clinic. The nurse is collecting data on the client and is providing instructions regarding the use of this medication. Which is most important for the nurse to determine before administration of this medication? 1. A history of hyperthyroidism 2. A history of diabetes insipidus 3. When the last full meal was consumed 4. When the last alcoholic drink was consumed

4. When the last alcoholic drink was consumed Rationale: Disulfiram is used as an adjunct treatment for selected clients with chronic alcoholism who want to remain in a state of enforced sobriety. Clients must abstain from alcohol intake for at least 12 hours before the initial dose of the medication is administered. The most important data are to determine when the last alcoholic drink was consumed. The medication is used with caution in clients with diabetes mellitus, hypothyroidism, epilepsy, cerebral damage, nephritis, and hepatic disease. It is also contraindicated in severe heart disease, psychosis, or hypersensitivity related to the medication.

a child with type 1 diabetes is brought to the ED by the mother, who states the child has been complaining of abdominal pain and has a fruity odor of the breath. diabetic ketoacidosis is dx. anticipating the plan of care, the nurse prepares to administer: 1. potassium iv infusion 2. NPH insulin iv infusion 3. 5% dextrose iv infusion 4. normal saline iv infusion

4. normal saline iv infusion hyperglycemia occurs with diabetic ketoacidosis. rehydration is the initial step. normal saline is the initial iv rehydration fluid.

Amphetamines

A class of stimulant drugs that includes amphetamine sulfate and all of its drug derivatives. Various amphetamine salts are the prototypical CNS stimulants used to treat ADHD and narcolepsy. Available in prescription form only for oral use, both as single component dextroamphetamine sulfate (Dexedrine) and as a mixture of dextroamphetamine sulfate, destroamphetamine sccharate, amphetamine sulfate, and amphetamine aspartate.

Sleep

A transient, reversible, and periodic state of rest in which there is a decrease in physical activity and consciousness.

Convulsion

A type of seizure involving excessive stimulation of neurons in the brain characterized by the spasmodic contraction of voluntary muscles.

Penicillin is ineffective for the treatment of primary atypical pneumonia because the causative organism (Mycoplasma pneumoniae):

has no peptidoglycan layer. Penicillins (and cephalosporins) kill bacteria by altering the peptidogylcan layer in the cell wall. Mycoplasma pneumoniae will not respond to penicillin because the organism does not have a cell wall.

Oxcarbazepine (Trileptal)

Chemical analogue of carbamazepine. Mechanism of action has not been identified, through it is known to block voltage-sensitive sodium channels, which aids in stabilizing excited neuronal membranes. Indicated for partial seizures and secondarily generalized seizures. Contraindications include known drug allergy. Common adverse reactions include headache, dizziness, and nausea. Not a hepatic enzyme inducer, so it has far fewer common drug interactions.

A patient diagnosed with type 2 diabetes mellitus is treated for HTN with propanolol. Hx reveals that the patient is dx with glaucoma and is allergic to sulfa. The nurse is MOST concerned if an order was written for which of the following medications? 1. Glycerin (Osmoglyn). 2. Pilocarpine (Isopto-Carpine). 3. Acetazolamide (Diamox). 4. Timolol maleate (Timoptic).

Acetazolamide (Diamox) contraindicated; cross-sensitivity can occur due to allergy to antibacterial sulfonamides and sulfonamide derivatives

Contraindications to the use of Fludrocortisone

Administer w/ caution in the presence of these conditions: HTN HF Cardiac disease renal function impairment (except when the drugs is used to treat type IV renal tubular acidosis)

Bismuth

Adsorbant Use in caution with children, chickenpox/flu Avoid Aspirin (potentiates effects)

Antidiarrheal drug that acts by coating the walls of the GI tract and binding to causative bacteria/toxin to allow elimination

Adsorbents: activated charcoal, bismuth

How to counteract the side effects of Niacin?

Advise pt to take Asa or NSAID 30 mins prior

Drug Interactions of Opioids

Alcohol, antihistamines, barbiturates, benzodiazepines, phenothiazine, and other central nervous system depressants can result in additive respiratory-depressant effects. Monoamine oxidase inhibitors can result in respiratory depression and hypotension.

Potassium Sparing Diuretic

Aldalactone (spironolactone)

Drug interactions with Opioids

Mixing opioids with: alcohol antihistamines barbiturates benzodiazepines phenothiazine other CNS depressants can result in additive respiratory depressant effects. Mixing with monamine oxidase inhibitors, such as selegiline, can result in respiratory depression, seizures, and hypotension.

Patient Teaching for AEDs

Avoid tasks requiring alertness until a steady state of the drug has been achieved (which takes 4 to 5 half-lives) to help prevent injury. ⎫ Avoid drinking alcohol and smoking. ⎫ Do not abruptly withdraw from an AED; rebound seizure activity could occur. ⎫ Phenytoin may turn urine to pink or red-brown. This color change commonly diminishes over time. ⎫ Drowsiness commonly decreases after several weeks because tolerance to this particular adverse effect occurs. ⎫ Frequent oral care and dental visits are necessary so as to prevent the adverse effect of gingival hyperplasia. ⎫ Avoid any form of stimulant (e.g., caffeine) because there is a higher risk for seizure. ⎫ Therapy is usually lifelong. Resources available include national and local support groups. ⎫ Contact the physician if any unusual reactions occur, such as glandular swelling, fever, sore throat, tarry stools, back pain, hematuria, easy bruising, lethargy, or mouth ulcers.

Mechanism of action and effects of Tricyclic Antidepressants

Believed to work by correcting imbalance in the neurotransmitter concentrations of serotonin and norepinephrine at the nerve endings in the CNS (the biogenic amine hypothesis). This is accomplished by blocking the presynaptic reuptake of the neurotransmitters, which makes them available for transmission of nerve impulses to adjacent neurons in the brain. Some also believe they may help regulate malfunctioning neurons.

Agonist

Binds to a receptor and causes a response

The nurse is providing information to a client about a prescribed medication. Which one of these statements, if made by the client, indicates that teaching about PROPANOLOL (Inderal) has been effective? A. "I could have an increase in my heart rate for a few weeks" B. "I will expect to feel nervouseness the first few weeks" C. "I may experience seizures if I stop the medication abruptly" D. "I can have a heart attack if I stop this medication suddenly"

D. "I can have a heart attack if I stop this medication suddenly"

ACE Inhibitors Contraindications

Drug allergy, hyperkalemia, bilateral renal stenosis, pregnancy, and lactation

Contraindications of Opioids

Drug allergy, severe asthma or other respiratory insufficiency, increased intracranial pressure, myasthenia gravis, paralytic ileus, acute abdominal conditions, and pregnancy and lactation

Antagonist

Drug binds to a receptor to prevent a response

REM Interference

Drug-induced reduction of REM sleep time.

Sedative-hypnotics

Drugs that can act in the body either as sedatives or as hypnotics.

Nonopioid Analgesic Assessment

Focus the assessment not only on general data, but also on the specific drug being given.

Toxicity and management of BB OD

For ODs of both injectable and oral dosage forms, treatment consists primarily of symptomatic and supportive care. Atropine may be given IV for the management of bradycardia. For the treatment of severe HTN, pressers are titrated until the desired BP and HR are achieved. IV admin diazepam may be useful for treatment of seizures. hemodialysis may be useful in enhancing elimination in the event of severe OD.

Treatment for chronic pain

Gabapentin, SNRIs, TCAs

Deep Pain

Occurs in tissues below skin level.

Thiazide Diuretics

Inhibit water/sodium reabsorption (HCTZ)

Interactions with benzodiazepines

Potential interactions with the benzodiazepines are significant bc of their intensity, particularly when they involve other CNS depressants (alcohol, opioids, muscle relaxants), which result in further CNS depressant effects including: reduced BP reduced reap rate sedation confusion diminished reflexes Herbal remedies that interact include kava and valerian, which may also lead to further CNS depression. Food interactions include grapefruit and juice.

Contraindications of AEDs

Pregnancy and Drug Allergy

Antiepileptic drugs

Prescription drugs that prevent or reduce the severity of epilepsy and different types of epileptic seizures, not just convulsive seizures. Many anti epileptic drugs are also used to treat other types of illnesses, including psychiatric disorders, migraines, and neuropathic pain syndromes.

Aldosterone

Regulates sodium and other electrolytes and fluid in the body.

Pentobarbital (Nembutal)

Short-acting barbiturate. Used for anxiety relief and sedation. Used occasionally to control status epileptics or acute seizure episodes resulting from meningitis, poisons, eclampsia, alcohol withdrawal, tetanus, and chorea. May also be used to treat withdrawal in patients who are physically dependent on barbiturates or non barbiturate hypnotics. Available in oral, injectable, and rectal dosage forms.

a 5 y/o is prescribed SULFAMETHOXAZOLE/ TRIMETHOPRIM for the tx of otitis media. Which severe adverse reaction is most common in drugs containing SULF?

Steven-johnson syndrome is a rare hypersensitivity reaction often caused by antibiotics, most frequently caused by sulfa containing drugs, such as sulfamethoxazole, a sulfamide

Mechanism of action and effects of analeptics

Stimulate areas of the CNS that control respiration, mainly the medulla and spinal cord. Methylxanthine analeptics also inhibit the enzyme phsophodiesterase. This enzyme breaks down a substance called cyclic adenosine monophosphate (cAMP); when analeptics block this enzyme, cAMP accumulates.

Indirect-acting cholinergic agonists

Stimulate the postsynaptic release of acetylcholine at the receptor site. This then allows acetylcholine to bind to and stimulate the receptor. -work by inhibiting the action of acetylcholinesterase. donepezil echothiophate endrophonium galantamine neostigmine physostigmine pyridostigmine rivastigmine tacrine

A patient has been taking phenobarbital for 2 weeks as part of his therapy for epilepsy. He says that he feels tense and that the "least little thing" bother him now. The nurse should explain to him which of the following:

This drug causes deprivation of REM sleep and may cause the inability to cope with normal stress

Orlistat (Xenical)

Unrelated to other drugs in its category. Works by bnding to gastric and pancreatic enzymes called lipase's. Blocking these enzymes reduces fat absorption by roughly 30%. Adverse effects include oily spotting, flatulence, fecal incontinence.

Caffeine Sodium Benzoate

Used for respiratory depression in adults only bc it contains the preservative benzyl alcohol.

Analeptics

Used for specific respiratory depression syndromes Doxapram (Dopram) Methylxantines aminophylline, theophylline, caffeine

Anxiolytics

Used to reduce the intensity of feelings of anxiety.

medication: benzodiazepine

antidote: flumazenil

medication: methotrexate

antidote: folinic acid

medication: morphine

antidote: naloxone

medication: demerol

antidote: naloxone hydrochloride, naltrexone

medication: atropine sulfate

antidote: physostigmine

medication: coumadin or warfarin

antidote: phytonadione (vitamin K)

medication: heparin

antidote: protamine sulfate

medication: lovenox

antidote: protamine sulfate

medication: lithium

antidote: sodium bicarbonate

Two classes of steroids secreted by adrenal cortex

- Corticosteroids (glucocorticoids and mineralocorticoids) - Androgens Hydrocortisone (cortisol) is the primary glucocorticoid, and aldosterone is the main mineralocorticoid.

quinapril

ACE inhibitor; avoid foods high in potassium for example: broccoli, bananas, and salt substitutes

Somatic Pain

Muscle or tissues

Serotonin Receptor Agonists

Used to treat migraines (sumatriptan: Imitrex)

Lidocaine is used for

Ventricular dysrhythmias

What labs must be monitored with a pt newly taking an ACE Inhibitor

potassium (hyperkalemia)

Warfarin is used for

prevention

SSRI Patient Teaching

⎫ Possible adverse effects include gastrointestinal upset, sexual dysfunction, and weight gain. ⎫ Use caution when driving and performing other activities requiring mental alertness until tolerance to the adverse effect develops. ⎫ It is important to take SSRIs as ordered and not to discontinue them abruptly, because of possible discontinuation syndrome (dizziness, diarrhea, movement disorders, insomnia, irritability, visual disturbances, lethargy, anorexia, and lowered mood). ⎫ Notify your health care provider immediately if you experience diarrhea, nausea, abdominal cramping, mental status changes, restlessness, diaphoresis, shivering, tremors, ataxia, headaches, or hyperreflexia because they may signify serotonin syndrome. ⎫ Check with your health care provider before using any over-the-counter medications.

Calcium Channel Blockers

antidysrhythmic, antianginal, antihypertensive *Norvasc decreased contracility = decreased workload of heart

prochlorperazine maleate

antiemetic

chlorpromazine

antiemetic, antipsychotic

miconazole

antifungal, tx yeast infection

atenolol

antihypertensive, antianginals

clonidine patch

antihypertensive; change the patch every 7 days, apply to non-hairy site

Malignant Hyperthermia

- A major concern w/ general anesthesia. - May be fatal if not promptly recognized and treated. - S/S: rapid rise in body temp, tachycardia, tachypnea, muscle rigidity, and unstable BP.

Mechanism of Action of Agonist-Antagonists

- Agonist-antogonists (partial agonist or mixed agonist): Binds to a pain receptor and causes a weaker pain response than does a full agonist. Not usually a first-line analgesic, can be useful in pain management in opioid -addicted patients as well as obstetrical patients.

Mechanism of Action of Agonists

- Agonists: Binds to an opioid pain receptors in the brain and causes an analgesic response, reduction of pain sensation.

Adverse Effects of opioid analgesics

- All opioids have strong abuse potential. - Histamine release causes: pruritus, rash, and hemodynamic changes, flushing, orthostatic hypotension. - Most serious is CNS depression, which may lead to respiratory depression. - GI effects are common due to stimulation of GI opioid receptors; N/V and constipation are most common. - Urinary retention

Psychotropic Drugs

- Among the most commonly prescribed drugs in the US. - Effects are less easily quantified than many other types of meds. - There is usually no known duration of action. - Common problem is non adherence.

Mechanism of Action of Antagonists

- Antagonists (nonanalgesic):Binds to a pain receptor but does not reduce pain signals. Functions as a competitive antagonist bc it competes with and reverses the effects of agonist and agonist-antagonist drugs at receptor sites.

atomoxetine (Strattera)

- Approved for treating ADHD in children >6 and in adults. - Lacks addictive properties, unlike amphetamines and phenidates. - May cause suicidal thought and behaviors

doxorubicin hydrochloride

antineoplastic assess for fever; causes bone marrow depression

Drugs for ADHD and Narcolepsy

- CNS stimulants are the first-line drugs of choice for both ADHD and narcolepsy. They are potent drugs with a strong potential for tolerance and psychological dependance. - In general, CNS stimulants elevate mood, produce a sense of increased energy and alertness, decrease appetite, and enhance task performance impaired by fatigue or boredom.

Thymolytic, Lympholytic, and Eosinopenic Actions of Glucocorticoids

- Can cause atrophy of the thymus and decrease number of lymphocytes, plasma cells, and eosinophils in the blood. By blocking the production and release of cytokines, coritocsteroids interfere with the integrated role of T and B lymphocytes, macrophages, and monocytes in the immune response and thus ultimately interfere with immune and allergic responses. This response, along with their anti-inflammatory action, makes them useful immunosuppressants for delaying rejection in clients with organ or tissue transplants, and useful antiallergenics for treating acute allergic reations such as urticaria, bronchial asthma, and anaphylactic shock infections.

Possible Effects of popular herbal products when combined with anesthetics

- Feverfew: migraines, insomnia, anxiety, joint stiffness, risk of increased bleeding times - Garlic: Changes in BP, risk of increased bleeding - Ginger: Sedating effects, risk of bleeding esp. if combined with aspirin or gingko - Ginseng: Irritability and insomnia, risk of cardio adverse effects Kava: Sedating effects, potential liver fox, risk of additive effects with meds St. John's Wort: Sedation, BP changes, risk of interactions with other meds that prolong the effects of anesthesia.

Major Side Effects of Diuretics

- GI irritation - Hyponatremia (inhibition of sodium reabsorption at the kidney tubule) - Orthostatic hypotension - Hyperuricemia (partial blockage of uric acid excretion) - Dehydration (excessive sodium and water loss) - All diuretics except potassium-sparers: hypokalemia, increased urinary excretion of mag and zinc - Potassium-sparers: hyperkalemia, hypomagnesemia, increased urinary excretion of calcium - Furosemide competes with aspirin for renal excretion sites and can cause aspirin tox - Thiazides and loop diuretics may cause hyperglycemia in diabetic clients

Pharmacokinetics/Dosing of Fludrocortisone

- Good oral absorption. - Half-life is approx. 3.5 hours in the plasma, and a biologic half-life of activity in the body of 18-36 hours. - Duration of action is 24-48 hours. - Metabolized in the liver and kidneys and excreted by the kidneys. - Oral dosage for adults and adolescents is 0.1 mg/day. - Usual dosage for children is 50 to 100 mcg/day

Treatment for autoimmune disorders consists of:

- Immunosuppression and control of clinical manifestations, primarily through systemic corticosteroid therapy. - Initial doses of 60 mg of oral prednisone often bring about a noticeable decrease in symptoms. Once symptoms are controlled, dosages are slowly decreased and then discontinued until the next exacerbation. - Cytotoxic drugs such as cyclophosphamide, azathioprine, and methotrexate are sometimes prescribed for patients with severe or persistent manifestations.

modafinil (Provigil)

- Indicated for improvement of wakefulness in patients with excessive daytime sleepiness associated with narcolepsy and also with shift work sleep disorder. - Less abuse potential than amphetamines and methylphenidate and is a Sched. IV drug. - R/t armodafinil (Nuvigil)

Diazepam (Valium) (anxiolytic)

- Indicated for the relief of anxiety, management of alcohol withdrawal, reversal of status epileptics, preop sedation, and less frequently as adjunct for relief of skeletal muscle spasms - Has active metabolites that can accumulate in patients w/ hepatic dysfunction bc it is primarily metabolized in the liver. Can result in additive, cumulative effects that may be manifested as prolonged sedation, resp depression, or coma - Contraindicated in patients w/ major hepatic compromise -Adverse effects include headache, confusion, slurred speech - Interacts w/ alcohol, oral contraceptives

Diuretics

- Interfere with sodium reabsorption in kidney - Increase urine output, which reduces hypervolemia; decreases preload and after load - Available in oral and parenteral preparations

Lorazepam (Ativan)

- Intermediate-acting benzodiazepine - Avail. oral and injectable; may be given IV or IM. - Has excellent absorption and bioavailability when given IM, but is irritating to muscles and must be diluted - Indications, contraindications, and adverse effects are similar to those of alprazolam

Contraindications to the use of amphetamine and nonamphetamine stimulants

- Known drug allergy - Cardiac structure abnormalities These drugs can also exacerbate the following conditions: - Marked anxiety - Agitation - Tourette's syndrome - Other tic disorders (hyper stimulation) - HTN - Glaucome (can increase intraocular pressure) **Must not be used in patients who have received therapy w/ any monoamine oxidase inhibitor in the preceding 14 days. Contraindications specific to atomoxetine: - drug alley - glaucoma - recent MAOI use

Contraindications for benzodiazepine use

- Known drug allergy - Narrow-angle glaucoma - Pregnancy

Indications for Opioid Use

- Main use of opioids is to alleviate moderate to severe pain. - Opioids also suppress the medullary cough center, which results in cough suppression. Most commonly used for this purpose is codeine.

Nursing Care

- Maintain intake and output; daily weights, assess for signs of fluid-electrolyte imbalance - Administer the drug in the morning so that the max effect occurs during waking hours - Monitor pulse and BP, instruct to change positions slowly - Encourage intake of foods high in calcium, mag, zinc, and potassium (except for potassium-sparers)

Feverfew

- Marigold family; known for anti-inflammatory properties. - Treatment of migraines, menstrual cramps, inflammation, and fever. - N/V, constipation, diarrhea, altered tast, muscle stiffness, joint pain. - Possible increase in bleeding with the use of aspirin and other NSAIDs, dipyridamole, and warfarin. - Contraindicated in those allergic to ragweed, chrysanthemums, and marigolds, as well as those about to undergo surgery.

Potassium-sparing diuretics: Block the effect of aldosterone on renal tubules

- Medications: spironolactone (Aldactone) (PO), Triamterene (Dyrenium) (PO) - Side effects: Menstrual irregularities, impotence, hypotension - Nursing implications: May be used in combo with other diuretics to reduce potassium loss; potassium-sparing effects may result in hyperkalemia; NOT used for clients experiencing renal failure.

fluphenazine

antipsychotic

Attention deficit hyperactivity disorder

- Most common psychiatric disorder in children, affecting 3% to 10% of school age children. Boys are affected two to nine times more often than girls. Primary symptoms: Developmentally inappropriate ability to maintain attention span and/or the presence of hyperactivity and impulsivity.

Alprazolam (Xanax)

- Most commonly used as anxiolytic - Indicated specifically for panic disorder - Adverse effects include confusion, ataxia, headache - Interacting drugs include alcohol, antacids. oral contraceptives

Methylphenidate (Ritalin)

- Most widely prescribed drug for ADHD, also used for narcolepsy. - Extended release dosage forms include Ritalin SR, Concerta, and Metadate CD.

Cortisol

- Multiple functions in the body, including carbohydrate metabolism. - Secreted in times of physiological stress.

Toxicity and management of TCA OD

- NOTORIOUSLY LETHAL - Primary organ systems affected are the CNS and cardiovascular system. - Death usually results from either seizures or dysrhythmias. - No specific antidote for TCA poisoning. Management is aimed at reducing drug absorption by administering multiple doses of activated charcoal. Admin of sodium bicarb speeds up elimination of TCA by alkalization of the urine. CNS damage may be minimized by the admin of diazepam, and cardiovascular events may be minimized by giving antidysrhythmics. Other care includes basic life support in the IC setting to maintain organ function.

Actions of adrenergic drugs

- Often resemble or mimic the actions of the sympathetic nervous system. For this reason, adrenergic drugs (and to a lesser degree, dopaminergic drugs as well) are also called sympathomimetic drugs.

Amitriptyline (Elavil)

- Oldest and most widely used TCA - Originally indicated for depression, now more commonly used to treat insomnia and neuropathic pain. - Contraindications include known drug allergy, pregnancy, and recent MI. Has very potent anticholinergic properties, which can lead to many adverse effects such as dry mouth, constipation, blurred vision, urinary retention, and dysrhythmias.

Withdrawal Symptoms

- Onset of symptoms is directly related to half-life of opioid being used. - Symptoms resulting from dc or reversal of therapy w/ short-acting opioids will appear within 6-12 hours and peak at 24 to 72 hours. - Symptoms associated with the long half-life drugs may not appear for 24 hours or longer after dc, and may be milder.

Anxiolytic Drugs

- Primary anxiolytics include the benzodiazepine drugs class and the miscellaneous drug buspirone. - Benzos are commonly used as first-line drugs for both acute and chronic anxiety disorders. - Other drugs that are as effective as anxiolytics are: Selective Serotonin Reuptake Inhibitors Tricyclic Antidepressants Monoamine Oxidase Inhibitors Antipsychotics Antihistamine hydroxyzine

Indications for use of benzodiazepines

- Sedation - Relief of agitation or anxiety - Treatment of anxiety-related depression - Sleep induction - Skeletal muscle relaxation - Treatment of acute seizure disorders Often used in balanced anesthesia, and also moderate sedation for their amnesic properties to reduce memory of painful procedures. Bc benzodiazepine receptors in the CNS are in the same area as those that play a role in alcohol addiction, some are used in the treatment and prevention of the symptoms of alcohol withdrawal.

Adverse Effects of Fludrocortisone

- Severe/persistant headaches - HTN - Dizziness - Edema of the lower extremities - Joint pain - Hypokalemia - Increased weakness - Tingling/numbness in legs that may progress to arms, trunk, or face - HF - Anaphylaxis All of these should be reported immediately.

Buspirone

- Should not be coadministered with MAOIs due to risk of HTN. Washout period of at least 14 days must be allowed between MAOIs and buspirone.

Potassium-sparing Diuretics

- Spironolactone, triamterine, amiloride - Interfere with aldosterone-induced reabsorption of sodium ions at distal nephron sites to increase sodium chloride excretion and decrease potassium ion loss.

Toxicity and management of OD of antiolytics

- Taken alone, benzodiazpine OD is generally not life threatening. When combined with alcohol or other CNS depressants, the outcome is much more severe. OD may result in the following: - somnolence - confusion - coma - respiratory depression Flumazenil is used to reverse the effects of benzos.

Mechanism of action and effects of adrenergic-blocking drugs

-Alpha-adrenergic-blocking drugs, or alpha blockers, interrupt stimulation of the SNS at the alpha1-adrenergic receptors. More specifically, alpha blockers work either by direct competition w/ norepinephrine or by a noncompetitive process. -Alpha blockers have a greater affinity of the alpha-adrenergic receptor than norepinephrine does and therefore can chemically displace norepinephrine molecules from the receptor. -Adrenergic blockade at these receptors leads to effects such as vasodilation, reduced BP, miosis (pupillary dilation), and reduced smooth muscle tone in organs such as the bladder and prostate.

Indications for cholinergic blockers

-Also has ophthalmic uses. -When cholinergic stimulation of PNS is blocked by cholinergic blockers, SNS effects go unopposed. -In the resp tract, results in decreased secretions from the nose, mouth, pharynx, and bronchi. -Causes relaxation of smooth muscles in the bronchi and bronchioles, BENEFICIAL IN TREATING ASTHMAS, CHRONIC BRONCHITIS, COPD. -PRE OP TO REDUCE SALIVATION. - CAUSE REDUCED SECRETIONS, RELAXATION OF SMOOTH MUSCLE, REDUCED GI MOTILITY AND PERISTALSIS; USED IN TREATMENT OF IBD AND GI HYPERSECRETORY STATES. -TREATMENT OF SUCH GU TACT DISORDERS AS REFLEX NEUROGENIC BLADDER AND INCONTINENCE. USEFUL FOR REDUCING GASTRIC AND PANCREATIC SECRETIONS IN PATIENTS W/ ACUTE PANCREATITIS.

Indications for BBs

-Angina:decrease demand for myocardial energy and oxygen consumption, which allows more oxygen to get to the heart muscle. -MI:inhibit stimulation of the myocardium by circulating catecholamines; MI causes catecholamine release. -Cardiac dysrhythmias:conduction through SA and AV nodes is slowed, reducing HR. -HTN:reduce SNS stimulation of the heart, including reducing HR and force of myocardial contraction. -HF:certain BBs such as carvedilol and metoprolol work best. HF w/ diastolic dysfunction component responds best to BBs. -Migraines: Bc lipophilicity, some BBs (such as proprianolol) can easily gain entry into the CNS. -Glaucoma/other ocular disorders:topical application of timolol.

Mechanism of action and effects of beta blockers

-Block SNS stimulation of the beta-adrenergic receptors by competing w/ norepi and epi. BBs can be selective or nonselective, depending on the type of receptors they antagonize. Beta1-adrenergic receptors are located primarily in the heart. BBs that are selective for these receptors are called cardioselective BBs or beta1-blocking drugs. -Other BBs block both beta1 and beta2 and are referred to as nonselective BBs. Beta2 receptors are located primarily on the smooth muscles of the bronchioles and blood vessels.

Mechanism of action of cholinergic-blocking drugs

-Block the action of the neurotransmitter acetylcholine at the muscarinic receptors in the parasympathetic nervous system. -Acetylcholine that is released from a stimulated nerve fiber is then unable to bind to the receptor site and fails to produce a cholinergic effect. -Blocking the parasympathetic nerves allows the sympathetic nervous system to dominate. Because of this, cholinergic blockers have many of the same effects of adrenergics. -Competitive antagonists. They compete w/ acetylcholine for binding at the muscarinic receptors of the parasympathetic nervous system. Once they have bound to the receptor, they inhibit cholinergic nerve transmission. -Have little effect at the nicotinic receptors, although at high doses they can have partial blocking effects.

Cardioselective beta1-blockers

-Block the beta1 receptors on the surface of the heart. This reduces myocardial stimulation, which in turn reduced HR, slows conduction through the AV node, prolongs SA node recovery, and decreases myocardial oxygen demand by decreasing myocardial contractile force (contractility). -Nonselective BBs not only have these cardiac effects, but they block beta2 receptors on the smooth muscle of the bronchioles and blood vessels as well.

Adverse effects of anticholinergics

-Cardiovascular: Increased HR, dysrhythmias (tachy, palps) -CNS: Excitation, restlessness, irritability, disorientation, hallucinations, delirium, ataxia, drowsiness, sedation, confusion -Eye: Dilated pupils, increased intraocular pressure -GI: Decreased salivation, gastric secretions, motility (causing constipation) -Genitourinary: Urinary retention -Glandular: Decreased sweating -Respiratory: Decreased bronchial secretions

Donepezil (Aricept)

-Cholinesterase inhibitor that works centrally in the brain to increase levels of acetylcholine by inhibiting acetylcholinesterase. -Used in the treatment of mild to moderate alzheimer's. -Similar cholinesterase inhibitors include: tacrine, galantine, and rivastigmine. Rivastigmine is also approved for treating dementia associated w/ Parkinson's. -Contraindications: allergy -Adverse effects normally mild and resolve on their own. Can often be avoided by careful dose titration. Include: GI upset, drowsiness, dizziness, insomnia, muscle cramps. -Cardiovascular effects: bradycardia, syncope, hypotension w/ reflex tachycardia, or HTN. -Interacting drugs include: anticholinergics (counteract donepezil effects) and NSAIDS.

Toxicity and management of MAOI OD

-Clinical symptoms of MAOI OD generally do not appear until about 12 hours after ingestion. -Primary S/S are cardiovascular and neurologic. -Most serious cardiovascular effects are tachycardia and circulatory collapse, and the neuro symptoms of greatest concern are seizures and coma. -Hyperthermia and mitosis - Treatment is aimed at eliminating the ingested toxin and protecting the organs at greatest risk for damage-the brain and the heart. -Recommended treatments are urine acidification to a pH of 5, and hemodialysis. -Treatment of hypertensive crisis resulting from consumption of tyramine-containing foods may require IV admin of hypertensive drugs and careful monitoring in ICU.

Indications for 2nd gen antidepressant use

-Depression bipolar obesity eating disorders OCD panic attacks or disorders social anxiety disorder PTSD premenstrual dysphoric disorder neuro disorder myoclonus various substance abuse probs such as alcoholism

bethanechol (Urecholine)

-Direct-acting cholinergic agonist. -Used in the treatment of acute post op and postpartum non obstructive ordinary retention and for the management of urinary retention associate with neurogenic atony of the bladder. -Has also been used to prevent and treat bladder dysfunction induced by phenothiazine and tricyclic antidepressants. -In addition, it is used in the treatment of post op GI atony and gastric retention, chronic refractory heartburn, as well as in diagnostic testing for infantile cystic fibrosis. -Contraindications include allergy hyperthyroidism, peptic ulcer, active bronchial asthma, cardiac disease or CAD, epilepsy, and Parkinsonism. Should be avoided in patients in whom the strength or integrity of GI tract or bladder wall is questionable or with conditions in which increase muscular activity could prove harmful, such as known or suspected mechanical obstruction. -Adverse effects include syncope, hypotension w/ reflex tachycardia, headache, seizure, GI upset, and asthmatic attacks. -Drugs that interact include acetylcholinesterase inhibitors, which can enhance the adverse effects of bethanechol.

Toxicity and management of anticholinergic OD

-Dosage is important, bc low therapeutic index. -Treatment of OD consists of symptomatic and supportive therapy -Patient should be hospitalized, w/ continuous monitoring, including ECG. Activated charcoal has proven effective. -Fluid therapy etc. PRN - Delirium, hallucinations, coma, cardiac dysrhythmias respond favorably to physostigmine.

Atropine

-Effective in the treatment of many conditions. Causes increased HR, treats bradycardia and ventricular systole. -Antidote for ACE-inhibitor tox or poison -Preop to reduce salivation and GI secretions, as is glycopyrrolate. -Contraindicated in angle-closure glaucoma, adhesions between iris and lens, certain types of asthma, advanced hepatic and renal dysfunction, hiatal hernia, intestinal atony, obstructive GI or GU conditions, severe ulcerative colitis. Use caution in dysrhythmic patients. -OD is associated w/ flushing, dry skin and mucous membranes, mydriasis, altered mental status, fever. -Adverse effects: sinus tachy, urinary retention, HTN, hallucinations, resp depression, cardiovascular collapse. -Use activated charcoal, physostigmine.

Epinephrine (Adrenalin)

-Endogenous vasoactive catecholamine. -Acts directly on both of the alpha- and beta-adrenergic receptors of tissues innervated by the SNS. Considered the prototypical nonselective adrenergic agonist. -Administered in emergency situations and is one of the primary vasoactive drugs used in many advanced cardiac life support protocols. -Physiological response is dose related. -At low dosages, stimulates mostly beta1-adrenergic receptors, increasing the force of contraction and HR. -Also used to treat acute asthma and anaphylactic shock at these dosages bc it has significant bronchodilatory effects via the beta2-adrenergic receptors in the lungs. -At high dosages, stimulates mostly alpha-adrenergic receptors, causing vasoconstriction, which elevates the BP.

Indications for cholinergic blockers

-In the CNS, cholinergic blockers have the therapeutic effect of decreasing muscle rigidity and diminishing tremors. This is of benefit in the treatment of both Parkinson's disease and drug-induced extrapyramidal reactions such as those associated w/ antipsychotic drugs. These conditions involve dysfunction of the extrapyramidal parts of the brain and include motor dysfunctions such as chorea, dystonia, and dyskinesia. -Cardiovascular effects of anticholinergics are related to their cholinergic-blocking actions on the heart's conduction system. -At low dosages, the anticholinergics may actually slow HR through their effects on the cardiac center in the medulla. At high dosages, cholinergic blockers block the inhibitory vagal effects on the pacemaker cells of the SA and AV nodes, which leads to acceleration of the HR due to unopposed sympathetic activity.

Effects of anticholinergics

-Major sites of action of the anticholinergics are the heart, resp tract, GI tract, urinary bladder, eye, and exocrine glands. -Anticholinergic effects on the cardiovascular system are seen as an increase in HR. -Resp system effects are dry mucous membranes and bronchial dilation. -In the GI tract, cholinergic blockers cause a decrease in GI motility, GI secretions, and salivation. -In the GU system, they lead to decreased bladder contraction, which can result in urinary retention. -In the skin, they reduce sweating -Anticholinergics cause the pupils to dilate and increase intraocular pressure.

Interactions w/ alpha blockers

-Most severe drug interactions w/ alpha blockers are the ones that potentiate the effects of the alpha blockers. Alpha blockers are highly protein bound; bc of the limited sites for binding on proteins and the increased competition for these sites, more free alpha blocker molecules circulate in the blood steam. More active drug results in a more pronounced drug effect. -Phentolamine: BBs, alcohol, erectile dysfunction drugs, epinephrine -Tamsulosin: warfarin, antiHTN, erectile dysfunction drugs, alcohol

Dopamine (Intropin)

-Naturally occurring catecholamine neurotransmitter -Potent dopaminergic as well as beta1- and alpha1-adrenergic receptor activity, depending on the dosage. -Dopamine, when used at low dosages, can dilate blood vessels in the brain, heart, kidneys, and mesentery, which increases blood flow to these areas. -At higher infusion rates, dopamine can improve cardiac contractility and output. - At highest doses, dopamine causes vasoconstriction. -Contraindicated in patients who have a catecholamine-secreting tumor of the adrenal gland known as a pheocromocytoma.

Indications for adrenergic-blocker use

-Other alpha blockers are effective at counteracting the effects of injected epinephrine and norepinephrine. They do this by CAUSING PERIPHERAL VASODILATION AND REDUCING PERIPHERAL RESISTANCE BY BLOCKING CATECHOLAMINE-STIMULATED VASOCONSTRICTION. Bc of their potent vasodilating properties and their fast onset of action, they are used to prevent skin necrosis and sloughing after the extravasation of vasopressors such as norepinephrine or epinephrine. When extravasation occurs, they cause vasoconstriction and ultimately tissue death. If not reversed quickly, entire limb can be lost. PHENTOLAMINE, IN PARTICULAR, CAN REVERSE THIS POTENT VASOCONSTRICTION AND RESTORE BLOOD FLOW TO THE ISCHEMIC TISSUE.

Indications for adrenergic-blocker use

-Other alpha blockers can inhibit responses to adrenergic stimulation. They non competitively block alpha-adrenergic receptors on smooth muscle and various exocrine glands. Bc of this action, they are very useful in controlling or preventing HTN in patients who have a pheochromocytoma. -Alpha blockers are also useful in the treatment of patients who have increased endogenous alpha-adrenergic agonist activity, which results in vasoconstriction. Three conditions in which this occurs are: Raynaud's disease Acrocyanosis Frostbite -Phenoxybenzamine, in particular, is an alpha blocker that is beneficial in the treatment of these syndromes, although its use is uncommon.

Mirtazapine (Remeron)

-Promotes the presynaptic release of both serotonin and norepinephrine in the brain -Indicated for depression, bipolar disorder, reducing sexual adverse effects in mails on SSRIs. -Contraindicated in cases of allergy, concurrent MAOI use. -Adverse effects include drowsiness, abnormal dreams, dry mouth, constipation, increased appetite, asthenia. -Drug interactions include CNS depressant effects with alcohol and CYP inhibitors.

Monoamine Oxidase Inhibitors (MAOIs)

-Rarely used as an antidepressant, mostly used for Parkinson's. -Serious disadvantage to use is their potential to cause a hypertensive crisis when taken with stimulant meds or with a substance containing tyramine, which is found in many foods and beverages. -Nonselective inhibitors of both MAO-A and MAO-B: isocarboxazid, phneelzine, and tranylcypromine. -Selective MAO-B inhibitor is selegiline; comes in a transdermal form. -Because these drugs inhibit the MAO enzyme system in the CNS, amines such as dopamine, serotonin, and norepinephrine are not broken down, and therefore higher levels of these substances occur, leading to alleviation of depression symptoms.

Selegiline transdermal patch (Emsam)

-Selective MAO-B inhibitor. -Currently indicated for major depression. -Contraindications include known drug allergy. -Patients should avoid exposing the patch to external sources of heat

Indications for adrenergic drug use

-Selectivity for either alpha- or beta-adrenergic receptors and their affinity for certain tissues or organs determine the settings in which they are most commonly used.

Nonselective beta2-blockers

-Smooth muscle that surrounds the airways in the lungs is called the bronchioles. When the beta2 receptors in the bronchioles are blocked, the end result is bronchial smooth muscle contraction and narrowing of the airways. This may lead to shortness of breath. In addition, the smooth muscle that surrounds blood vessels can cause dilation or constriction, depending on whether the beta1 or beta2 receptors are stimulated. -When this beta2 stimulation is blocked, the muscles are then stimulated by unopposed sympathetic activity at the beta1 receptors, which causes them to contract. This CAUSES INCREASED PERIPHERAL VASCULAR RESISTANCE. -Furthermore, catecholamines promote glycogenolysis (production of glucose from glycogen) and mobilize glucose in response to hypoglycemia. Nonselective BBs IMPAIR THIS PROCESS AND ALSO IMPEDE THE SECRETION OF INSULIN FROM THE PANCREAS, WHICH CAUSES ELEVATION OF BLODD GLUCOSE LEVEL. -Finally, beta blockers can cause the release of free fatty acids from adipose tissue. This may result in moderately elevated blood levels of triglycerides and reduced levels of HDLs.

bupropion

-Strongest therapeutic activity appears to be primarily dopaminergic and noradrenergic. -Indicated for depression, smoking cessation -Contraindicated in patients w/ allergy, seizure disorder, anorexia nervosa or bulimia, concurrent MAOI therapy. -Adverse effects include dizziness, confusion, tachycardia, agitation, tremors, and dry mouth. -Drugs that interact include azole antifungals, and CNS depressants.

Pyridostigmine (Mestinon)

-Synthetic quaternary ammonium compound that is similar in structure to other drugs in this class, including edrophonium, physostigmine, and neostigmine. All are indirect-acting cholinergic drugs that work to increase acetylcholine by inhibiting acetylcholinesterase. -Has been shown to improve muscle strength and is used to relieve the symptoms of myasthenia gravis, and it is the most commonly used drug for symptomatic treatment of myasthenia gravis. -Puridostigmine, neostigmine, and physostigmine are also useful for reversing the effects of non depolarizing neuromuscular blocking drugs after surgery. -Also used in the treatment of sever ID of tricyclic antidepressants bc of the significant anticholinergic effects associated with the tricyclic antidepressants. -Also as an antidote after toxic exposure to non drug anticholinergic agents, including those used in chemical warfare. -Contraindications: allergy, prior sever cholinergic reactions, asthma, gangrene, hyperthyroidism, cardiovascular disease, and mechanical obstruction of the GI or GU tracts. -Adverse effects: GI upset, excessive salivation. -Interacting drugs: anticholinergic drugs, which counteract the therapeutic effects of indirect-acting cholinergic drugs.

Mechanism of action and effects of cholinergic drugs

-When acetylcholine directly binds to its receptor, stimulation occurs. Once binding takes place on the membranes of an effector cell (cell of the target tissue or organ), the permeability of the cell changes, and the calcium and sodium are permitted to flow into the cell. This then depolarizes the cell membrane and stimulates the effector organ. -Effects of direct- and indirect-acting cholinergic drugs are seen when the parasympathetic nervous system is stimulated. -Cholinergic drugs are used primarily for their effects on the GI tract, bladder, and eye. These drugs stimulate the intestine and bladder, which results in increased gastric secretions, GI motility, and urinary frequency. They also stimulate constriction of the pupil. This helps decrease intraocular pressure. In addition, cholinergic drugs cause increased salivation and sweating. -Cardiovascular effects include reduced HR and vasodilation. Pulmonary effects include causing the bronchi of the lungs to constrict and the airways to narrow. At recommended dosages, cholinergic drugs primarily affect the muscarinic receptors, but at high dosages the nicotinic receptors can also be stimulated. The desired effects come from muscarinic receptor stimulation; many of the desirable adverse effects are due to nicotinic receptor stimulation.

Phentolamine (Regitine)

-alpha blocker that reduces peripheral vascular resistance and is also used to treat HTN. -Like phenoxybenzamine, it is used to treat the high BP caused by pheochromocytoma, but phentolamine can also be used in the diagnosis of this catecholamine-secreting tumor. -Most often used to treat the extravasation of vasoconstriction drugs such as norepi, epi, and dope, which when given IV can leak out of the vein esp if the IV tube is not correctly positioned. -Contraindicated in hypersensitivity, MI, and those with CAD. -Adverse effects include: tachycardia, dizziness, GI upset.

Indications for adrenergic-blocker use

-alpha blockers such as doxazosin, prazosin, and terazosin cause both arterial and venous dilation, which reduces peripheral vascular resistance and BP. USED TO TREAT HTN. -There are also alpha-adrenergic receptors in the prostate and bladder. By blocking stimulation of alpha1 receptors, these drugs reduce smooth muscle contraction of the bladder neck and the prostatic portion of the urethra. For this reason, ALPHA BLOCKERS ARE GIVEN TO PATIENTS WITH BPH TO DECREASE RESISTANCE TO URINARY OUTFLOW. This reduces urinary obstruction and relieves some of the effects of BPH. Tamsulosin and alfuzosin are used exclusively for treating BPH, whereas terazosin and doxazosin can be used for HTN and BPH.

173.) A nurse reviews the medication history of a client admitted to the hospital and notes that the client is taking leflunomide (Arava). During data collection, the nurse asks which question to determine medication effectiveness? 1. "Do you have any joint pain?" 2. "Are you having any diarrhea?" 3. "Do you have frequent headaches?" 4. "Are you experiencing heartburn?"

1. "Do you have any joint pain?" Rationale: Leflunomide is an immunosuppressive agent and has an anti-inflammatory action. The medication provides symptomatic relief of rheumatoid arthritis. Diarrhea can occur as a side effect of the medication. The other options are unrelated to medication effectiveness.

127.) The nurse provides medication instructions to an older hypertensive client who is taking 20 mg of lisinopril (Prinivil, Zestril) orally daily. The nurse evaluates the need for further teaching when the client states which of the following? 1. "I can skip a dose once a week." 2. "I need to change my position slowly." 3. "I take the pill after breakfast each day." 4. "If I get a bad headache, I should call my doctor immediately."

1. "I can skip a dose once a week." Rationale: Lisinopril is an antihypertensive angiotensin-converting enzyme (ACE) inhibitor. The usual dosage range is 20 to 40 mg per day. Adverse effects include headache, dizziness, fatigue, orthostatic hypotension, tachycardia, and angioedema. Specific client teaching points include taking one pill a day, not stopping the medication without consulting the health care provider (HCP), and monitoring for side effects and adverse reactions. The client should notify the HCP if side effects occur.

77.) Phenytoin (Dilantin), 100 mg orally three times daily, has been prescribed for a client for seizure control. The nurse reinforces instructions regarding the medication to the client. Which statement by the client indicates an understanding of the instructions? 1. "I will use a soft toothbrush to brush my teeth." 2. "It's all right to break the capsules to make it easier for me to swallow them." 3. "If I forget to take my medication, I can wait until the next dose and eliminate that dose." 4. "If my throat becomes sore, it's a normal effect of the medication and it's nothing to be concerned about."

1. "I will use a soft toothbrush to brush my teeth." Rationale: Phenytoin (Dilantin) is an anticonvulsant. Gingival hyperplasia, bleeding, swelling, and tenderness of the gums can occur with the use of this medication. The client needs to be taught good oral hygiene, gum massage, and the need for regular dentist visits. The client should not skip medication doses, because this could precipitate a seizure. Capsules should not be chewed or broken and they must be swallowed. The client needs to be instructed to report a sore throat, fever, glandular swelling, or any skin reaction, because this indicates hematological toxicity.

91.) Cyclobenzaprine (Flexeril) is prescribed for a client to treat muscle spasms, and the nurse is reviewing the client's record. Which of the following disorders, if noted in the client's record, would indicate a need to contact the health care provider regarding the administration of this medication? 1. Glaucoma 2. Emphysema 3. Hyperthyroidism 4. Diabetes mellitus

1. Glaucoma Rationale: Because this medication has anticholinergic effects, it should be used with caution in clients with a history of urinary retention, angle-closure glaucoma, and increased intraocular pressure. Cyclobenzaprine hydrochloride should be used only for short-term 2- to 3-week therapy.

113.) A nurse is reinforcing discharge instructions to a client receiving sulfisoxazole. Which of the following would be included in the plan of care for instructions? 1. Maintain a high fluid intake. 2. Discontinue the medication when feeling better. 3. If the urine turns dark brown, call the health care provider immediately. 4. Decrease the dosage when symptoms are improving to prevent an allergic response.

1. Maintain a high fluid intake. Rationale: Each dose of sulfisoxazole should be administered with a full glass of water, and the client should maintain a high fluid intake. The medication is more soluble in alkaline urine. The client should not be instructed to taper or discontinue the dose. Some forms of sulfisoxazole cause the urine to turn dark brown or red. This does not indicate the need to notify the health care provider.

235.) A tricyclic antidepressant is administered to a client daily. The nurse plans to monitor for the common side effects of the medication and includes which of the following in the plan of care? 1. Offer hard candy or gum periodically. 2. Offer a nutritious snack between meals. 3. Monitor the blood pressure every 2 hours. 4. Review the white blood cell (WBC) count results daily.

1. Offer hard candy or gum periodically. Rationale: Dry mouth is a common side effect of tricyclic antidepressants. Frequent mouth rinsing with water, sucking on hard candy, and chewing gum will alleviate this common side effect. It is not necessary to monitor the blood pressure every 2 hours. In addition, it is not necessary to check the WBC daily. Weight gain is a common side effect and frequent snacks will aggravate this problem.

152.) Intravenous heparin therapy is prescribed for a client. While implementing this prescription, a nurse ensures that which of the following medications is available on the nursing unit? 1. Protamine sulfate 2. Potassium chloride 3. Phytonadione (vitamin K ) 4. Aminocaproic acid (Amicar)

1. Protamine sulfate Rationale: The antidote to heparin is protamine sulfate; it should be readily available for use if excessive bleeding or hemorrhage occurs. Potassium chloride is administered for a potassium deficit. Vitamin K is an antidote for warfarin sodium. Aminocaproic acid is the antidote for thrombolytic therapy.

A clinic nurse is reviewing the physician's prescription for a child who has been diagnosed with scabies. LINDANE has been prescribed for the child. The nurse questions the prescription if which of the following is noted in child's record? 1. The child is 18 months old. 2. The child is being bottle-fed. 3. A sibling is using lindane for the treatment of scabies. 4. The child has history of frequent respiratory infection.

1. The child is 18 months old. LINDANE is a pediculicide product that treats scabies. its contraindicated for children younger than 2 years b/c they have more permeable skin and high systemic absorption can occur placing them at risk for seizures and cps toxicity.

149.) A client taking fexofenadine (Allegra) is scheduled for allergy skin testing and tells the nurse in the health care provider's office that a dose was taken this morning. The nurse determines that: 1. The client should reschedule the appointment. 2. A lower dose of allergen will need to be injected. 3. A higher dose of allergen will need to be injected. 4. The client should have the skin test read a day later than usual.

1. The client should reschedule the appointment. Rationale: Fexofenadine is an antihistamine, which provides relief of symptoms caused by allergy. Antihistamines should be discontinued for at least 3 days (72 hours) before allergy skin testing to avoid false-negative readings. This client should have the appointment rescheduled for 3 days after discontinuing the medication.

221.) A nurse is reviewing the health care provider's prescriptions for an adult client who has been admitted to the hospital following a back injury. Carisoprodol (Soma) is prescribed for the client to relieve the muscle spasms; the health care provider has prescribed 350 mg to be administered four times a day. When preparing to give this medication, the nurse determines that this dosage is: 1. The normal adult dosage 2. A lower than normal dosage 3. A higher than normal dosage 4. A dosage requiring further clarification

1. The normal adult dosage Rationale: The normal adult dosage for carisoprodol is 350 mg orally three or four times daily.

3.) Salicylic acid is prescribed for a client with a diagnosis of psoriasis. The nurse monitors the client, knowing that which of the following would indicate the presence of systemic toxicity from this medication? 1. Tinnitus 2. Diarrhea 3. Constipation 4. Decreased respirations

1. Tinnitus Rationale: Salicylic acid is absorbed readily through the skin, and systemic toxicity (salicylism) can result. Symptoms include tinnitus, dizziness, hyperpnea, and psychological disturbances. Constipation and diarrhea are not associated with salicylism.

110.) A client taking lithium carbonate (Lithobid) reports vomiting, abdominal pain, diarrhea, blurred vision, tinnitus, and tremors. The lithium level is checked as a part of the routine follow-up and the level is 3.0 mEq/L. The nurse knows that this level is: 1. Toxic 2. Normal 3. Slightly above normal 4. Excessively below normal

1. Toxic Rationale: The therapeutic serum level of lithium is 0.6 to 1.2 mEq/L. A level of 3 mEq/L indicates toxicity.

195.) A nurse is caring for a client who is taking metoprolol (Lopressor). The nurse measures the client's blood pressure (BP) and apical pulse (AP) immediately before administration. The client's BP is 122/78 mm/Hg and the AP is 58 beats/min. Based on this data, which of the following is the appropriate action? 1. Withhold the medication. 2. Notify the registered nurse immediately. 3. Administer the medication as prescribed. 4. Administer half of the prescribed medication.

1. Withhold the medication. Rationale: Metoprolol (Lopressor) is classified as a beta-adrenergic blocker and is used in the treatment of hypertension, angina, and myocardial infarction. Baseline nursing assessments include measurement of BP and AP immediately before administration. If the systolic BP is below 90 mm/Hg and the AP is below 60 beats/min, the nurse should withhold the medication and document this action. Although the registered nurse should be informed of the client's vital signs, it is not necessary to do so immediately. The medication should not be administered because the data is outside of the prescribed parameters for this medication. The nurse should not administer half of the medication, or alter any dosages at any point in time.

176.) A nurse notes that a client is taking lansoprazole (Prevacid). On data collection, the nurse asks which question to determine medication effectiveness? 1. "Has your appetite increased?" 2. "Are you experiencing any heartburn?" 3. "Do you have any problems with vision?" 4. "Do you experience any leg pain when walking?"

2. "Are you experiencing any heartburn?" Rationale: Lansoprazole is a gastric acid pump inhibitor used to treat gastric and duodenal ulcers, erosive esophagitis, and hypersecretory conditions. It also is used to treat gastroesophageal reflux disease (GERD). It is not used to treat visual problems, problems with appetite, or leg pain. **NOTE: "-zole" refers to gastric acid pump inhibitors**

34.) A client with Crohn's disease is scheduled to receive an infusion of infliximab (Remicade). The nurse assisting in caring for the client should take which action to monitor the effectiveness of treatment? 1. Monitoring the leukocyte count for 2 days after the infusion 2. Checking the frequency and consistency of bowel movements 3. Checking serum liver enzyme levels before and after the infusion 4. Carrying out a Hematest on gastric fluids after the infusion is completed

2. Checking the frequency and consistency of bowel movements Rationale: The principal manifestations of Crohn's disease are diarrhea and abdominal pain. Infliximab (Remicade) is an immunomodulator that reduces the degree of inflammation in the colon, thereby reducing the diarrhea. Options 1, 3, and 4 are unrelated to this medication.

63.) A client with coronary artery disease complains of substernal chest pain. After checking the client's heart rate and blood pressure, a nurse administers nitroglycerin, 0.4 mg, sublingually. After 5 minutes, the client states, "My chest still hurts." Select the appropriate actions that the nurse should take. Select all that apply. 1. Call a code blue. 2. Contact the registered nurse. 3. Contact the client's family. 4. Assess the client's pain level. 5. Check the client's blood pressure. 6. Administer a second nitroglycerin, 0.4 mg, sublingually.

2. Contact the registered nurse. 4. Assess the client's pain level. 5. Check the client's blood pressure. 6. Administer a second nitroglycerin, 0.4 mg, sublingually. Rationale: The usual guideline for administering nitroglycerin tablets for a hospitalized client with chest pain is to administer one tablet every 5 minutes PRN for chest pain, for a total dose of three tablets. The registered nurse should be notified of the client's condition, who will then notify the health care provider as appropriate. Because the client is still complaining of chest pain, the nurse would administer a second nitroglycerin tablet. The nurse would assess the client's pain level and check the client's blood pressure before administering each nitroglycerin dose. There are no data in the question that indicate the need to call a code blue. In addition, it is not necessary to contact the client's family unless the client has requested this.

240.) A client with Parkinson's disease has been prescribed benztropine (Cogentin). The nurse monitors for which gastrointestinal (GI) side effect of this medication? 1. Diarrhea 2. Dry mouth 3. Increased appetite 4. Hyperactive bowel sounds

2. Dry mouth Rationale: Common GI side effects of benztropine therapy include constipation and dry mouth. Other GI side effects include nausea and ileus. These effects are the result of the anticholinergic properties of the medication. **Eliminate options 1 and 4 because they are comparable or alike. Recall that the medication is an anticholinergic, which causes dry mouth**

136.) A nurse performs an admission assessment on a client who visits a health care clinic for the first time. The client tells the nurse that propylthiouracil (PTU) is taken daily. The nurse continues to collect data from the client, suspecting that the client has a history of: 1. Myxedema 2. Graves' disease 3. Addison's disease 4. Cushing's syndrome

2. Graves' disease Rationale: PTU inhibits thyroid hormone synthesis and is used to treat hyperthyroidism, or Graves' disease. Myxedema indicates hypothyroidism. Cushing's syndrome and Addison's disease are disorders related to adrenal function.

41.) The client has been taking omeprazole (Prilosec) for 4 weeks. The ambulatory care nurse evaluates that the client is receiving optimal intended effect of the medication if the client reports the absence of which symptom? 1. Diarrhea 2. Heartburn 3. Flatulence 4. Constipation

2. Heartburn Rationale: Omeprazole is a proton pump inhibitor classified as an antiulcer agent. The intended effect of the medication is relief of pain from gastric irritation, often called heartburn by clients. Omeprazole is not used to treat the conditions identified in options 1, 3, and 4.

131.) The nurse is reinforcing medication instructions to a client with breast cancer who is receiving cyclophosphamide (Neosar). The nurse tells the client to: 1. Take the medication with food. 2. Increase fluid intake to 2000 to 3000 mL daily. 3. Decrease sodium intake while taking the medication. 4. Increase potassium intake while taking the medication.

2. Increase fluid intake to 2000 to 3000 mL daily. Rationale: Hemorrhagic cystitis is a toxic effect that can occur with the use of cyclophosphamide. The client needs to be instructed to drink copious amounts of fluid during the administration of this medication. Clients also should monitor urine output for hematuria. The medication should be taken on an empty stomach, unless gastrointestinal (GI) upset occurs. Hyperkalemia can result from the use of the medication; therefore the client would not be told to increase potassium intake. The client would not be instructed to alter sodium intake.

161.) A nurse is caring for a client with severe back pain, and codeine sulfate has been prescribed for the client. Which of the following would the nurse include in the plan of care while the client is taking this medication? 1. Restrict fluid intake. 2. Monitor bowel activity. 3. Monitor for hypertension. 4. Monitor peripheral pulses.

2. Monitor bowel activity. Rationale: While the client is taking codeine sulfate, an opioid analgesic, the nurse would monitor vital signs and monitor for hypotension. The nurse should also increase fluid intake, palpate the bladder for urinary retention, auscultate bowel sounds, and monitor the pattern of daily bowel activity and stool consistency (codeine can cause constipation). The nurse should monitor respiratory status and initiate breathing and coughing exercises. In addition, the nurse monitors the effectiveness of the pain medication.

48.) A client is to begin a 6-month course of therapy with isoniazid (INH). A nurse plans to teach the client to: 1. Drink alcohol in small amounts only. 2. Report yellow eyes or skin immediately. 3. Increase intake of Swiss or aged cheeses. 4. Avoid vitamin supplements during therapy.

2. Report yellow eyes or skin immediately. Rationale: INH is hepatotoxic, and therefore the client is taught to report signs and symptoms of hepatitis immediately (which include yellow skin and sclera). For the same reason, alcohol should be avoided during therapy. The client should avoid intake of Swiss cheese, fish such as tuna, and foods containing tyramine because they may cause a reaction characterized by redness and itching of the skin, flushing, sweating, tachycardia, headache, or lightheadedness. The client can avoid developing peripheral neuritis by increasing the intake of pyridoxine (vitamin B6) during the course of INH therapy for TB.

107.) A client receiving a tricyclic antidepressant arrives at the mental health clinic. Which observation indicates that the client is correctly following the medication plan? 1. Reports not going to work for this past week 2. Complains of not being able to "do anything" anymore 3. Arrives at the clinic neat and appropriate in appearance 4. Reports sleeping 12 hours per night and 3 to 4 hours during the day

3. Arrives at the clinic neat and appropriate in appearance Rationale: Depressed individuals will sleep for long periods, are not able to go to work, and feel as if they cannot "do anything." Once they have had some therapeutic effect from their medication, they will report resolution of many of these complaints as well as demonstrate an improvement in their appearance.

230.) A client is placed on chloral hydrate (Somnote) for short-term treatment. Which nursing action indicates an understanding of the major side effect of this medication? 1. Monitoring neurological signs every 2 hours 2. Monitoring the blood pressure every 4 hours 3. Instructing the client to call for ambulation assistance 4. Lowering the bed and clearing a path to the bathroom at bedtime

3. Instructing the client to call for ambulation assistance Rationale: Chloral hydrate (a sedative-hypnotic) causes sedation and impairment of motor coordination; therefore, safety measures need to be implemented. The client is instructed to call for assistance with ambulation. Options 1 and 2 are not specifically associated with the use of this medication. Although option 4 is an appropriate nursing intervention, it is most important to instruct the client to call for assistance with ambulation.

80.) A nurse is caring for a client who is taking phenytoin (Dilantin) for control of seizures. During data collection, the nurse notes that the client is taking birth control pills. Which of the following information should the nurse provide to the client? 1. Pregnancy should be avoided while taking phenytoin (Dilantin). 2. The client may stop taking the phenytoin (Dilantin) if it is causing severe gastrointestinal effects. 3. The potential for decreased effectiveness of the birth control pills exists while taking phenytoin (Dilantin). 4. The increased risk of thrombophlebitis exists while taking phenytoin (Dilantin) and birth control pills together.

3. The potential for decreased effectiveness of the birth control pills exists while taking phenytoin (Dilantin). Rationale: Phenytoin (Dilantin) enhances the rate of estrogen metabolism, which can decrease the effectiveness of some birth control pills. Options 1, 2, are 4 are not accurate.

129.) Megestrol acetate (Megace), an antineoplastic medication, is prescribed for the client with metastatic endometrial carcinoma. The nurse reviews the client's history and contacts the registered nurse if which diagnosis is documented in the client's history? 1. Gout 2. Asthma 3. Thrombophlebitis 4. Myocardial infarction

3. Thrombophlebitis Rationale: Megestrol acetate (Megace) suppresses the release of luteinizing hormone from the anterior pituitary by inhibiting pituitary function and regressing tumor size. Megestrol is used with caution if the client has a history of thrombophlebitis. **megestrol acetate is a hormonal antagonist enzyme and that a side effect is thrombotic disorders**

27.) Sildenafil (Viagra) is prescribed to treat a client with erectile dysfunction. A nurse reviews the client's medical record and would question the prescription if which of the following is noted in the client's history? 1. Neuralgia 2. Insomnia 3. Use of nitroglycerin 4. Use of multivitamins

3. Use of nitroglycerin Rationale: Sildenafil (Viagra) enhances the vasodilating effect of nitric oxide in the corpus cavernosum of the penis, thus sustaining an erection. Because of the effect of the medication, it is contraindicated with concurrent use of organic nitrates and nitroglycerin. Sildenafil is not contraindicated with the use of vitamins. Neuralgia and insomnia are side effects of the medication.

103.) A nurse is caring for a hospitalized client who has been taking clozapine (Clozaril) for the treatment of a schizophrenic disorder. Which laboratory study prescribed for the client will the nurse specifically review to monitor for an adverse effect associated with the use of this medication? 1. Platelet count 2. Cholesterol level 3. White blood cell count 4. Blood urea nitrogen level

3. White blood cell count Rationale: Hematological reactions can occur in the client taking clozapine and include agranulocytosis and mild leukopenia. The white blood cell count should be checked before initiating treatment and should be monitored closely during the use of this medication. The client should also be monitored for signs indicating agranulocytosis, which may include sore throat, malaise, and fever. Options 1, 2, and 4 are unrelated to this medication.

81.) A client with trigeminal neuralgia is being treated with carbamazepine (Tegretol). Which laboratory result would indicate that the client is experiencing an adverse reaction to the medication? 1. Sodium level, 140 mEq/L 2. Uric acid level, 5.0 mg/dL 3. White blood cell count, 3000 cells/mm3 4. Blood urea nitrogen (BUN) level, 15 mg/dL

3. White blood cell count, 3000 cells/mm3 Rationale: Adverse effects of carbamazepine (Tegretol) appear as blood dyscrasias, including aplastic anemia, agranulocytosis, thrombocytopenia, leukopenia, cardiovascular disturbances, thrombophlebitis, dysrhythmias, and dermatological effects. Options 1, 2, and 4 identify normal laboratory values.

You are providing care for a client with an acute HEMORRHAGIC stroke. The client's spouse tells you that he has been reading a lot about strokes and asks why his wife has not received ALTEPLASE (Activase). What is your best response? 1."Your wife was not admitted within the timeframe that alteplase is usually given." 2."This drug is used primarily for clients who experience an acute heart attack." 3."Alteplase dissolves clots and may cause more bleeding into your wife's brain." 4."Your wife had gallbladder surgery just 6 months ago, and this prevents the use of alteplase."

3."ALTEPLASE dissolves clots and may cause more bleeding into your wife's brain." Alteplase is a clot buster. In a client who has experienced hemorrhagic stroke, there is already bleeding into the brain. A drug such as alteplase can worsen the bleeding. The other statements about the use of alteplase are accurate but are not pertinent to this client's diagnosis

54.) A nurse reinforces discharge instructions to a postoperative client who is taking warfarin sodium (Coumadin). Which statement, if made by the client, reflects the need for further teaching? 1. "I will take my pills every day at the same time." 2. "I will be certain to avoid alcohol consumption." 3. "I have already called my family to pick up a Medic-Alert bracelet." 4. "I will take Ecotrin (enteric-coated aspirin) for my headaches because it is coated."

4. "I will take Ecotrin (enteric-coated aspirin) for my headaches because it is coated." Rationale: Ecotrin is an aspirin-containing product and should be avoided. Alcohol consumption should be avoided by a client taking warfarin sodium. Taking prescribed medication at the same time each day increases client compliance. The Medic-Alert bracelet provides health care personnel emergency information.

87.) A client with acute muscle spasms has been taking baclofen (Lioresal). The client calls the clinic nurse because of continuous feelings of weakness and fatigue and asks the nurse about discontinuing the medication. The nurse should make which appropriate response to the client? 1. "You should never stop the medication." 2. "It is best that you taper the dose if you intend to stop the medication." 3. "It is okay to stop the medication if you think that you can tolerate the muscle spasms." 4. "Weakness and fatigue commonly occur and will diminish with continued medication use."

4. "Weakness and fatigue commonly occur and will diminish with continued medication use." Rationale: The client should be instructed that symptoms such as drowsiness, weakness, and fatigue are more intense in the early phase of therapy and diminish with continued medication use. The client should be instructed never to withdraw or stop the medication abruptly, because abrupt withdrawal can cause visual hallucinations, paranoid ideation, and seizures. It is best for the nurse to inform the client that these symptoms will subside and encourage the client to continue the use of the medication.

163.) A client with trigeminal neuralgia tells the nurse that acetaminophen (Tylenol) is taken on a frequent daily basis for relief of generalized discomfort. The nurse reviews the client's laboratory results and determines that which of the following indicates toxicity associated with the medication? 1. Sodium of 140 mEq/L 2. Prothrombin time of 12 seconds 3. Platelet count of 400,000 cells/mm3 4. A direct bilirubin level of 2 mg/dL

4. A direct bilirubin level of 2 mg/dL Rationale: In adults, overdose of acetaminophen (Tylenol) causes liver damage. Option 4 is an indicator of liver function and is the only option that indicates an abnormal laboratory value. The normal direct bilirubin is 0 to 0.4 mg/dL. The normal platelet count is 150,000 to 400,000 cells/mm3. The normal prothrombin time is 10 to 13 seconds. The normal sodium level is 135 to 145 mEq/L.

56.) Heparin sodium is prescribed for the client. The nurse expects that the health care provider will prescribe which of the following to monitor for a therapeutic effect of the medication? 1. Hematocrit level 2. Hemoglobin level 3. Prothrombin time (PT) 4. Activated partial thromboplastin time (aPTT)

4. Activated partial thromboplastin time (aPTT) Rationale: The PT will assess for the therapeutic effect of warfarin sodium (Coumadin) and the aPTT will assess the therapeutic effect of heparin sodium. Heparin sodium doses are determined based on these laboratory results. The hemoglobin and hematocrit values assess red blood cell concentrations.

4.) The camp nurse asks the children preparing to swim in the lake if they have applied sunscreen. The nurse reminds the children that chemical sunscreens are most effective when applied: 1. Immediately before swimming 2. 15 minutes before exposure to the sun 3. Immediately before exposure to the sun 4. At least 30 minutes before exposure to the sun

4. At least 30 minutes before exposure to the sun Rationale: Sunscreens are most effective when applied at least 30 minutes before exposure to the sun so that they can penetrate the skin. All sunscreens should be reapplied after swimming or sweating.

237.) A client who is on lithium carbonate (Lithobid) will be discharged at the end of the week. In formulating a discharge teaching plan, the nurse will instruct the client that it is most important to: 1. Avoid soy sauce, wine, and aged cheese. 2. Have the lithium level checked every week. 3. Take medication only as prescribed because it can become addicting. 4. Check with the psychiatrist before using any over-the-counter (OTC) medications or prescription medications.

4. Check with the psychiatrist before using any over-the-counter (OTC) medications or prescription medications. Rationale: Lithium is the medication of choice to treat manic-depressive illness. Many OTC medications interact with lithium, and the client is instructed to avoid OTC medications while taking lithium. Lithium is not addicting, and, although serum lithium levels need to be monitored, it is not necessary to check these levels every week. A tyramine-free diet is associated with monoamine oxidase inhibitors.

The mother of a 6 year old child who has type 1 diabetes mellitus calls a clinic nurse and tells the nurse that the child has been sick. The mother reports that she checked the child's urine and it was positive for ketones. The nurse instructs the mother to: 1. Hold the next dose of insulin. 2. Come to the clinic immediately. 3. Administer an additional dose of regular insulin. 4. Encourage the child to drink calorie-free liquids.

4. Encourage the child to drink calorie-free liquids. when the child is sick, the mother should test for urinary ketones with each voiding. If ketones is present, liquids are essential to aid in clearing the ketones. The child should be encouraged to drink calorie-free liquids. Bringing the child to the clinic immediately is unnecessary. Insulin doses should not be adjusted or changed.

205.) A nurse is assisting in preparing to administer acetylcysteine (Mucomyst) to a client with an overdose of acetaminophen (Tylenol). The nurse prepares to administer the medication by: 1. Administering the medication subcutaneously in the deltoid muscle 2. Administering the medication by the intramuscular route in the gluteal muscle 3. Administering the medication by the intramuscular route, mixed in 10 mL of normal saline 4. Mixing the medication in a flavored ice drink and allowing the client to drink the medication through a straw

4. Mixing the medication in a flavored ice drink and allowing the client to drink the medication through a straw Rationale: Because acetylcysteine has a pervasive odor of rotten eggs, it must be disguised in a flavored ice drink. It is consumed preferably through a straw to minimize contact with the mouth. It is not administered by the intramuscular or subcutaneous route. **Knowing that the medication is a solution that is also used for nebulization treatments will assist you to select the option that indicates an oral route**

222.) A nurse has administered a dose of diazepam (Valium) to a client. The nurse would take which important action before leaving the client's room? 1. Giving the client a bedpan 2. Drawing the shades or blinds closed 3. Turning down the volume on the television 4. Per agency policy, putting up the side rails on the bed

4. Per agency policy, putting up the side rails on the bed Rationale: Diazepam is a sedative-hypnotic with anticonvulsant and skeletal muscle relaxant properties. The nurse should institute safety measures before leaving the client's room to ensure that the client does not injure herself or himself. The most frequent side effects of this medication are dizziness, drowsiness, and lethargy. For this reason, the nurse puts the side rails up on the bed before leaving the room to prevent falls. Options 1, 2, and 3 may be helpful measures that provide a comfortable, restful environment, but option 4 is the one that provides for the client's safety needs.

121.) A client who is taking hydrochlorothiazide (HydroDIURIL, HCTZ) has been started on triamterene (Dyrenium) as well. The client asks the nurse why both medications are required. The nurse formulates a response, based on the understanding that: 1. Both are weak potassium-losing diuretics. 2. The combination of these medications prevents renal toxicity. 3. Hydrochlorothiazide is an expensive medication, so using a combination of diuretics is cost-effective. 4. Triamterene is a potassium-sparing diuretic, whereas hydrochlorothiazide is a potassium-losing diuretic.

4. Triamterene is a potassium-sparing diuretic, whereas hydrochlorothiazide is a potassium-losing diuretic. Rationale: Potassium-sparing diuretics include amiloride (Midamor), spironolactone (Aldactone), and triamterene (Dyrenium). They are weak diuretics that are used in combination with potassium-losing diuretics. This combination is useful when medication and dietary supplement of potassium is not appropriate. The use of two different diuretics does not prevent renal toxicity. Hydrochlorothiazide is an effective and inexpensive generic form of the thiazide classification of diuretics. **It is especially helpful to remember that hydrochlorothiazide is a potassium-losing diuretic and triamterene is a potassium-sparing diuretic**

Calan (verapamil) is used for

Atrial fibrillation and flutter, SVT

Mechanism of action of acetaminophen

Blocks peripheral pain impulses by inhibition of prostaglandin synthesis. Also lowers febrile body temp by acting on the hypothalamus. Lacks anti-inflammatory effects.

Autonomic functions

Bodily functions that are involuntary and result from the physiologic activity of the autonomic nervous system. Function often occurs in pairs of opposing actions between the sympathetic and parasympathetic divisions of the autonomic nervous system.

Adverse effects of TCAs

Blockade of cholinergic receptors results in undesirable anticholinergic adverse effects, the mod common being constipation and urinary retention. Adrenergic and dopaminergic receptor blockade can lead to disturbances in cardiac coeducation and hypotension. Histaminergic blockade can cause sedation, and serotonergic blockade can alter the seizure threshold, and cause sexual dysfunction.

Mechanism of action of anorexiants

CNS stimulants that are believed to work by suppressing appetite control centers int eh brain. May also increase the body's basal metabolic rate, including mobilization of adipose tissue stores and enhanced cellular glucose uptake, as well as reduce dietary fat absorption. Phentermine, diethylpropion, methamphetamine, and benphetamine resemble amphetamine sulfate in their structures and CNS effects. Classified as both anorexiants and adrenergic drugs. All appear to suppress appetite centers in the CNS through dopamine- and norepinephrine-mediated pathways Orlistat differs from other anti obesity drugs in that it is not a CNS stimulant. Works by inhibiting the enzyme lipase, resulting in reduced absorption of dietary fat from the intestinal tract and increased fat elimination in the feces.

Analeptics

CNS stimulants that have generalized effects on the brainstem and spinal cord, which produce an increase in responsiveness to external stimuli and stimulate respiration. Include doxapram and the methylxanthines aminophylline, theophyline, and caffeine. Sometimes used to treat neonatal and postop reap depression. Neonatal uses are more common.

Adverse Effects of Aminoglutethimide

CNS: Ataxia Dizziness Sedation Loss of Energy Uncontrolled eye movements Anorexia N/V Maculopapular rash on the face and/or the palms of the hands Rarely: Fever Chills Sore throat caused by leukopenia or Agranulocytosis Jaundice of the eyes and skin Increased bleeding episodes Unusual bruising (thrombocytopenia) Usually dose related and may decline in w/ 2 to 6 weeks of continuous therapy. Drug may need to be discontinued if effects are severe.

Adverse effects of benzodiazepines

Can be harmful if given in excessive doses or when mixed with alcohol. Adverse effects usually involve CNS. Commonly reported effects: - headache - drowsiness - paradoxical excitement or nervousness - dizziness/vertigo - cognitive impairment - lethargy Can create a significant fall hazard in elderly patients. Hangover effect is sometimes reported.

Lab Test Interactions of Opioids

Can cause abnormal increase in the serum levels of: amylase alanine aminotransferase alkaline phosphatase bilirubin lipase creatinine kinase lactate dehydrogenase Decrease in urinary 17-ketosteroid levels and increase in urinary alkaloid and glucose concentrations.

Gabapentin (Neurontin)

Chemical analogue of GABA, a neurotransmitter that inhibits brain activity. Exact mechanism of action is unknown. Believed to work by increased synthesis and synaptic accumulation of GABA between neurons. Indicate as an adjunct drug for the treatment of partial seizures and for prophylaxis of partial seizures. Also shown to be effective as single-drug therapy for new-onset epilepsy. Most commonly used to treat neuropathic pain. Contraindications include known allergy. Adverse effects include CNS and GI symptoms.

Ergot Alkaloids

Drugs that narrow or constrict blood vessels in the brain and provide relief of pain for certain migraine headaches.

Anesthetics

Drugs that reduce or eliminate pain by depressing nerve function in the CNS and/or the PNS.

CNS Stimulants

Drugs that stimulate specific areas of the brain or spinal cord. Most act by stimulating the excitatory neurons in the brain. These neurons contain receptors for excitatory neurotransmitters, including dopamine, norepinephrine, and serotonin.

Hypnotic

Drugs that, when given at low to moderate dosages, calm the CNS. Can cause sleep at high doses and are much more potent than sedatives

Adjunct anesthetis

Drugs used in combination with anesthetic drugs to control the adverse effects of anethetics or to help maintain

SNRIs

Duloxetine (Cymbalta) Venlafaxine (Effexor) reduces pain: fibromyalgia, diabetic neuropathy

Lithium

Effective for the treatment of bipolar disorder; thought to potentiate serotonergic neurotransmission. Research indicates that lithium ions alter sodium transport in nerve cells, which results in a shift in catecholamine metabolism. Small therapeutic index. Indicated for the treatment of manic episodes in bipolar disorder as well as for maintenance therapy to prevent such episodes. Contraindications to lithium therapy are relative and include: dehydration, known sodium imbalance, and major renal or cardiovascular disease, bc all of these increase risk of lithium toxicity. Toxicity can manifest as: Gi discomfort, tremor, confusion, somnolence, seizures, possible death. Long term treatment can cause hyperthyroidism. Potentially interacting drugs include thiazide diuretics, ACE inhibitors, NSAIDs, all of which increase lithium toxicity.

Tricyclic Antidepressants

Effective, but used judiciously due to side effects and lethal overdose

Antiinflammatory actions of glucocorticoids

Esp. cortisol in > normal dosages, can stabilize lysosomal membranes and prevent the release of proteolytic enzymes during inflammation. Can also potentiate vasoconstrictor effects.

Secondary or Symptomatic epilepsy

Epilepsy with a distinct cause, such as trauma, infection, cerebrovascular disorder, or other illness. Chief causes in children an infants are developmental defects, metabolic disease, and injury at birth. Chief causes in adults is acquired brain disorder such as head injury, metabolic disorders, adverse drug reactions, primary or metastatic brain tumor, or other nonspecific neuro diseases.

Which meds interact with sumatriptan (imitrex) for migraines?

Ergot Alkaloids MAOIs SSRIs must wait 2 weeks after

Common Adverse Effects of Glucocorticoids

Euphoria, increased appetite, insomnia, restlessness, anxiety, gas, hyper pigmentation, HTN, headache, hirsutism, lowered resistance to infections, visual disturbances (cataracts), increased urination or thirst, decreased growth in children. Anorexia may occur with traimcinolone. Redness, swelling, rash, pain, tingling, or numbness may occur at the injection site. Chronic use may result in abd pain, acne, GI bleeding, peptic ulcers, round face (Cushing's), HTN, edema, weight gain, muscle cramps, weakness, irregular heart rate, N/V, bone pain, increased bruising, and wounds that are difficult to heal.

REM Rebound

Excessive REM sleep following dc of a sleep-altering drug.

Adverse effects of Adjunct antiepileptic drugs

Gabapentin: - dizziness, drowsiness, nausea, visual and speech changes, edema Pregabalin: - dizziness, drowsiness, peripheral edema, blurred vision. Lamotrigine: - Drowsiness, ataxia, headache, nausea, blurred or double vision Levetiracetam: - dizziness, drowsiness, hyperactivity, behavior changes such s anxiety, hostility, agitation, or suicidal ideation, uncoordination Succinimides (ethosuximide): - nausea, abd pain, dizziness, drowsiness Tiagabine: - dizziness, drowsiness, agitation, asthenia, GI upset, abd pain, rash tremor Topiramate: - dizziness, drowsiness, GI upset, ataxia Zonisamide: - drowsiness, anorexia, ataxia, confusion, agitation, cognitive impairment

Epilepsy

General term for any of a group of neurologic disorders characterized by recurrent episodes of convulsive seizures, sensory disturbances, abnormal behavior, loss of consciousness, or any combo of these.

Midazolam

Generic name for the drug that is most often used for moderate sedation procedures

Administration of glucocorticoids

Give glucocorticoids as a single daily dose in the morning (before 9 am if possible) and with food or milk. Administer IM injections of suspensions deep in the gluteal muscle to avert local tissue atrophy at the injection sites. Injections in the deltoid muscle can cause atrophy.

PEG-3350

Go-Lytely for bowel preperations: onset: 30-60 mins duration 4 hours

Fludrocortisone

Has potent mineralocorticoid activity w/ minimal glucocorticoid effects. It acts primarily on the renal distal tubule to reabsorb sodium and enhance the excretion of potassium and hydrogen.

Irreversible cholinesterase inhibitors

Have a long duration of activity, and the body must generate new cholinesterase enzymes to override the effects of the irreversible drugs.

Drug Interactions of AEDs

Increased hydantoin levels occur with disulfiram, isoniazid, and valproic acid. Increased risk for seizures occurs with tricyclic antidepressants.

Mechanism of action CNS stimulants

Increases pulse rate and BP, and can lead to seizures, intracerebral bleeding, and toxicity (due to decreased drug metabolism and excretion). Stimulation of the respiratory system is actually desirable, and this action is beneficial in those patients suffering from CNS depression, such as postop.

Levetiracetam (Keppra)

Indicated as adjunct therapy for partial seizures w/ and w/o secondary generalization. Contraindicated in cases of known allergy. Mech of action is unknown. Most common adverse effects r/t CNS. No drug interactions are currently listed, potential for CNS depression when combined with other sedating drugs.

Tiagabine (Gabitril)

Indicated as adjunct therapy for partial seizures. Contraindications include known drug allergy. Exact mechanism of action has not been identified, but is known to have beneficial effects by inhibiting the reuptake of GABA from the neuronal synapses in the brain. There have been several case reports of paradoxical seizures in non epileptic patines who are treated with the drug for other indications. Common adverse effects are CNS and GI symptoms. Drug interactions chiefly involve other CNS depressant drugs.

Lamotrigine (Lamictal)

Indicated for simple or complex partial seizures, for generalized seizures r/t Lennox-Gastaut syndrome (atypical form of absence epilepsy that may persist into adulthood), and most recently for primary generalized tonic-clonic seizures. Also for bipolar disorder. No known contraindications other than allergy. Common adverse effects include relatively minor CNS and GI symptoms. One potentially serious adverse effect is a rash that can progress to the major derm reaction Stevens-Johnson syndrome. This condition involves inflammation and sloughing of skin, potentially over the entire body, in a manner that resembles a third-degree burn. Often reversible but can also be fatal.

Phenytoin

Indicated for the management of tonic-clonic and partial seizures. Contraindications include known drug allergy and heart conditions that involve bradycardia or blockage of electrocardiac function. Adverse effects and drug interactions both are numerous. Most common adverse effects are lethargy, abnormal movements, mental confusion, and cognitive changes. Gingival hyperplaseia is a well-knwon adverse effect of long-term oral phenytoin therapy. Long-term oral therapy can also cause acne, hirsutism, and hypertrophy of subcut facial tissue resulting in an appearance known as Dilantin facies, osteoporosis. at toxic levels of phenytoin can cause nystagmus, ataxia, dysarthria, and encehalopathy. Can interact with other medications for two main reasons: 1. It is highly bound to plasma proteins and competes with other highly protein-bound meds for binding sites. 2. Induces hepatic microsomal enzymes, mainly cytochrome P-450 enzymes, which increases the metabolism of other drugs that are metabolized by these enzymes and reduces their blood levels. Low serum albumin concentrations may cause exaggerated phenytoin effects.

Fentanyl

Indications: Moderate to severe pain. Methods of admin: - buccal lozenges esp. helpful w/ breakthrough pain. - Injectable form most common in preoperative settings and ICU for sedation during mechanical ventilation. - Oral/transdermal primarily for long-term control of both malignant and nonmalignant chronic pain. - New patch to be reapplied q72hr

codeine sulfate

Indications: Pain, Antitussive Adverse effects: GI tract upset is most common. Duration of action: 4-6 hours

morphine sulfate

Indications: Severe pain Method of admin: Oral, IV, rectal Adverse effects/contraindications: If given in presence of renal impairment, accumulation of morphine-6-glucoronide may occur. Duration of action: 6-7 hours

Prednisone (Deltasone, Prednisone Intensol, Apo-Prednisone, Winpred)

Indications: adrenocortical insufficiency states, inflammation disorders, and organ transplant. Used to treat a heat number of respiratory, GI, rheumatologic, dermatologic, ocular, and neoplastic conditions in which moderate to sever inflammation is a component. Dosing: Well absorbed orally. Metabolized in the liver to the the active prednisolone and excreted but he kidneys. Onset is 1-2 hours, Duration is 30-36 hours. Adverse Effects: insomnia, nervousness, increased appetite, GI upset, dizziness, glucose intolerance, joint discomfort, seizures, mood changes, hallucinations, edema, HTN, nose bleed, and potential to aggravate cataracts and glaucoma. Osteoporosis w/ chronic use. Prednisone metabolism can be accelerated w/ concurrent barbiturate, phenytoin, or rifampin therapy.

Mechanism of action and effects of SSRIs

Inhibit serotonin reuptake, may also have weak effects on norepinephrine and dopamine reuptake

Mechanism of action and effects of SNRIs

Inhibit the reuptake of both serotonin and norepinephrine.

Primacor (Milrinone)

Is a phosphodiesterase inhibitor used in HF. (positive inotropic effects) Monitor for ventricular dysrhythmias, hypotension, angina, hypokalemia, tremor, and thrombocytopenia do not administer in same IV line as lasix

in reviewing the morning lab data, a nurse finds that a Pt's serum K+ level is 6.9. Which tx's will decrease the serum K+ level? SATA

KAYEXALATE-Promote the exchange of potassium for sodium in (GI) system CALCIUM CHLORIDE - Reduces the risk of ventricular fibrillation caused by hyperkalemia REGULAR INSULIN-Facilitates the uptake of glucose into the cell, which brings potassium with it SODIUM BICARBONATE-Alkalinizing agents: Increases the pH, which results in a temporary potassium shift from the extracellular to the intracellular environment; these agents enhance the effectiveness of insulin in patients with acidemia

Contraindications for 2nd gen antidepressants

Known allergy MAOIs therapy w/ certain antipsychotic drugs such as thioridazine or mesoridazine Significant history of cardiac disease or seizures may be a contraindication due to the relatively uncommon but reported cardiac effects and alterations in seizure threshold. Bupropion s also contraindicated in eating disorder patients and seizure disorders bc it can lower the seizure threshold.

Contraindications to benzodiazepines

Known allergy Narrow-angle glaucoma Pregnancy

Contraindications to acetaminophen

Known allergy Severe liver disease Glucose-6-phosphate dehydrogenase deficiency (G6PD deficiency)

Contraindications to the use of opioid analgesics

Known drug allergy Severe asthma **Itching is pharmacological effect of morphine due to histamine release, NOT ALLERGY.

Contraindications for TCAs

Known drug allergy Use of MAOIs within the previous 14 days Pregnancy Also not recommended in patients with any acute or chronic cardiac problems or history of seizures, bc both conditions are associated with a greater likelihood of death upon TCA OD.

Contraindications to barbiturate use

Known drug allergy pregnancy significant respiratory difficulties severe kidney or liver disease Must be used with caution in elderly patients due to their sedative properties and increased fall risk.

Non-REM Sleep

Largest portion of sleep cycle. Four stages, and precedes REM sleep. Most of a normal sleep cycle consists of non-REM.

What type of antacid causes diarrhea

Magnesium: milk of magnesia

Toxicity and management of adrenergic OD

Majority of adrenergic compounds have very short half-lives, and thus their effects are short-lived. Therefore, when these drugs are taken in OD or toxicity develops, stopping the drug causes the toxic symptoms to subside in a relatively short period of time. Recommended treatment for OD is often to manage the symptoms and support the patient.

Osmotic Diuretics

Mannitol (rapid diuresis)

SSRI Nursing Implications

Monitor the patient's mental status. ¬ Monitor for postural hypotension. ¬ Administer with meals to minimize gastrointestinal upset.

Phophorated carbohydrate solution (Emetrol)

Mint-flavored oral solution to relive nausea

Phenobarbital

Most commonly prescribed barbiturate, either alone or in combo with other drugs. Classified as a long-acting drug. Used for prevention of generalized tonic-clonic seizures and fever-induced convulsions. Has been useful in treatment of hyperbilirubinemia in neonates.

ibuprofen

NSAID, take with meals

What is administered to reverse respiratory depression

Naloxone (narcan): an opioid reversal agent

Phantom Pain

Occurs in the area of a body part that has been removed, surgically or traumatically, and is often described as burning, itching, tingling, or stabbing. Can also occur in paralyzed limbs following spinal cord injury.

Referred Pain

Occurs when visceral nerve fibers synapse at a level in the spinal cord close to fibers that supply specific subcutaneous tissues in the body.

Central Pain

Occurs with tumors, trauma, inflammation, or disease affecting CNS tissues.

When a child receives medication for ADHD, what will the nurse instruct the caregivers to closely monitor?

Physical Growth and weight

Neuromuscular Blocking Drugs NMBDs

Prevent nerve transmission in skeletal and smooth muscle, leading to paralysis: First sensation is muscle weakness, followed by total flaccid paralysis, then the intercostal muscles and diaphram are paralyzed leading to respiratory arrest. *Succinylcholine: depolarizing, *Pancuronium: non depolarizing

Indications for Muscle Relaxants

Primarily used for the relief of painful musculoskeletal conditions such as muscle spasms, often following injuries such as low back strain. Most effective when used in conduction with physical therapy. May also be used in the management of spasticity associated with severe chronic disorderd such as MS and other types of cerebral lesions, cerebral palsy, and rheumatic disorders.

Adverse Effects of cholinergic drugs

Primary adverse effects of cholinergic drugs are the consequence of overstimulation of the parasympathetic nervous system. They are extensions of the cholinergic reactions that affect many body functions. -Cardiovascular:bradycardia/tachycardia, hypotension, hypertension, syncope, conduction abnormalities (AV block and cardiac arrest) -CNS:Headache, dizzness, convulsions, ataxia -GI:Abd cramps, increased secretions, N/V, diarrhea -Respiratory: Increased bronchial secretions, bronchospasm -Other: Lacrimation, sweating, salivation, miosis

Adverse effects of alpha blockers

Primary effects are those related to their effects on the vasculature. -First-dose phenomenon: severe and sudden drop in BP after admin of first dose of an alpha blocker; can cause patient to fall or pass out. -Orthostatic hypotension; warn to change positions slowly. -Palpitations, tachycardia, edema, chest pain -dizziness, headache, anxiety, depression, weakness, numbness, fatigue -N/V, diarrhea, constipation, abd pain -Incontinence, dry mouth, pharyngitis.

Gamma-aminobutyric acid (GABA)

Primary neurotransmitter found in the brain. Key compound affected by sedative, anxiolytic, psychotropic, and muscle relaxing meds.

Phenobarbital and Primidone

Primidone is metabolized in the liver to phenobarbital and phenylethylmalonamide, both of which have anticonvulsant properties. Can provide anticonvulsant activity w/ a lower serum level of phenobarbital than that attained with phenobarbital itself, which can reduce the likelihood of sedation and fatigue associated w/ phenobarbital. Phenobarbital is used for the management of status epileptics and is an effective prophylactic drug for the control of febrile seizures. Use of oral phenobarbital for seizure prevention is much less common. Most common adverse effect of phenobarbital is sedation, though tolerance usually develops. Long half-life allows once a day dosing. Contraindications incude known drug allergy, porphyria, liver or kidney impairment, and respiratory illness. Adverse effects include cardiovascular, CNS, GI, and dermatologic reactions. Is a major inducer of hepatic enzymes, which causes more rapid clearance of some drugs.

Overall Effect of general anesthetics

Progressive reduction of sensory and motor CNS functions. General anesthesia initially produces a loss of the senses of sight, touch, taste, smell, and hearing, along w/ a loss of consciousness. Mechanical vent is absolutely necessary.

Glucocorticoids in fat metabolism

Promote mobilization of fatty acids from adipose tissue, increasing their concentration in the plasma and their use for energy. Despite this, client on glucocorticoids may accumulate fat stores (rounded buffalo hump).

Adverse effects of barbiturates

Relate to CNS and include: drowsiness lethargy dizziness hangover (prolongation of the three) paradoxical restlessness or excitement Long-term effects on normal sleep architecture can be detrimental bc barbiturates deprive people of REM sleep. Rebound REM can occur when barbiturate is stopped.

Muscle relaxants are most effective when combined with

Rest and Physical Therapy Dantrolen (Dantrium) Baclofen Cyclobenzaprine (Flexeril)

Neuropathic pain

Results from damage to peripheral or CNS nerve fibers

Cancer Pain

Results from pressure of the tumor against organs, nerves, or tissues or from radiation/chemotherapry

Laxative that increases osmotic pressure

Saline

Mechanism of action and drug effects of amphetamines

Stimulate ares of the brain associated with mental alertness, such as the cerebral cortex and the thalamus. Pharmacologic actions of the CNS stimulants are similar to the actions of the sympathetic nervous system in that the CNS and respiratory systems are the primary systems affected. CNS effects include mood elevation or euphoria, increased mental alertness and capacity for work, decreased fatigue and drowsiness, and prolonged wakefulness. Resp effects most commonly seen are relaxation of bronchial smooth muscle, increased respiration, and dilation of pulmonary arteries.

Potential Opioid Adverse Effects and their Management (3 of 3)

Subacute Overdose: May be more common than acute respiratory depression and may progress slowly (over hours to days), with somnolence and respiratory depression. Before analgesic dosages are changed or reduced, advancing disease must be considered, especially in the dying patient. Often holding one or two doses is enough to judge if mental and respiratory depression is associated with the opioid. If there is improvement with this measure, the opioid dosage is often decreased by 25%. Other: Dry mouth, urinary retention, pruritus, myoclonus, dysphoria, euphoria, sleep disturbances, sexual dysfunction, and inappropriate secretion of antidiuretic hormone may occur but are less common than other adverse effects. Ongoing assessment is needed for each of the adverse effects so that appropriate measures may be implemented.

Zonisamide (Zonegran)

Sulfonamide derivative indicated for a variety of seizure types, including partial and secondary generalized, primary generalized, absence, and myoclonic. Contraindicated in patients with known allergy to the drug or to sulfa drugs. Common adverse effects include CNS and GI symptoms. Interacts with a number of drugs metabolized by cytochrome P-450 enzymes, which increase or decrease clearance of zonisamide.

Contraindications to antiepileptic use

Usually only drug allergy Pregnancy is a common contraindication; however, prescriber must consider the risks to mother and infant of untreated maternal epilepsy and the increased risk for seizure activity. Newer generation drugs appear to be safer in pregnancy than the traditional drugs.

Ethosuximide (Zarontin)

Used in the treatment of uncomplicated absence seizures. Not effective for secondary generalized tonic-clonic seizures. Only listed contraindication for either use is known allergy to succinimides. Adverse effects include GI and CNS effects. Drug interactions most commonly involve hepatic enzyme-inducing drugs.

Valproic acid

Used primarily in the treatment of generalized seizures (absence, myoclonic, and tonic-clonic). Also used for bipolar disorder and has been shown to be effective in controlling partial seizures. Contraindications include allergy, liver impairment, and urea cycle disorders. Common adverse effects include drowsiness, N/V, and other GI disturbances, tremor, weight gain, and transient hair loss. Most serious effects are hepatotoxicity and pancreatitis. Can interact with many meds, mainly bc of protein binding and liver metabolism. It is highly bound to plasma proteins and competes with other highly protein-bound meds for binding sites. Also metabolized by hepatic microsomal enzymes and competes for metabolism with other drugs. Not a hepatic enzyme inducer.

Anorexiants

Used to control or suppress appetite; effectiveness hasn't been proved yet. Include: phentermine (Ionamin) benzphetamine (Didrex) methamphetamine (Desoxyn) diethylpropion (Tenuate)

Indications for Fludrocortisone use

Used to treat Addison disease (chronic primary adrenocortical insufficiency) and congenital adrenogenital syndrome. Has also been used to treat symptomatic orthostatic hypotension.

Gate Theory

Uses analogy of a gate to describe how impulses from damaged tissues are sensed in the brain.

Neuropathic Pain

Usually results from damage to peripheral or CNS nerve fibers by disease or injury but may also be idiopathic. Common symptoms include hypersensitivity or hyperalgesia to mild stimuli such as light touch or a pinprick, or the bed sheets on a person's feet. Also known as allodynia. Can also manifest as hyperalgesia to uncomfortable stimuli, such as inflated BP cuff on a patient's limb. May be described as heat, cold, numbness and tingling, burning, or electrical sensations.

Indications for amphetamine use

Varios amphetamine derivatives, including methylphenidate: - ADHD - Narcolepsy Desmethylphenidate: - ADHD Benzphetamine and methamphetamine: - Obesity Nonamphetamine stimulant modafinil: - Narcolepsy

What is the antidote for warfarin

Vitamin K (phytonadione)

The provider orders digoxin and furosemide every day. which of these foods would the nurse reinforce for the client to eat at least daily? 1. watermelon 2. chicken 3. tomato

WATERMELON Watermelon is high in potassium and will replace potassium lost by the diuretic. The other foods are not high in potassium.

Interactions with adrenergics

When alpha- and beta-adrenergic drugs are given w/ adrenergic antagonists, the drugs directly antagonize each other, which results in reduced therapeutic effects. Concurrent admin of adrenergics w/ anesthetics can increase risk of cardiac dysrhythmias. Tricyclic antidepressants, when given with adrenergics, can cause increased vasopressor effects, acute hypertensive crisis. Concurrent admin of adrenergics with MAOIs may cause a possibly life-threatening hypertensive crisis. Antihistamines and thyroid preparations can also increase the effects of adrenergic drugs.

Hypnotics

When given at low to moderate dosages, calm or soothe the CNS w/o inducing sleep but when given at high dosages cause sleep.

Education - Glucocorticoids/corticosteroids

With systemic administration, instruct to report any signs of infection, such as sore throat, fever, and poor wound healing. Corticosteroids can mask infection and increase its spread. Client should individuals w/ known contagious illnesses. Advise that glucocorticoids impair the antibody response and that the response to immunizations may be reduced. Report any visual disturbances; cataracts and glaucoma can occur with long-term glucocorticoid therapy. Reports any persistent GI symptoms. Client should follow a high-potassium, low-sodium diet to counter the potassium-depleting and sodium-retaining effects of the drug. Limit alcohol, caffeine, aspiring, and other gastric irritants. Report any symptoms of and pain, infection, bone pain, tiredness, bruising, or tarry stools. Report changes in mental status (euphoria, mood swings, depression), or insomnia to the prescriber. Female clients may experience menstrual irregularities. All unsafe during pregnancy.

Because of the risks associated with administration of factor VIII concentrate, the nurse would teach the client's family to recognize and report which of the following?

Yellowing of the skin Because factor VIII concentrate is derived from large pools of human plasma, the risk of hepatitis is always present. Clinical manifestations of hepatitis include yellowing of the skin, mucous membranes, and sclera. Use of factor VIII concentrate is not associated with constipation, abdominal distention, or puffiness around the eyes.

Your patient is about to be prescribed rifampin because she was exposed to Neisseria meningitides by a housemate. Your patient is undergoing methadone maintenance therapy for a heroin addition. You know that the rifampin may:

You know that the rifampin may: induce the metabolism of methadone and she may have symptoms of withdrawal.

ethambutol

anti-tuberculars report to the doc if client has difficulty discriminating red and green may lead to optic neuritis

isoniazid

anti-tuberculars, assess for signs of drug induced hepatitis ex jaundice

aluminum hydroxide

anti-ulcer take one hour after meals

sulfasalazine

anti-ulcer, tx of inflammatory bowel disease and arthritis, continue even after symptoms subside

ranitidine

anti-ulcer,avoid NSAIDS

phenytoin toxicity

antiarrhythmics, anticonvulsant symptoms include ataxia, slurred speech, nystagmus

medication: digoxin

antibodies

carbamazepine tx

anticonvulsant, mood stabilizer tx trigeminal neuralgia = pain relief

amitriptyline hydrochloride

antidepressant

fluoxetine

antidepressant SSRI be aware of suicidal behavior for the first 4 weeks

medication: acetaminophen

antidote: acetylcysteine

medication: mestonin

antidote: atropine sulfate

medication: magnesium sulfate

antidote: calcium gluconate

medication: penicillin

antidote: epinephrine

medication: curare

antidote: tensilon

medication: oxytocin

antidote: terbutaline

haloperidol

antipsychotic at risk for neuroleptic malignant syndrome; a temp of 102 is a medical emergency. Also treats tourette syndrome.

208.) A client with myasthenia gravis verbalizes complaints of feeling much weaker than normal. The health care provider plans to implement a diagnostic test to determine if the client is experiencing a myasthenic crisis and administers edrophonium (Enlon). Which of the following would indicate that the client is experiencing a myasthenic crisis? 1. Increasing weakness 2. No change in the condition 3. An increase in muscle spasms 4. A temporary improvement in the condition

auto-define "A client with myasthen..." Rationale: Edrophonium (Enlon) is administered to determine whether the client is reacting to an overdose of a medication (cholinergic crisis) or to an increasing severity of the disease (myasthenic crisis). When the edrophonium (Enlon) injection is given and the condition improves temporarily, the client is in myasthenic crisis. This is known as a positive test. Increasing weakness would occur in cholinergic crisis. Options 2 and 3 would not occur in either crisis.

Nursing Implications of Opioids

¬ Assess vital signs (blood pressure, pulse, respirations, and pain level) prior to administration. ¬ Withhold medication if respiratory rate is <12 breaths/minute or systolic blood pressure is <90. ¬ Administer oral medications with food to decrease nausea and vomiting. ¬ Monitor urinary output and bowel status. ¬ Institute safety precautions (i.e., side rail up, call bell in place).


संबंधित स्टडी सेट्स

Prelude: And just what is Geology?

View Set

08 Salesforce Admin Exam Prep: Data Management

View Set

what is a cover letter? // how to write professional emails

View Set

TX-Brokerage (SAE), Texas Real Estate Brokerage 30 Hours 2019, texas sae real estate brokerage, Real Estate: Brokerage Final Exam Questions/Answers, Real Estate: Practice Exam (Questions/Answers), Real Estate: Agency Exam Questions/Answers, Real Esta...

View Set

新航道‧雅思真詞彙 - Level 1 (yet)

View Set